You are on page 1of 441

CÂU HỎI LÍ THUYẾT HÓA HỌC HỮU CƠ OLYMPIAVN

Phần 3

Hợp chất
tạp chức
Lời mở đầu
Thân chào quý độc giả.

Giai đoạn trước 2018, Tạp chí Olympiad Hóa học KEM (tiền thân là box Hóa
học OlympiaVN) có biên soạn một số chuyên đề Hóa học hữu cơ - từ chính
các câu hỏi được thảo luận trên diễn đàn hoặc bài tập do thanh viên đề xuất
- và tổng hợp lại thành bộ sách Hóa học "Hữu cơ OlympiaVN".

Bộ sách này tuy có lượng kiến thức tương đối nhiều, nhưng lại có một số
khiếm khuyết tương đối quan trọng. Điểm đầu tiên là tài liệu chỉ mới dừng ở
mức "tổng hợp", chưa có sự sắp xếp về mặt nội dung và trình độ khó-dễ, gây
ra những khó khăn cho độc giả, đặc biệt là các bạn mới tiếp xúc với Hóa học
hữu cơ. Khiếm khuyết thứ hai là còn thiếu những phần kiến thức quan trọng
(như các chuyên đề hợp chất thiên nhiên) hoặc chưa bổ sung kiến thức mới
(như ứng dụng lí thuyết FMO).

Bởi vậy, trong năm 2019 này, đội ngũ biên tập viên của KEM đã nỗ lực để
hoàn thành bộ sách này - lấy tên là "Câu hỏi lí thuyết Hóa học Hữu cơ
OlympiaVN". Bên cạnh việc giữ lại những tinh túy của bộ sách "Hữu cơ
OlympiaVN" cũ (khoảng 20% các bài tập hay) thì KEM còn bổ sung thêm rất
nhiều kiến thức mới (khoảng 80%), mà đa số được chọn lọc và sắp xếp lại từ
các giáo trình, bài giảng Hóa hữu cơ ở bậc Đại học hoặc tài liệu bồi dưỡng
Olympiad của Nhật Bản và các quốc gia châu Âu (như Ukraine, Ba Lan). Tuy
vẫn còn nhiều điểm han chế, như sự thiếu thống nhất về mặt danh pháp (sử
dụng nhiều nguồn tài liệu khác nhau) và hình vẽ chưa được trau chuốt, nhưng
bộ sách này đã mang đến sự cải thiện rất lớn về mặt nội dung lẫn bố cục -
để giúp các bạn học sinh, sinh viên ngành Hóa đón nhận kiến thức dễ dàng
hơn.

Nhằm tránh sự trùng lặp nội dung với các bộ sách "50 chuyên đề Olympiad
Hóa học" và "Tự học Hóa" mà KEM đã biên soạn vào năm 2018, 2019 - trong
bộ sách này, KEM chỉ tập trung vào những vấn đề lớn mà các bộ sách trên
còn thiếu, bao gồm 3 chủ đề chính:

- Cấu trúc hợp chất hữu cơ.


- Cơ chế phản ứng.
- Hợp chất tạp chức.

Mong rằng, trong quá trình đọc sách, nếu có vấn đề gì sai sót hoặc cần góp
ý thêm cho KEM, độc giả sẽ gửi ý kiến qua địa chỉ mail:
tapchikem@gmai.com. Trân trọng cảm ơn!
Mục lục
Monosaccharide ......................................................................................................................... 10

Bài 1 ...................................................................................................................................10

Bài 2 ..................................................................................................................................12

Bài 3 ..................................................................................................................................15

Bài 4 ..................................................................................................................................18

Bài 4A .............................................................................................................................. 20

Bài 5 ................................................................................................................................... 3

Bài 6 ................................................................................................................................... 5

Bài 7 ................................................................................................................................... 9

Bài 8 .................................................................................................................................. 11

Bài 9 ..................................................................................................................................14

Bài 10 ................................................................................................................................16

Bài 11 ................................................................................................................................. 17

Bài 12 ................................................................................................................................19

Bài 13 ............................................................................................................................... 20

Bài 14 ............................................................................................................................... 22

Bài 15 ............................................................................................................................... 27

Bài 16 ............................................................................................................................... 29

Bài 17 ................................................................................................................................ 30

Bài 17A ..............................................................................................................................31

Bài 18 ............................................................................................................................... 33

Bài 19 ............................................................................................................................... 36

Bài 19A ............................................................................................................................. 38

Bài 20 ............................................................................................................................... 40

Bài 21 ............................................................................................................................... 42
Bài 22 ............................................................................................................................... 43

Bài 23 ............................................................................................................................... 44

Bài 23A ............................................................................................................................ 45

Bài 23B ............................................................................................................................ 47

Bài 24 ............................................................................................................................... 49

Bài 25 ................................................................................................................................51

Bài 26 ............................................................................................................................... 55

Bài 27 ............................................................................................................................... 56

Bài 28 ............................................................................................................................... 60

Bài 29 ............................................................................................................................... 63

Bài 30 ............................................................................................................................... 65

Bài 31 ............................................................................................................................... 68

Bài 32 ............................................................................................................................... 72

Bài 33 ............................................................................................................................... 74

Di-, tri- và polysaccharide ......................................................................................................... 75

Bài 1 .................................................................................................................................. 75

Bài 2 ................................................................................................................................. 80

Bài 3 ..................................................................................................................................81

Bài 4 ................................................................................................................................. 83

Bài 5 ................................................................................................................................. 86

Bài 6 ................................................................................................................................. 87

Bài 6A .............................................................................................................................. 90

Bài 7 ................................................................................................................................. 93

Bài 8 ................................................................................................................................. 95

Bài 9 ................................................................................................................................. 97

Bài 10 ............................................................................................................................. 104

Bài 11 .............................................................................................................................. 108


Bài 12 ............................................................................................................................... 111

Bài 13 .............................................................................................................................. 114

Bài 14 .............................................................................................................................. 116

Bài 15 .............................................................................................................................. 118

Glycoside .................................................................................................................................... 119

Bài 1 ................................................................................................................................. 119

Bài 2 ............................................................................................................................... 124

Bài 3 ............................................................................................................................... 128

Bài 4 ............................................................................................................................... 129

Bài 5 ............................................................................................................................... 132

Bài 6 ............................................................................................................................... 135

Bài 7 ............................................................................................................................... 140

Bài 8 ............................................................................................................................... 143

Bài 9 ............................................................................................................................... 144

Bài 10 ............................................................................................................................. 146

Bài 11 .............................................................................................................................. 148

Bài 12 ............................................................................................................................. 150

Bài 13 ............................................................................................................................. 154

Bài 14 ............................................................................................................................. 155

Bài 15 ..............................................................................................................................157

Bài 16 ............................................................................................................................. 162

Amino acid ................................................................................................................................ 168

Bài 1 ................................................................................................................................ 168

Bài 2 ................................................................................................................................170

Bài 3 ................................................................................................................................172

Bài 4 ................................................................................................................................ 177

Bài 5 ............................................................................................................................... 180


Bài 6 ............................................................................................................................... 183

Bài 7 ............................................................................................................................... 188

Bài 8 ................................................................................................................................ 191

Bài 9 ............................................................................................................................... 192

Bài 10 ............................................................................................................................. 194

Bài 11 ..............................................................................................................................200

Bài 12 .............................................................................................................................202

Bài 13 .............................................................................................................................203

Peptide.......................................................................................................................................204

Bài 1 ................................................................................................................................204

Bài 2 ............................................................................................................................... 207

Bài 3 ...............................................................................................................................209

Bài 4 ............................................................................................................................... 213

Bài 5 ................................................................................................................................217

Bài 6 ............................................................................................................................... 219

Bài 7 ...............................................................................................................................220

Bài 8 ...............................................................................................................................223

Bài 9 ...............................................................................................................................226

Bài 10 ............................................................................................................................. 227

Bài 11 ..............................................................................................................................229

Bài 12 .............................................................................................................................230

Bài 13 ............................................................................................................................. 231

Bài 14 .............................................................................................................................233

Bài 15 .............................................................................................................................236

Bài 16 .............................................................................................................................238

Bài 17 ..............................................................................................................................240

Bài 18 .............................................................................................................................242
Bài 19 .............................................................................................................................246

Bài 20 .............................................................................................................................249

Bài 21 .............................................................................................................................253

Bài 22 .............................................................................................................................259

Bài 23 .............................................................................................................................263

Bài 24 .............................................................................................................................266

Bài 25 .............................................................................................................................268

Bài 26 ..............................................................................................................................271

Bài 27 ............................................................................................................................. 272

Bài 28 ............................................................................................................................. 275

Bài 29 ............................................................................................................................. 278

Phụ lục: Các chuyên đề hóa sinh & polymer .......................................................................283

Hóa sinh ....................................................................................................................................284

Bài 1 ................................................................................................................................284

Bài 2 ...............................................................................................................................288

Bài 3 ...............................................................................................................................290

Bài 4 ...............................................................................................................................294

Bài 5 ............................................................................................................................... 301

Bài 6 ...............................................................................................................................303

Bài 7 ...............................................................................................................................308

Bài 8 ............................................................................................................................... 310

Bài 9 ............................................................................................................................... 314

Bài 10 ............................................................................................................................. 318

Bài 11 ..............................................................................................................................322

Bài 12 .............................................................................................................................324

Bài 13 ............................................................................................................................. 327

Bài 14 .............................................................................................................................335
Bài 15 .............................................................................................................................338

Bài 16 .............................................................................................................................344

Bài 17 ..............................................................................................................................348

Bài 18 .............................................................................................................................355

Bài 19 .............................................................................................................................358

Polymer......................................................................................................................................362

Bài 1 ................................................................................................................................362

Bài 2 ................................................................................................................................371

Bài 3 ............................................................................................................................... 374

Bài 4 ............................................................................................................................... 375

Bài 5 ............................................................................................................................... 377

Bài 6 ...............................................................................................................................380

Bài 7 ...............................................................................................................................382

Bài 8 ...............................................................................................................................384

Bài 9 ...............................................................................................................................386

Bài 10 .............................................................................................................................390

Bài 11 ..............................................................................................................................393

Bài 12 ............................................................................................................................. 397

Bài 13 .............................................................................................................................400

Bài 14 .............................................................................................................................406

Bài 15 .............................................................................................................................409

Bài 16 ............................................................................................................................. 412

Bài 17 .............................................................................................................................. 415

Bài 18 ............................................................................................................................. 418


Monosaccharide
Bài 1
1) Xác định cấu hình R/S của các tâm thủ tính (chiral) sau:

2) Vẽ lại cấu trúc các đường sau ở dạng công thức chiếu Newman và cho
biết tên thông thường của chúng.

Hướng dẫn
1)
2)
Bài 2
Do tất cả các carbohydrate đều có các nguyên tử
carbon quang hoạt nên cần có phương pháp tiêu
chuẩn để biểu diễn chúng. Một trong những phương
pháp thường được dùng là công thức chiếu Fischer,
lấy glyceraldehyde tồn tại trong tự nhiên làm tiêu
chuẩn để kí hiệu tâm lập thể.
Các đường ngang biểu diễn các liên kết hướng ra
ngoài mặt phẳng giấy, các đường dọc biểu diễn các liên kết hướng vào
trong mặt phẳng giấy. Các đường có cùng cấu hình như D-glyceraldehyde
ở nguyên tử carbon ở xa nhóm carbonyl nhất (nhóm hydroxyl ở nguyên tử
quang hoạt thấp nhất hướng sang phải) thì được gọi là đường D. Ngược
lại với đường D là các đường L có nhóm hydroxyl ở nguyên tử carbon
quang hoạt thấp nhất hướng sang trái trong hình chiếu Fischer.
1) Sử dụng quy tắc Cahn, Ingold, Prelog để xác định liệu D-glyceraldehyde
có cấu hình R hay S?
2) Vẽ công thức chiếu Fischer của các đồng phân lập thể của 2,3,4-
trihydroxypentane. Những chất nào là đối quang? Những chất nào là
đồng phân dia? Những chất nào quang hoạt?
Hình dưới biểu diễn hình chiếu Fischer của D-glucose.

3) Gọi tên hệ thống của D-glucose, sử dụng quy tắc Cahn, Ingold, Prelog.
Hướng dẫn
1) (2R)-2,3-dihydroxypropanal.
2) Các cấu trúc giống nhau: a = e, b = f, g = h, c = d

Các đối quang:

Quang hoạt Quang hoạt


Đồng phân dia:

Dạng meso Dạng meso


3) (2R, 3S, 4R, 5R)-2,3,4,5,6-Pentahydroxyhexanal.
Bài 3
1) Phân loại mỗi carbohydrate sau là aldose hay ketose, và chèn vào
thuật ngữ phù hợp để chỉ số nguyên tử carbon của chúng, ví dụ
aldopentose.

2) Chỉ rõ mỗi carbohydrate sau là đường D hay đường L và gắn cấu hình
R/S cho mỗi tâm thủ tính. Bạn có nhận ra bất kì xu hướng nào để gán
cấu hình nhanh hơn cho mỗi tâm thủ tính của carbohydrate không?

3) Xác định cấu hình mỗi tâm thủ tính trong các hợp chất sau:

4) Phân loại các monosaccharide sau thuộc loại D hay L, là đường aldo
hay keto, và là tetrose, pentose hay hexose?
5) Xác định tên gọi các aldohexose sau:

6) Với mỗi hợp chất sau, hãy xác định liệu chúng là đối quang, epimer,
đồng phân dia mà không phải epimer, hay là các hợp chất giống nhau:

Hướng dẫn
1) (a) aldohexose; (b) aldopentose; (c) ketopentose; (d) aldotetrose; (e)
ketohexose.
2) Tất cả đều là đường D, ngoại trừ (b) là đường L. Cấu hình của mỗi tâm
thủ tính là:
(a) 2S, 3S, 4R, 5R
(b) 2R, 3S, 4S
(c) 3R, 4R
(d) 2S, 3R
(e) 3S, 4S, 5R
Bạn có thể nhận thấy cấu hình của mỗi tâm thủ tính là R khi nhóm OH ở
bên phải của công thức chiếu Fischer, và cấu hình là S khi nhóm OH ở bên
trái.
3)

4) Phân loại:
(a) D-aldotetrose
(b) L-aldopentose
(c) D-aldopentose
(d) D-aldohexose
(e) D-ketopentose
5) Tên gọi:
(a) D-glucose
(b) D-mannose
(c) D-galactose
(d) L-glucose
6)
(a) Epimer
(b) Đồng phân dia
(c) Đối quang
(d) Cùng một chất (-D-glucopyranose).
Bài 4
1) Vẽ công thức chiếu Haworth cho các hợp chất sau:
(a) β-D-Galactopyranose
(b) α-D-Mannopyranose
(c) α-D-Allopyranose
(d) β-D-Mannopyranose
(e) β-D-Glucopyranose
(f) α-D-Glucopyranose
2) Xác định tên gọi hoàn chỉnh cho hợp chất sau:

3) Sự nghịch đảo gây ra sự chuyển hóa β-D-mannopyranose thành α-D-


mannopyranose. Sử dụng công thức Haworth, vẽ cân bằng giữa hai
dạng pyranose và dạng mạch hở của D-mannose.
4) Khi hoa tan D-talose trong nước, một cân bằng sẽ được thiết lập, trong
đó có sự hiện diện của hai dạng pyranose. Vẽ các dạng pyranose và
gọi tên chúng.
Hướng dẫn
1)
a) b) c)

d) e) f)
2) Cấu trúc này biểu diễn đồng phân β anomer của dạng vòng của D-
galactose và do đó có tên hoàn chỉnh là β-D-galactopyranose.
3)

4)
Bài 4A
Đưa ra các cấu trúc phù hợp để làm ví dụ minh họa cho mỗi trường hợp
sau:
(a) Một aldopentose
(b) Một ketohexose
(c) Một L-monosaccharide
(d) Một glycoside
(e) Một aldonic acid
(f) Một aldaric acid
(g) Một aldonolactone
(h) Một pyranose
(i) Một furanose
(j) Một đường khử
(k) Một pyranoside
(l) Một furanoside
(m) Các epimer
(n) Các anomer
(o) Một phenylosazone
(p) Một disaccharide
(q) Một polysaccharide
Hướng dẫn
1 | Bản quyền thuộc về Tạp chí Olympiad Hóa học KEM
2 | Bản quyền thuộc về Tạp chí Olympiad Hóa học KEM
Bài 5
1) Cấu dạng bền nhất của đa số aldopyranose là cấu dạng mà trong đó
nhóm lớn nhất, -CH2OH, ở vị trí biên (equatorial). Tuy nhiên, D-
idopyranose chủ yếu tồn tại ở cấu dạng với nhóm -CH2OH trục (axial).
Biểu diễn hai cấu dạng ghế của α-D-idopyranose (một với nhóm -
CH2OH ở vị trí trục, một với nhóm -CH2OH ở vị trí biên) và giải thích cho
sự chiếm ưu thế của cấu dạng trục.
2) Một trong hai anomer của methyl 2,3-anhydro-D-ribofuranoside, I, là
dạng β có nhiệt độ nóng chảy thấp hơn đến mức đáng kinh ngạc (so
với dạng còn lại). Giải thích dựa vào cấu trúc của chúng.

Hướng dẫn
1) Cấu dạng của D-idopyranose với bốn nhóm OH biên và một nhóm
CH2OH trục bền hơn cấu dạng với bốn nhóm OH trục và một nhóm
CH2OH biên.

3 | Bản quyền thuộc về Tạp chí Olympiad Hóa học KEM


2) Dạng β-anomer có thể tạo thành liên kết hydrogen nội phân tử:

Ngược lại, dạng α-anomer chỉ tạo thành liên kết hydrogen liên phân tử, dẫn
đến nhiệt độ sôi cao hơn nhiều.

4 | Bản quyền thuộc về Tạp chí Olympiad Hóa học KEM


Bài 6
2) Vẽ cấu trúc hemiacetal vòng được tạo thành khi xử lí mỗi hợp chất
lưỡng chức sau với dung dịch acid:

3) Vẽ cấu trúc hemiacetal vòng được tạo thành khi xử lí các hợp chất
lưỡng chức sau với dung dịch acid:

4) Xác định hydroxyaldehyde bị vòng hóa trong môi trường acid tạo thành
hemiacetal sau:

5) Hợp chất dưới đây có một nhóm aldehyde và hai nhóm OH. Trong môi
trường acid, một trong hai nhóm OH có thể đóng vai trò như một
nucleophile và tác kích nhóm carbonyl, nên có thể tạo thành hai kích
thước vòng.
a) Bỏ qua yếu tố hóa lập thể, hãy vẽ hai vòng có thể tạo thành.
b) Sự biến dạng vòng có mối liên hệ với các kích thước vòng khác nhau.
Hãy dự đoán trong trường hợp này thì hemiacetal vòng nào sẽ chiếm
ưu thế.

5 | Bản quyền thuộc về Tạp chí Olympiad Hóa học KEM


6) Chất nào sau đây có thể nghịch quay?

7) Chất nào sau đây có thể nghịch quay?

8) Mặc dù các ketone thường bền hơn đồng phân aldehyde xấp xỉ 12 kJ
mol-1, tuy nhiên fructose-6-phosphate lại kém bền hơn glucose-6-
phosphate khoảng 1.7 kJ mol-1. Giải thích tại sao.

6 | Bản quyền thuộc về Tạp chí Olympiad Hóa học KEM


Hướng dẫn
1) Chúng ta sử dụng hệ thống đánh số để xác định kích thước của vòng
được tạo thành. Ví dụ với chất (a) thì bốn nguyên tử carbon và một nguyên
tử oxygen kết hợp thành vòng năm cạnh, có hai nhóm methyl ở C4 sẽ phải
được biểu diễn trong sản phẩm. Các chất còn lại cũng tương tự.
a) b)

c) d)

2)
(a)

(b)

(c)

3)

7 | Bản quyền thuộc về Tạp chí Olympiad Hóa học KEM


4) Nhóm carbonyl có thể bị tác kích bởi nhóm OH-C4 tạo thành vòng năm
cạnh:

Hoặc bị tác kích bởi nhóm OH-C5, tạo thành vòng sáu cạnh:

Vòng sáu cạnh chiếm ưu thế do nó có biến dạng vòng ít hơn vòng năm
cạnh.
5) Hợp chất a và hemiacetal và b là hemiketal, Do đó chúng có thể nghịch
quay, còn c là ketal và không thể nghịch quay.
6) Chất a có tâm hemiacetal ở vòng bên phải, có thể nghịch quay. Hợp
chất b có các tâm acetal trên cả hai vòng và không thể nghịch quay.
7) Glucose-6-phosphate và fructose-6-phosphate không tồn tại trong
dung dịch ở dạng mạch hở, mà ở dạng các dẫn xuất hemiacetal và
hemiketal vòng. Do đó, độ bền tương đối phản ánh những khác biệt khác
giữa các dẫn xuất vòng, như cấu trúc vòng và các tương tác không gian.

8 | Bản quyền thuộc về Tạp chí Olympiad Hóa học KEM


Bài 7
Đường kính là sucrose tinh khiết, được tìm thấy trong củ cải đường và mía.
Trong nước, sucrose bị thủy phân thành dung dịch của D-glucose và D-
fructose. Hỗn hợp này được gọi là “đường nghịch chuyển”. Phản ứng thủy
phân có thể được khảo sát bởi phân cực kế. Dung dịch sucrose có tính
hữu triền (quay phải) (   D = 66.53o cm3 g-1), trong khi đó dung dịch
20

đường nghịch chuyển lại có tính tả triền (quay trái) (   D của glucose và
20

fructose lần lượt là +52.7 o và -92.4o cm3 g-1). Tốc độ thủy phân phụ thuộc
nhiều vào nồng độ hydrogen ion.
1) Viết phương trình phản ứng (sơ đồ phản ứng) thủy phân sucrose, sử
dụng công thức chiếu Haworth để biểu diễn tác nhân và sản phẩm
phản ứng.
Sự nghịch chuyển sucrose được khảo sát ở 25 oC. Các giá trị sau đây của
góc quay, αt, ở dạng hàm của thời gian, t, quan sát được là:

2) α∞ tương ứng với chuyển hóa hoàn toàn. Do đó αt - α∞ là thước đo


nồng độ của sucrose. Sử dụng các giá trị cho ở trên để chứng minh
rằng phản ứng có bậc 1 theo nồng độ sucrose, và tính hằng số tốc độ.
3) Trong HCl 0.1 M, tốc độ tăng gấp đôi khi tăng nhiệt độ từ 25 lên 30 oC.
Tính năng lượng hoạt hóa của phản ứng.

9 | Bản quyền thuộc về Tạp chí Olympiad Hóa học KEM


Hướng dẫn
1)

2) ln(αt - α∞) ở dạng hàm của thời gian là một đường thẳng, có nghĩa đây
là phản ứng bậc 1. Độ dốc là -0.0053 min-1.

3)

10 | Bản quyền thuộc về Tạp chí Olympiad Hóa học KEM


Bài 8
Sự chuyển hóa [bởi tác động của] vi khuẩn của saccharose tạo thành (S)-
(+)-2-hydroxypropanoic acid (L-(+)- lactic acid), chất này tạo thành ester
vòng từ 2 phân tử. Dilactide này có thể bị trùng hợp tạo thành một
polylactide, được sử dụng trong phẫu thuật.
1) Xác định cấu trúc không gian và công thức chiếu của L-(+)-lactic acid
và dilactide của nó.
2) Vẽ cấu trúc của polylactide được nói đến ở trên (với ít nhất 3 mắt xích).
Xác định tính điều hòa của nó (isotactic, syndiotactic hay atatic?).
3) Vẽ công thức các dilactide đồng phân được tạo thành từ racemic của
lactic acid. Chỉ rõ cấu hình của các tâm chiral.
L-(+)-lactic acid được sử dụng để điều chế thuốc diệt cỏ Barnon. Trong
trường hợp này, (+)-lactic acid được ester hóa với 2-propanol, sau đó nhóm
hydroxyl được xử lí với methanesulfonyl chloride. Sau đó, sản phẩm tạo
thành tham gia vào phản ứng thế SN2 với 3-fluoro-4-chloro-phenylamine,
trong đó nhóm methanesulfonate rời đi ở dạng CH3SO3-. Cuối cùng, nhóm
benzoyl được đưa vào qua phản ứng với benzoyl chloride.
4) Hoàn thành chuỗi phản ứng. Biểu diễn các sản phẩm ở dạng công thức
chiếu Fischer.

11 | Bản quyền thuộc về Tạp chí Olympiad Hóa học KEM


Hướng dẫn
1)

2)

3)

12 | Bản quyền thuộc về Tạp chí Olympiad Hóa học KEM


4)

13 | Bản quyền thuộc về Tạp chí Olympiad Hóa học KEM


Bài 9
1,3-Dihydroxyacetone có thể được chuyển hóa thành glyceraldehyde. Sau
một khoảng thời gian, glyceraldehyde này có thể chuyển thành dimer vòng
6 cạnh C6H12O6. Phổ hồng ngoại của dimer này cho thấy không có peak
hấp thụ giữa 1600-1800 cm-1, và moment lưỡng cực của dimer được xác
định là bằng 0.
1) Vẽ công thức chiếu Fischer của glyceraldehyde tạo thành và chỉ rõ cấu
hình của nó là dạng D(+) hay L(-).
2) Xác định công thức cấu tạo của trung gian phản ứng trong chuyển hóa
của 1,3- dihydroxyacetone thành glyceraldehyde.
3) Viết công thức cấu tạo của dimer tạo thành.
4) Sử dụng công thức chiếu Haworth, hãy biểu diễn các đồng phân lập
thể có thể có phù hợp với dữ kiện về moment lưỡng cực.
5) Xác định cấu hình tuyệt đối (R/S) của mỗi nguyên tử chiral carbon
trong các công thức ở trên.
Hướng dẫn
1)

2)

3)

14 | Bản quyền thuộc về Tạp chí Olympiad Hóa học KEM


4)

5)

15 | Bản quyền thuộc về Tạp chí Olympiad Hóa học KEM


Bài 10
Có thể chuyển hóa R(+) – glyxerandehit thành axit(-)-malic (axit 2-
hydroxibutandioic) qua con đường như sau: Đầu tiên xử lý R(+) –
glyxerandehit với hydro xianua để thu được hai đồng phân A và B có thể
tách ra được bằng phương pháp kết tinh phân đoạn. Chỉ B được chuyển
hóa thành C (C4H8O5) mà khi oxy hóa chất này bởi axit nitric cho D
(C4H6O6). Xử lý tiếp D bằng photpho tribromua cho E (C4H5BrO5) để cuối
cùng chuyển thành axit(-)-malic bằng cách khử với kẽm trong axit.
Viết các phản ứng xảy ra ở dạng công thức chiếu Fischer. Giải thích tại
sao chỉ duy nhất E được tạo thành từ D. Cho biết nếu sử dụng A thay cho
B thì sẽ tạo thành bao nhiêu đồng phân chất E ?
Hướng dẫn

Phản ứng chuyển từ D sang E là SN2 nên quay cấu hình. Chính vì vậy nếu
sử dụng đồng phân A sẽ tạo hai đồng phân E do lúc này dạng meso D sẽ
chuyển thành dạng đối quang có hai C* mang cấu hình khác nhau, tức sản
phẩm thu được nhiều khả năng là hỗn hợp raxemic.

16 | Bản quyền thuộc về Tạp chí Olympiad Hóa học KEM


Bài 11
1) Vẽ cấu dạng ghế bền hơn của mỗi hợp chất sau:
(a) β-D-Galactopyranose
(b) α-D-Glucopyranose
(c) β-D-Glucopyranose
2) Vẽ dạng mạch hở của monosaccharide vòng sau:

3) β-D-glucopyranose có hai cấu dạng ghế. Vẽ và xác định cấu dạng kém
bền hơn.
Hướng dẫn
1)
a) b) c)

2)

3) Cấu dạng kém bền hơn là cấu dạng có tất cả các nhóm thế chiếm vị trí
trục (axial).

17 | Bản quyền thuộc về Tạp chí Olympiad Hóa học KEM


18 | Bản quyền thuộc về Tạp chí Olympiad Hóa học KEM
Bài 12
Hai liên kết carbon-fluorine trong phân tử A dưới đây có độ dài khác nhau,
chênh lệch khoảng 0.015 Å. Dựa vào quan điểm thuyết FMO, hãy giải thích
quan sát này và chỉ rõ liên kết C-F nào dài hơn.

Hướng dẫn

19 | Bản quyền thuộc về Tạp chí Olympiad Hóa học KEM


Bài 13
1) Xét cấu trúc của hai D-aldotetrose sau:

Mỗi hợp chất này tồn tại ở dạng vòng furanose, được tạo thành khi OH ở
C4 tác kích vào nhóm aldehyde. Vẽ cấu trúc các vòng furanose sau:
(a) α-D-Erythrofuranose
(b) β-D-Erythrofuranose
(c) α-D-Threofuranose
(d) β-D-Threofuranose
2) Vẽ cơ chế cho phản ứng vòng hóa xúc tác acid của L-threose thành β-
L-threofuranose.
3) Vẽ cơ chế cho phản ứng vòng hóa xúc tác acid của D-fructose thành
β-D-fructofuranose.
Hướng dẫn
1)
a) b) c) d)

2) Trước tiên nhóm carbonyl bị proton hóa, khiến cho nó có tính


electrophile mạnh hơn và phù hợp hơn cho sự tác kích nucleophile bởi một
trong các nhóm OH. Nhóm OH ở C4 sẽ tác kích nhóm carbonyl được
proton hóa tạo thành một vòng furanose (năm cạnh). Sự deproton hóa
(với nước đóng vai trò như một base) tạo thành sản phẩm.

20 | Bản quyền thuộc về Tạp chí Olympiad Hóa học KEM


3)

21 | Bản quyền thuộc về Tạp chí Olympiad Hóa học KEM


Bài 14
1) Vẽ sản phẩm nhận được khi xử lí mỗi hợp chất sau với acetic anhydride
khi có mặt pyridine:
(a) α-D-Galactopyranose
(b) α-D-Glucopyranose
(c) β-D-Galactopyranose
2) Vẽ sản phẩm nhận được khi xử lí mỗi hợp chất ở ý 1 với methyl iodide
khi có mặt silver oxide (Ag2O).
3) Vẽ các sản phẩm dự đoán tạo thành khi xử lí α-D-galactopyranose với
lượng dư methyl iodide khi có mặt silver oxide, sau đó xử lí với dung
dịch acid.
4) Khi xử lí D-glucose với dung dịch nước bromine (trong đệm pH 6) thì
tạo thành một aldonic acid gọi là D-gluconic acid. Xử lí D-gluconic acid
với xúc tác acid tạo thành một lactone (ester vòng) có vòng sáu cạnh.
a) Vẽ cấu trúc D-gluconic acid.
b) Vẽ cấu trúc lactone tạo thành từ D-gluconic acid, biểu diễn cấu
hình mỗi tâm thủ tính.
c) Dự đoán lactone này có tính quang hoạt không?
d) Giải thích làm thế nào mà bạn có thể phân biệt được D-gluconic
acid và lactone bởi phổ IR.
5) Glycoside nào được tạo thành khi xử lí mỗi monosaccharide này với
CH3CH2OH, HCl: (a) β-D-mannose; (b) α-D-gulose; (c) β-D-fructose?
6) Trình bày cơ chế phản ứng sau:

22 | Bản quyền thuộc về Tạp chí Olympiad Hóa học KEM


Hướng dẫn
1) Khi xử lí với acetic anhydride và pyridine, tất cả các nhóm OH bị acetyl
hóa:
(a)

(b)

(c)

23 | Bản quyền thuộc về Tạp chí Olympiad Hóa học KEM


2) Khi xử lí với lượng dư methyl iodide khi có mặt silver oxide, tất cả các
nhóm OH bị chuyển thành nhóm methoxy:
(a)

(b)

(c)

3)

24 | Bản quyền thuộc về Tạp chí Olympiad Hóa học KEM


4)
a)

b)

c) Hợp chất này có các tâm thủ tính và không phải hợp chất meso. Do đó
nó quang hoạt.
d) Gluconic acid là một carboxylic acid và trong phổ IR được dự đoán sẽ
có một tín hiệu rộng trong khoảng 2200 và 3600 cm-1. Phổ IR của
lactone thì không có tín hiệu rộng này.

25 | Bản quyền thuộc về Tạp chí Olympiad Hóa học KEM


5)

6)

26 | Bản quyền thuộc về Tạp chí Olympiad Hóa học KEM


Bài 15
Glucozơ phản ứng với metanol trong HCl để sinh ra metyl glucopyranozơ.
a) Đề nghị cơ chế phản ứng này.
b) Trên lý thuyết, sản phẩm thu được sẽ là hỗn hợp giữa hai đồng
phân α và β-metyl glycozit. Tuy nhiên thực tế chỉ thu được α-
glycozit. Hãy giải thích điều này.
c) Đề nghị một phương thức tổng hợp chọn lọc β-D-metyl
glucopyranozơ từ glucozơ.
Hướng dẫn
a) Cơ chế phản ứng như sau:

b) Có thể giải thích theo một trong ba cách sau:


Cách 1: Obitan p của carbocation phải nằm thẳng góc với cặp electron n
của oxy để tạo được sự liên hợp bền.
Cách 2: Sản phẩm α-glycozit không xảy ra sự đẩy lưỡng cực giữa cặp
electron n của oxy và liên kết C-O như sản phẩm β-glycozit.

Cách 3: Tồn tại tương tác n - *C-O giữa cặp e của O1 với liên kết C2 –
O(CH3).

27 | Bản quyền thuộc về Tạp chí Olympiad Hóa học KEM


c) Sơ đồ tổng hợp.

28 | Bản quyền thuộc về Tạp chí Olympiad Hóa học KEM


Bài 16
a) Đề xuất cơ chế cho phản ứng dưới đây:

b) Sử dụng những hiểu biết về lí thuyết FMO và các hiệu ứng điện tử, hãy
giải thích sự tạo thành ưu tiên của anomer ở trên.
Hướng dẫn
a)

b) Phản ứng xúc tác


acid tạo thành acetal
như mô tả ở trên là một
phản ứng thuận
nghịch, tạo thành sản
phẩm bền nhiệt động.
α-anomer bền nhiệt
động hơn β-anomer do chỉ có α-anomer được hưởng lợi từ sự nhường
[donation] electron của cặp electron chưa liên kết trên oxygen vào liên kết
C-O liền kề.

29 | Bản quyền thuộc về Tạp chí Olympiad Hóa học KEM


Bài 17
Xác định các sản phẩm được tạo thành khi xử lí β-D-galactose với mỗi tác
nhân sau:
a) Ag2O + CH3I
b) NaH + C6H5CH2Cl
c) Sản phẩm ở (b), sau đó với H3O+
d) Ac2O + pyridine
e) C6H5COCl + pyridine
f) Sản phẩm ở (c), sau đó với C6H5COCl + pyridine
Hướng dẫn

30 | Bản quyền thuộc về Tạp chí Olympiad Hóa học KEM


Bài 17A
-D-(+)-mannopyranose là một epimer của -D-(+)-glucopyranose.
- Vẽ cấu dạng ghế bền nhất của nó.
- Xác định sản phẩm của -D-(+)-mannopyranose với các tác nhân sau:
a) Cu2+ (trong hệ đệm pH >7)
b) Br2, H2O (pH = 6)
c) HNO3
d) CH3OH, HCl khan
e) sản phẩm phản ứng d + (CH3)2SO2, NaOH
f) 1. NaBH4; 2. H2O
g) 5 đương lượng HIO4
h) acetic anhydride dư trong pyridine
i) 3 đương lượng phenylhydrazine, H+
j) 1. Br2/H2O; 2. Fe(III) sulfate, H2O2
k) 1. HCN; 2. Ba(OH)2; 3. H3O+; 4. Na-Hg, H2O, pH 3-5
Hướng dẫn

a) b) c)

d) e) f)

31 | Bản quyền thuộc về Tạp chí Olympiad Hóa học KEM


g) h) i)

j) k)

32 | Bản quyền thuộc về Tạp chí Olympiad Hóa học KEM


Bài 18
1) Nguyên liệu đầu để tổng hợp thương mại vitamin C là L-sorbose. Nó
có thể được tổng hợp từ D-glucose qua chuỗi phản ứng dưới đây. Giai
đoạn thứ hai của chuỗi phản ứng này minh họa việc sử dụng quá trình
oxid hóa xúc tác bởi vi khuẩn; vi sinh vật A. suboxydans hoàn thành
giai đoạn này với hiệu suất 90%. Kết quả tổng thể của quy trình tổng
hợp là chuyển hóa của một D-aldohexose (D-glucose) thành một L-
ketohexose (L-sorbose). Điều này có ý nghĩa gì về tính đặc hiệu của
quá trình oxid hóa vi khuẩn?

2) Hai aldose nào cũng tạo thành phenylosazone như L-sorbose?


3) Ngoài fructose và sorbose, có hai 2-ketohexose khác là psicose and
tagatose. D-psicose tạo thành cùng phenylosazone như D-allose
(hoặc D-altrose); D- tagatose tạo thành cùng osazone như D-galactose
(hoặc D-talose). Xác định cấu trúc của D-psicose và D-tagatose.
4) A, B, C là các aldohexose. Hợp chất A và B tạo thành cùng alditol
quang hoạt khi chúng bị khử bởi hydrogen xúc tác; A và B tạo thành
các phenylosazone khác nhau khi xử lí với phenylhydrazine; B và C tạo
thành phenylosazone giống nhau nhưng alditol khác nhau. Giả sử
chúng đều là đường D, xác định tên và cấu trúc của A-C.
Hướng dẫn
1) Vi sinh vật oxid hóa chọn lọc nhóm -CHOH của D-glucitol tương ứng
với C5 của D-glucose.

33 | Bản quyền thuộc về Tạp chí Olympiad Hóa học KEM


2) L-Gulose và L-idose

3)

34 | Bản quyền thuộc về Tạp chí Olympiad Hóa học KEM


4) A = D-altrose, B = D-talose, C = D-galactose

35 | Bản quyền thuộc về Tạp chí Olympiad Hóa học KEM


Bài 19
D-idose có cấu hình ngược với D-glucose ở C-2, C-3, C-4.
D-idose tồn tại trong một cân bằng của hai dạng pyranose (75 %) và
furanose (25 %).
a) Vẽ cấu dạng vòng cyclohexane của các đồng phân anomer  và -D-
idopyranose. Theo bạn thì dạng nào bền nhất với mỗi anomer? Tại
sao?
b) Qua chuyển hoá Lobry de Bruyn — Alberda van Ekenstein, D-idose có
thể đồng phân hoá thành một 2-ketose (D-sorbose). Vẽ cấu trúc dạng
furanose của D-sorobose.
c) Khi đun nóng D-idose bị mất nước và tồn tại chủ yếu ở dạng 1,6-
anhydro-D-idopyranose. Phản ứng này ưu tiên tạo thành anomer nào
hơn? Vẽ cấu trúc của anomer đó. Giải thích tại sao phản ứng tương tự
không xảy ra với glucose?
Hướng dẫn
a) Dưới đây là cấu dạng ghế của hai anomer D-idose:

Bền hơn

Bền hơn
Các cấu dạng bền hơn đều có nhiều nhóm thế ở vị trí liên kết biên
(equatorial) hơn.
b) Cấu trúc dạng furanose của D-sorobose

36 | Bản quyền thuộc về Tạp chí Olympiad Hóa học KEM


c) Cấu dạng -anomer của D-idose có thể vòng hoá nội phân tử, tạo thành
dẫn xuất anhydro. Để thực hiện phản ứng này thì ở C-6 phải có nhóm thế
liên kết trục (axial). Cấu dạng này của glucose rất kém bền, nên nó không
có phản ứng tương tự.

37 | Bản quyền thuộc về Tạp chí Olympiad Hóa học KEM


Bài 19A
Đun nóng D-altrose với acid loãng tạo thành một đường anhydro (khan)
không khử (C6H10O5). Methyl hóa đường anhydro này sau đó xử lí với acid
tạo thành 2,3,4-tri-O-methyl-D-altrose. Sự tạo thành đường anhydro diễn ra
qua một cấu dạng ghế của β-D-altropyranose trong đó nhóm -CH2OH ở vị
trí trục.
1) Xác định cấu trúc đường anhydro và nó được tạo thành như thế nào?
2) D-glucose cũng tạo thành một đường anhydro nhưng các điều kiện
cần thiết khắc nghiệt hơn so với phản ứng tương ứng của D-altrose.
Giải thích tại sao.
Hướng dẫn
1) Đường anhydro được tạo thành khi nhóm CH2OH trục phản ứng với C1
tạo thành acetal vòng.

Do đường anhydro là acetal (nghĩa là một glycoside nội) nên nó là đường


không khử. Khi methyl hóa, sau đó thủy phân acid sẽ chuyển đường
anhydro thành 2,3,4-tri-O-methyl-D-altrose:

38 | Bản quyền thuộc về Tạp chí Olympiad Hóa học KEM


2) Sự tạo thành một đường anhydro đòi hỏi monosaccharide đáp ứng
cấu djang ghế với nhóm CH2OH ở vị trí trục. Với β-D-altropyranose quá
trình này đòi hỏi hai nhóm OH cũng ở vị trí OH. Tuy nhiên, với β-D-
glucopyranose thì nó đòi hỏi bốn nhóm OH phải có vị trí trục và do đó
phân tử đáp ứng một cấu dạng rất kém bền:

39 | Bản quyền thuộc về Tạp chí Olympiad Hóa học KEM


Bài 20
Xác định cấu trúc các chất chưa biết trong các dãy chuyển hóa sau:
1. Từ axit glucuronic chuyển thành monosaccarit 6-deoxyhexopyranozơ
(C7H14O5) theo sơ đồ:

2. Cũng có thể chuyển axit glucuronic thành một aminosaccarit theo sơ


đồ sau:

Hướng dẫn
1)

40 | Bản quyền thuộc về Tạp chí Olympiad Hóa học KEM


2)

41 | Bản quyền thuộc về Tạp chí Olympiad Hóa học KEM


Bài 21
Cho α-D-glucopyranozơ phản ứng với hỗn hợp Ac2O/HBr thu được chất A.
Chất này tiếp tục phản ứng với trietylamin cho chất B. B dưới ảnh hưởng
của dung dịch nước Ba(OH)2 thu được chất C (C6H10O5). Chất C không có
khả năng phản ứng với thuốc thử Fehling và không tham gia vào quá trình
lên men rượu. 1 mol B phản ứng được với 2 mol HIO4 cho ra G và 1 mol
HCOOH. Oxy hóa G bằng Br2 đầu tiên cho ra sản phẩm D, oxy hóa tiếp chất
này thu được hỗn hợp đẳng số mol của H2C2O4 và axit D – CHOH(COOH)2.
Hãy xác định cấu trúc các chất chưa biết.
Hướng dẫn
Dựa vào công thức phân tử chất C có thể thấy rằng nó là sản phẩm
dehydrat hóa của glucozơ. Rất có khả năng đây là sự tạo thành một liên
kết ete từ hai nhóm ancol. Việc chất C không có khả năng lên men cũng
như cho kết quả âm tính với thuốc thử Fehling cho thấy nhóm hydroxyl ở
C1 đã tham gia phản ứng loại nước này. Các dữ kiện oxy hóa tiếp theo
bằng HIO4 và dung dịch Br2 cho thấy rằng chất C là 1,2,3-triol, tức nhóm
OH tham gia phản ứng ete hóa với OH ở C1 là OH ở C6. Vậy C chính là 1,6-
anhydro-β-D-pyranozơ. Như vậy sơ đồ phản ứng sẽ như sau:

42 | Bản quyền thuộc về Tạp chí Olympiad Hóa học KEM


Bài 22
Hợp chất C (C6H8O6) phản ứng được với 2 mol CH2N2 ở 0 oC và 1 mol CH2N2
ở -78 oC. C có khả năng làm mất màu nước brom và tạo dị vòng 1,3-
dioxolan với axeton. Sản phẩm phản ứng giữa C và 2 mol CH2N2 được
metyl hóa hoàn toàn rồi ozon phân sẽ thu được este D. Chất này phản ứng
với NH3 cho oxamit (CONH2)2 và amit của axit 3,4-dimetyl-L-threonic. Mặt
khác oxy hóa C rồi cắt mạch bằng NaOI thu được axit E (C2H2O2) và axit F
(C4H8O5). Oxy hóa F bằng HNO3 thu được axit L-(+)-tactric. Lập luận xác
định cấu trúc các chất chưa biết với lập thể chính xác. Biết rằng khi cho D
phản ứng với HIO4 thu được hỗn hợp sản phẩm, trong số đó có HCHO.
Hướng dẫn
C phản ứng được với 2 mol CH2N2 ở 0 oC nhưng chỉ có 1 mol CH2N2 ở -78
o
C, chứng tỏ trong C có ít nhất 2 nhóm OH với khả năng phản ứng khác
nhau.
C làm mất màu nước brom và tạo dị vòng 1,3-dioxolan với axeton  Trong
C có ít nhất một liên kết đôi và 2 nhóm OH ở cạnh nhau.
Este D phản ứng với NH3 cho oxamit (CONH2)2, chứng tỏ D phải có đơn vị
RCOO-COOH. D được tạo thành từ quá trình ozon phân, có nghĩa D phải là
một enol.
Việc tạo thành hỗn hợp gồm axit 3,4-dimetyl-L-threonic + oxamit (6C)
chứng tỏ ozon phân không làm gẫy mạch phân tử. Có nghĩa là phân tử
ban đầu phải có cấu trúc vòng, và vị trí endiol sẽ bị cắt mạch bằng ozon.
Bên cạnh đó cắt mạch D bằng HIO4 thu được HCHO chứng tỏ D phải có
đoạn mạch R-CHOH-CH2OH.
Tổng hơp các thông tin trên có thể xác định được C là vitamin C (axit
ascorbic).
Như vậy cấu trúc lập thể của các chất như sau:

43 | Bản quyền thuộc về Tạp chí Olympiad Hóa học KEM


Bài 23
Trong khi các monosaccharide chịu sự đồng phân hóa phức tạp trong
base thì các aldonic acid bị epimer hóa đặc thù ở C2 khi đun nóng với
pyridine. Bạn có thể sử dụng phản ứng này như thế nào để tổng hợp D-
mannose từ D-glucose.
Hướng dẫn

44 | Bản quyền thuộc về Tạp chí Olympiad Hóa học KEM


Bài 23A
Chuỗi phản ứng dưới đây biểu diễn một phương pháp thanh lịch để tổng
hợp 2-deoxy-D-ribose, IV, được D. C. C. Smith công bố năm 1955:

1) Xác định cấu trúc II và III.


2) Đề xuất cơ chế cho chuyển hóa III thành IV.

45 | Bản quyền thuộc về Tạp chí Olympiad Hóa học KEM


Hướng dẫn

46 | Bản quyền thuộc về Tạp chí Olympiad Hóa học KEM


Bài 23B
Khi xử lí D-glucose với sodium hydroxide, thì tạo thành một hỗn hợp phức
tạp của các carbohydrate, bao gồm D-mannose và D-fructose. Theo thời
gian, gần như toàn bộ các chất hiện diện trong hỗn hợp là aldohexose.
Thậm chí có thể phát hiện L-glucose, mặc dù với nồng độ cực kì nhỏ. Trình
bày một cơ chế chấp nhận được, với ít giai đoạn nhất, biểu diễn sự tạo
thành L-glucose từ D-glucose.

47 | Bản quyền thuộc về Tạp chí Olympiad Hóa học KEM


Hướng dẫn

48 | Bản quyền thuộc về Tạp chí Olympiad Hóa học KEM


Bài 24
1) Khi xử lí α-D-galactopyranose với ethanol có mặt xúc tác acid như HCl
thì tạo thành hai sản phẩm. Xác định cấu trúc hai sản phẩm và vẽ cơ
chế giải thích sự tạo thành chúng.
2) Methyl α-D-glucopyranoside là một hợp chất kém bền và không trải
qua sự nghịch đảo đường trong các môi trường trung tính hoặc base.
Tuy nhiên, khi đưa vào môi trường acid thì một cân bằng được thiết
lập giữa methyl α-D-glucopyranoside và methyl β-D-glucopyranoside.
Trình bày cơ chế giải thích cho hiện tượng này.
Hướng dẫn
1) Trong điều kiện acid, nhóm OH anomer có thể bị proton hóa, tạo thành
một nhóm rời đi tốt (H2O). Việc mất nhóm rời đi tạo thành một trung
gian cation được bền hóa cộng hưởng.

Sau đó trung gian này bị tác kích bởi ethanol, tạo thành một oxonium ion,
tiểu phân này sau đó bị deproton hóa tạo thành acetal:

Chú ý rằng như biểu diễn ở trên thì ethanol bị tác kích từ phía trên, tuy
nhiên nó cũng có thể bị tác kích từ phía dưới, tạo thành acetal sau đây:

49 | Bản quyền thuộc về Tạp chí Olympiad Hóa học KEM


2) Trong môi trường acid, nhóm anomer methoxy có thể bị proton hóa, tạo
thành một nhóm rời đi tốt (methanol). Sự mất nhóm rời đi tạo thành một
trung gian cation được bền hóa cộng hưởng (các cấu trúc cộng hưởng
không được biểu diễn dưới đây), có thể bị tác kích bởi methanol. Sự tác
kích này có thể diễn ra từ phía trên hoặc phía dưới tạo thành hỗn hợp hai
anomer. Trong giai đoạn cuối của cơ chế, oxonium ion bị deproton hóa:

50 | Bản quyền thuộc về Tạp chí Olympiad Hóa học KEM


Bài 25
1) Trình bày cơ chế cho chuyển hóa qua lại xúc tác acid của hai glucose
anomer bởi sự nghịch đảo đường:

2) Trình bày cơ chế cho phản ứng sau:

3) Trình bày cơ chế phản ứng thủy phân:

4) Trong quá trình oxid hóa D-allose thành D-allonic acid, phân lập được
một lactone có cấu trúc tổng quát A. Trình bày cơ chế giải thích sự tạo
thành A. Chỉ rõ hóa lập thể của A.

5) Phản ứng đồng phân hóa sau diễn ra với tất cả các aldohexose khi có
mặt base. Trình bày cơ chế biểu diễn cách các hợp chất được tạo
thành.

51 | Bản quyền thuộc về Tạp chí Olympiad Hóa học KEM


Hướng dẫn
1)

2)

52 | Bản quyền thuộc về Tạp chí Olympiad Hóa học KEM


3)

4)

53 | Bản quyền thuộc về Tạp chí Olympiad Hóa học KEM


5)

54 | Bản quyền thuộc về Tạp chí Olympiad Hóa học KEM


Bài 26
Trình bày cơ chế cho phản ứng sau:

Hướng dẫn

55 | Bản quyền thuộc về Tạp chí Olympiad Hóa học KEM


Bài 27
1) Khi xử lí D-altrose hoặc D-talose với sodium borohydride khi có nước
thì đều tạo thành cùng sản phẩm. Giải thích hiện tượng này.
2) Khi xử lí D-allose hoặc L-allose với sodium borohydride khi có nước thì
đều tạo thành cùng sản phẩm. Giải thích hiện tượng này.
3) Trong số tám D-aldohexose, chỉ có hai chất tạo thành alditol không
quang hoạt khi xử lí với sodium borohydride khi có nước. Xác định hai
aldohexose này và giải thích tại sao alditol của chúng không quang
hoạt.
4) Khi xử lí với sodium borohydride, D-glucose bị chuyển thành một
alditol.
a) Xác định cấu trúc alditol.
b) L-aldohexose nào tạo thành cùng alditol khi xử lí với sodium
borohydride?
5) Hợp chất A là một D-aldopentose bị chuyển hóa thành một alditol
quang hoạt khi xử lí với sodium borohydride. Vẽ hai cấu trúc có thể có
của hợp chất A.
Hướng dẫn
1) Trong hai trường hợp, sự khử nhóm carbonyl đều tạo thành cùng sản
phẩm. Có thể thấy được điều này bằng cách xoáy các sản phẩm 180o như
dưới đây:

56 | Bản quyền thuộc về Tạp chí Olympiad Hóa học KEM


2)

57 | Bản quyền thuộc về Tạp chí Olympiad Hóa học KEM


3) Sự khử D-allose hoặc D-galactose sẽ tạo thành meso alditol. Các hợp
chất meso không quang hoạt.

5)

5)

58 | Bản quyền thuộc về Tạp chí Olympiad Hóa học KEM


59 | Bản quyền thuộc về Tạp chí Olympiad Hóa học KEM
Bài 28
1) Vẽ và gọi tên các cặp epimer được tạo thành khi tiến hành quá trình
nối mạch Kiliani-Fischer với các aldopentose sau:

2) Xác định các tác nhân có thể sử dụng để chuyển D-erythrose thành D-
ribose.
3) Xác định hai aldohexose tạo thành khi D-arabinose trải qua tổng hợp
Kiliani-Fischer.
Hướng dẫn
Trong tổng hợp Kiliani-Fischer, mạch carbon được kéo dài, với C1 trở
thành C2 trong sản phẩm. Ở vị trí C2 có thể nhận được hai cấu hình có thể
có, tương ứng với cặp epimer dưới đây:
a)

60 | Bản quyền thuộc về Tạp chí Olympiad Hóa học KEM


b)

c)

2) Chuyển hóa D-erythrose (có bốn carbon) thành D-ribose (có năm
carbon) đòi hỏi một quá trình kéo dài mạch. Quá trình này sẽ tạo thành D-
ribose cùng với C2 epimer của nó, là D-arabinose, như dưới đây:

61 | Bản quyền thuộc về Tạp chí Olympiad Hóa học KEM


3)

62 | Bản quyền thuộc về Tạp chí Olympiad Hóa học KEM


Bài 29
1) Vẽ và gọi tên hai aldohexose có thể chuyển hóa thành D-ribose sử
dụng thoái phân Wohl.
2) Xác định các tác nhân bạn có thể sử dụng để chuyển D-ribose thành
D-erythrose.
3) Khi D-glucose trải qua thoái phân Wohl, sau đó kéo dài mạch Kiliani-
Fischer, thì tạo thành một hỗn hợp epimer. Xác định hai epimer này.
4) Xác định hai aldohexose khi trải qua thoái phân Wohl đều tạo thành D-
ribose. Vẽ công thức chiếu Fischer cho dạng mạch hở của mỗi
aldohexose.
Hướng dẫn
1)

2)

3)

63 | Bản quyền thuộc về Tạp chí Olympiad Hóa học KEM


4)

64 | Bản quyền thuộc về Tạp chí Olympiad Hóa học KEM


Bài 30
1) Vẽ cấu trúc được dự đoán tạo thành khi xử lí dạng β-pyranose của hợp
chất A với lượng dư ethyl iodide khi có mặt silver oxide. Thông tin sau
có thể được dùng để xác định hợp chất A:
- Khối lượng mol của hợp chất A là C6H12O6.
- Hợp chất A là đường khử.
- Khi hợp chất A tham gia vào thoái phân Wohl hai lần liên tiếp thì tạo
thành D-erythrose.
- Hợp chất A là epimer với D-glucose ở C3.
- Cấu hình ở C2 là R.
2) Hợp chất X là một D-aldohexose có thể tồn tại dạng β-pyranose với chỉ
một nhóm thế trục (axial). Hợp chất X trải qua thoái phân Wohl tạo
thành một aldopentose, chất này khi xử lí với sodium borohydrie tạo
thành một alditol quang hoạt. Từ dữ liệu được cho, chỉ có hai cấu trúc
khả dĩ với hợp chất X. Xác định hai cấu trúc này và đề xuất một phép
thử hóa học cho phép bạn phân biệt hai chất này, và từ đó xác định
được cấu trúc của hợp chất X.
3) Một aldohexose bị methyl hóa bởi dimethyl sulfate và sau đó xử lí với
acid êm dịu. Sản phẩm tạo thành, khi tiếp xúc với một chất oxid hóa
mạnh, tạo thành một dicarboxylic acid không quang hoạt chứa năm
nguyên tử carbon. Xác định cấu trúc có thể có của aldohexose.
Hướng dẫn
1) Công thức phân tử C6H12O6 cho thấy A là hexose. A là đường khử, nên
nó phải là aldohexose. Thoái phân Wohl liên tiếp hai lần hợp chất A
tạo thành D-erythrose, cho thấy cấu hình ở C4 và C5 của aldohexose
đều là R (giống như D-erythrose). Hợp chất A là epimer với glucose ở
C3, cho thấy cấu hình C3 là R (D-glucose có cấu hình S ở C3). Cuối
cùng, C2 có cấu hình R. Từ các dữ kiện trên xác định được A là D-allose
(cấu trúc bên dưới). Dạng β-pyranose của D-allose cũng được biểu
diễn dưới đây. Khi xử lí với ethyl iodide dư, có mặt silver oxide thì tất
cả các nhóm OH trong dạng β-pyranose của A bị alkyl hóa, chuyển
thành các nhóm ethoxy:

65 | Bản quyền thuộc về Tạp chí Olympiad Hóa học KEM


2) Hợp chất X là D-aldohexose có dạng β-pyranose với chỉ một nhóm thế
ở vị trí trục. Hãy nhớ lại rằng, D-glucose có tất cả các nhóm thế ở vị trí
biên (equatorial), do đó hợp chất X phải là epimer với D-glucoser hoặc
là ở C2 (D-mannose) hoặc C3 (D-allose) hoặc C4 (D-galactose). Hợp
chất X trải qua thoái phân Wohl tạo thành một aldopentose, chất này
chuyển hóa thành alditol quang hoạt khi xử lí với sodium borohydride.
Do đó, hợp chất X không thể là D-allose, do thoái phân Wohl của D-
allose, sau đó khử tạo thành một alditol không quang hoạt. Chúng ta
kết luận rằng hợp chất X phải là D-mannose hoặc D-galactose. Có thể
xác định hợp chất X bằng cách xử lí X với soium borohydride. Sự khử
D-mannose sẽ tạo thành một alditol quang hoạt, trong khi sự khử D-
galactose tạo thành một alditol không quang hoạt.

66 | Bản quyền thuộc về Tạp chí Olympiad Hóa học KEM


3)

67 | Bản quyền thuộc về Tạp chí Olympiad Hóa học KEM


Bài 31
1) D-aldopentose nào bị oxid hóa thành aldaric acid quang hoạt và trải
qua thoái phân Wohl tạo thành một D-aldotetrose mà khi oxid hóa
cũng tạo thành aldaric acid quang hoạt?
2) D-aldopentose nào khác tạo thành cùng alditol như D-arabinose khi bị
khử bởi NaBH4 trong CH3OH?
3) D-aldopentose A bị oixd hóa bởi HNO3 tạo thành aldaric acid không
quang hoạt B. A trải qua tổng hợp Kiliani-Fischer tạo thành C và D. C
bị oxid hóa thành aldaric acid quang hoạt. Còn D bị oxid hóa thành
aldaric acid không quang hoạt. Xác định các hợp chất A-D.
4) D-aldopentose A bị khử thành một alditol quang hoạt. Sau tổng hợp
Kiliani-Fischer, A chuyển thành hai D-aldohexose B và C. B bị oxid hóa
thành aldaric acid không quang hoạt. C bị oxid hóa thành aldaric acid
quang hoạt. Xác định cấu trúc A-C.
5) D-aldohexose A bị khử thành alditol B quang hoạt, sử dụng NaBH4
trong CH3OH. A bị chuyển hóa bởi thoái phân Wohl tạo thành
aldopentose C, chất này bị khử thành alditol không quang hoạt D. C bị
chuyển hóa bởi thoái phân Wohl tạo thành aldotetrose E, chất này bị
oxid hóa thành aldaric acid quang hoạt F. Khi chuyển đổi qua lại hai
đầu của aldohexose A thì tạo thành một aldohexose G khác. Xác định
cấu trúc A-G.

68 | Bản quyền thuộc về Tạp chí Olympiad Hóa học KEM


Hướng dẫn
1)

Chỉ có A’’ trải qua thoái phân Wohl tạo thành aldotetrose mà bị oxid hóa
thành aldaric acid quang hoạt. Do đó A’’ là cấu trúc của D-aldopentose.
2)

3) Chỉ có hai D-aldopentose (A’ và A’’) tạo thành các aldaric acid không
quang hoạt (B’ và B’’).

69 | Bản quyền thuộc về Tạp chí Olympiad Hóa học KEM


Chỉ có A’ thỏa mãn các dữ kiện. Tổng hợp Kiliani-Fischer của A’ tạo thành
C’ và D’ mà khi bị oxid hóa thành một aldaric acid quang hoạt và một acid
không quang hoạt. Quy trình tương tự với A’’ thì tạo thành hai acid đều
quang hoạt. Do đó, cấu trúc A-D tương ứng với A’-D’.

70 | Bản quyền thuộc về Tạp chí Olympiad Hóa học KEM


4)

5)

71 | Bản quyền thuộc về Tạp chí Olympiad Hóa học KEM


Bài 32
D-aldotetrose A phản ứng nitric acid, tạo thành một hợp chất không quang
hoạt. Cũng aldotetrose này phản ứng với HCN, sau đó xử lý với dung dịch
Ba(OH)2 tạo thành hai aldonic acid B và C, là đồng phân epimer. Các
aldonic acid này tồn tại trong một cân bằng với các -aldolactone tương
ứng, D và E. Cho hỗn hợp này phản ứng với Na-Hg và nước ở pH 3-5, tạo
thành các sản phẩm tương ứng là F và G. Oxid hoá F bằng nitric acid, tạo
thành một aldaric acid H không quang hoạt, trong khi phản ứng tương tự
của E lại tạo thành aldaric acid I quang hoạt. Xác định công thức cấu tạo
các chất A-I.
Hướng dẫn
D-aldotetrose duy nhất khi oxid hoá bằng HNO3 tạo thành sản phẩm không
quang hoạt là:

Khi xử lý với HCN, sau đó là dung dịch Ba(OH)2, A tạo thành B và C

Các -aldolactone D và E tương ứng là

72 | Bản quyền thuộc về Tạp chí Olympiad Hóa học KEM


Cho hỗn hợp cân bằng trên phản ứng với Na-Hg và nước ở pH 3-5, tạo
thành F và G. Oxid hoá F, G bằng nitric acid lần lượt tạo thành các aldaric
acid H (không quang hoạt) và I (quang hoạt).

73 | Bản quyền thuộc về Tạp chí Olympiad Hóa học KEM


Bài 33
Tiến hành phân cắt một monosaccarit X bằng HIO4 thu được hỗn hợp gồm
malondiandehit, fomandehit và axit formic theo tỉ lệ 1:1. X tham gia vào
quá trình chuyển hóa như sau:

a) Xác định cấu trúc các chất chưa biết.


b) Viết bốn công thức cấu tạo có thể có của E. Biết E tồn tại ở dạng
vòng sáu cạnh.
Hướng dẫn
Từ sản phẩm cắt mạch X suy ra cấu trúc mạch hở của A sẽ là HOC
– CH2 – CHOH – CHOH – CH2OH và như vậy cấu trúc mạch vòng của X
và của các chất A, B, C, D sẽ như sau.

Bốn công thức cấu tạo có thể có của E:

74 | Bản quyền thuộc về Tạp chí Olympiad Hóa học KEM


Di-, tri- và polysaccharide
Bài 1
1) Xác định các disaccharide sau có phải đường khử hay không:

2) Isomaltose có cấu trúc tương tự với maltose, ngoại trừ nó có liên kết
1→6 α-glycoside, thay vì 1→4 α-glycoside. Vẽ cấu trúc của
isomaltose.
3) Vẽ cấu trúc sản phẩm tạo thành khi xử lí disaccharide sau với NaBH4
trong methanol:

4) Dự đoán sản phẩm tạo thành khi xử lí cellobiose với các tác nhân sau:
(a) NaBH4, H2O
(b) Br2, H2O (pH=6)
(c) CH3OH, HCl
(d) Ac2O, pyridine

75 | Bản quyền thuộc về Tạp chí Olympiad Hóa học KEM


5) Xác định các tác nhân bạn có thể sử dụng để chuyển β-D-
glucopyranose thành các hợp chất sau:

6) Trình bày cơ chế phản ứng thủy phân xúc tác acid của maltose thành
hai phân tử glucose.
Hướng dẫn
1) (a) Một trong các vị trí anomer (phía dưới bên phải) có một nhóm OH.
Do đó, disaccharide này là đường khử.
(b) Cả hai vị trí anomer đều có nhóm acetal, do đó disaccharide này không
phải đường khử.
(c) Tương tự ý b, không phải đường khử.
2)

76 | Bản quyền thuộc về Tạp chí Olympiad Hóa học KEM


3) Một trong các vòng (phía dưới bên phải) có một nhóm OH.

4) Một trong các vòng (phía dưới bên phải) có một nhóm OH anomer.
a)

b)

c)

d)

77 | Bản quyền thuộc về Tạp chí Olympiad Hóa học KEM


5)
(a) CH3OH, HCl
(b) CH3OH, HCl
(c) HNO3, H2O, heat
(d) CH3I, Ag2O dư, sau đó bởi H3O+

78 | Bản quyền thuộc về Tạp chí Olympiad Hóa học KEM


6)

79 | Bản quyền thuộc về Tạp chí Olympiad Hóa học KEM


Bài 2
Xác định các hợp chất kí hiệu bởi chữ cái trong các phản ứng sau:

Hướng dẫn

80 | Bản quyền thuộc về Tạp chí Olympiad Hóa học KEM


Bài 3
1) Đề xuất (các) cấu trúc vòng có thể có, chỉ rõ hóa lập thể, của (D)-
tagalose trong dung dịch theo công thức chiếu Haworth.

2) Hai sản phẩm có cùng công thức phân tử C6H10O6 được tạo thành khi
xử lí (D)-arabinose với sodium cyanide trong môi trường acid, sau đó
thủy phân acid. Viết các cấu trúc có thể có, chỉ rõ hóa lập thể, của hai
hợp chất này và cho biết chúng được tạo thành như thế nào.

3) Khi một disaccharide khử, turanose, tham gia vào quá trình thủy phân
thù thu được D-glucose và D-fructose với lượng đẳng mol với
saccharide đã dùng. Methyl hóa turanose với methyl iodide khi có mặt
silver oxide, sau đó thủy phân, thu được 2,3,4,6-tetraO-methyl-D-
fructose. Đề xuất cấu trúc có thể có của turanose, không cần quan tâm
đến hóa lập thể ở (các) vị trí anomeric.

81 | Bản quyền thuộc về Tạp chí Olympiad Hóa học KEM


Hướng dẫn
1)

2)

3)

82 | Bản quyền thuộc về Tạp chí Olympiad Hóa học KEM


Bài 4
Ở Hà Lan, lactose (đường sữa) được sản xuất trên diện tương đối rộng từ
váng sữa (một sản phẩm phụ trong công nghiệp sản xuất phomat).
Lactose được dùng trong thực phẩm trẻ em và cả trong thuốc viên. Nó là
hợp chất dissaccharide của các monosaccharide D-galactose và D-
glucose. Cấu trúc của nó (công thức chiếu Haworth) được cho dưới đây.
Đơn vị monosaccharide bên trải là D-galactose.

1) Vẽ công thức chiếu Fischer của D-galactose và D-glucose.


Phản ứng thủy phân xúc tác acid của lactose tạo thành D-galactose và D-
glucose.
2) Sử dụng mũi tên, chỉ ra trong công thức lactose: (a) nguyên tử oxygen
nào sẽ bị proton hóa để thủy phân hiệu quả; (b) liên kết carbon-oxygen
nào sẽ bị phân cắt trong phản ứng thủy phân; (c) nguyên tử carbon
nào sẽ tham gia vào phản ứng với tác nhân Fehling (tác nhân này được
sử dụng để nhận biết đường khử).
Quá trình thủy phân lactose có thể được ghép cặp với quá trình hydrogen
hóa, sử dụng xúc tác kim loại, dẫn đến sự tạo thành các polyalcohol
sorbitol và galactitol, còn được biết đến là glucitol và dulcitol.
3) Vẽ công thức chiếu Fischer của sorbitol và galactitol. Cho biết các hợp
chất này có quang hoạt hay không.
Trong các quá trình công nghiệp, lactose bị đồng phân hóa thành
lactulose, là một dược phẩm đường ruột. Phản ứng hydrogen hóa lactose
tạo thành một C12-polyol là lactitol, đây là một chất làm ngọt ít calor. Cả
hai quá trình này đều được tiến hành ở Hà Lan.
4) a) Vẽ công thức Haworth của lactulose. (Gợi ý: Phần glucose của
lactose bị đồng phân hóa thành đường keto fructose); b) Vẽ công thức
Haworth của lactitol.
83 | Bản quyền thuộc về Tạp chí Olympiad Hóa học KEM
Hướng dẫn
1)
CHO CHO
H OH H OH
HO H HO H
H OH HO H
H OH H OH
CH 2OH CH 2OH

D-glucose D-galactose
2)

a
c

CH 2OH OH
OH O
O
OH
OH OH
O
b
OH OH

3)
CH 2OH CH 2OH
H OH H OH
HO H HO H
H OH HO H
H OH H OH
CH 2OH CH 2OH

Sorbitol (quang hoạt) Galactitol (không quang hoạt)


4)
CH 2OH OH CH 2 OH
OH O OH O O OH
O
OH
OH CH 2OH OH OH CH 2OH
OH O
OH OH OH CH 2 OH

Lactitol Lactolose (f uranose)


CH 2OH OH
OH O CH2 OH
O OH
OH OH O

OH
Lactolose (pyranose)

84 | Bản quyền thuộc về Tạp chí Olympiad Hóa học KEM


85 | Bản quyền thuộc về Tạp chí Olympiad Hóa học KEM
Bài 5
Disaccharide A (C12H22O12) có phản ứng âm tính với dung dịch thuốc thử
Benedict và không bị chuyển quay. A bị thuỷ phân bởi enzyme α-
glucosidases nhưng không bị thuỷ phân bởi β-glucosidases. Methyl hoá
rồi thuỷ phân, tạo ra hai đương lượng 2,3,4,6-tetra-O-methyl-D-glucose.
a) Xác định công thức cấu tạo của A.
b) 1 mol A phản ứng được với bao nhiêu mol periodic acid?
c) Có bao nhiêu mol formaldehyde và bao nhiêu mol formic acid tạo
thành trong phản ứng của 1 mol A với periodic acid?
Hướng dẫn
a) Từ các dữ kiện, ta thấy:
- Hợp chất A là một đường không khử với cầu  ở nguyên tử carbon
anomer.
- Thuỷ phân A chỉ tạo thành 2,3,4,6-tetra-O-methyl-D-glucose.
do vậy cấu trúc duy nhất của A là gồm hai đơn vị glucose nối với nhau ở vị
trí carbon anomer tương ứng.

b, c) Cần 10 mol HI để oxid hoá-cắt mạch hoàn toàn A, tạo thành 10 mol
formic acid và 2 mol formaldehyde.

86 | Bản quyền thuộc về Tạp chí Olympiad Hóa học KEM


Bài 6
1) Suy ra cấu trúc của disaccharide isomaltose từ dữ kiện sau:
- Thủy phân chỉ thu được D-glucose.
- Isomaltose bị phân cắt bởi các enzyme α-glycosidase.
- Isomaltose là đường khử.
- Methyl hóa với CH3I dư, Ag2I sau đó thủy phân với H3O+ tạo
thành hai sản phẩm:

(mỗi sản phẩm có cả hai anomer)


2) Suy ra cấu trúc của disaccharide trehalose từ dữ kiện sau. Trehalose
là “đường huyết” của thế giới côn trùng. Nó được tìm thấy trong bào
tử vi khuẩn, nấm và nhiều loại côn trùng có môi trường tự nhiên vơi sự
biến đổi nhiều về nhiệt độ.
- Thủy phân chỉ tạo thành D-glucose.
- Trehalose bị thủy phân bởi các enzyme α-glycosidase.
- Trehalose là đường không khử.
- Methyl hóa với lượng dư CH3I, Ag2O, sau đó thủy phân với H3O+
chỉ tạo thành một sản phẩm:

3) Xác định cấu trúc trisaccharide X từ các thông tin dưới đây:
- Methyl hóa với CH3I dư, Ag2O sau đó thủy phân với H3O+ tạo
thành ba sản phẩm. Mỗi chất đều tương ứng với hai epimer.

87 | Bản quyền thuộc về Tạp chí Olympiad Hóa học KEM


- X bị phân cắt bởi enzyme β-glycosidase tạo thành một
disaccharide và D-galactose.
- X bị phân cắt bởi enzyme α-glycosidase tạo thành một
disaccharide và D-fructose.
4) Thủy phân disaccharide primeverose tạo thành D-xylose và D-glucose.
Methyl hóa primeverose bởi dimethyl sulfate sau đó thủy phân acid
êm dịu tạo thành 2,3,4-tri-O-methyl-D-xylose và 2,3,4-tri-O-methyl-D-
glucose. Xác định cấu trúc primeverose dựa vào những dữ kiện trên.
Hướng dẫn
1)

2)

88 | Bản quyền thuộc về Tạp chí Olympiad Hóa học KEM


3)

4)

89 | Bản quyền thuộc về Tạp chí Olympiad Hóa học KEM


Bài 6A
1) Suy ra cấu trúc của dissaccharide melibiose từ các dữ kiện sau:
- Melibiose là một đường khử bị nghịch đảo đường và tạo thành một
phenylosazone.
- Thủy phân melibiose với acid hoặc với α-galactosidase tạo thành
D-galactose và D-glucose.
- Oxid hóa melibiose bởi nước bromine tạo thành melibionic acid.
Methyl hóa melibionic acid sau đó thủy phân tạo thành 2,3,4,6-
tetra-O-methyl-D-galactose và 2,3,4,5-tetra-O-methyl-D-gluconic
acid.
- Methyl hóa melibiose sau đó thủy phân tạo thành2,3,4,6-tetra-O-
methyl-D-galactose và 2,3,4-tri-O-methyl-D-glucose.
2) Stachyose tồn tại trong rễ nhiều loại thực vật. Hãy suy luận cấu trúc
của stachyose từ các dữ kiện sau:
- Thủy phân acid 1 mol stachyose tạo thành 2 D-galactose, 1 mol D-
glucose, và 1 mol D-fructose.
- Stachyose là đường không khử.
- Xử lí stachyose với α-galactosidase tạo thành hỗn hợp gồm D-
galactose, sucrose, và một trisaccharide không khử gọi là
raffinose.
- Thủy phân acid raffinose tạo thành D-glucose, D-fructose, và D-
galactose. Xử lí raffinose với α-galactosidase tạo thành D-
galactose và sucrose. Xử lí raffinose với invertase (một enzyme
thủy phân sucrose) tạo thành fructose và melibiose (xem ý 1).
- Methyl hóa stachyose, sau đó thủy phân tạo thành 2,3,4,6-tetra-O-
methyl-D-galactose, 2,3,4-tri-O-methyl-D-galactose, 2,3,4-tri-O-
methyl-D-glucose, và 1,3,4,6-tetra-O-methyl-D-fructose.
Hướng dẫn
1)
- Melibiose là một đường khử và bị nghịch đảo đường và tạo thành
một phenylosazone - những dữ kiện này cho thấy một
monosaccharide hiện diện ở dạng hemiacetal vòng.
- Thủy phân acid tạo thành D-galactose và D-glucose cho thấy
melibiose là một disaccharide tạo thành từ một đơn vị D-galactose

90 | Bản quyền thuộc về Tạp chí Olympiad Hóa học KEM


và một đơn vị D-glucose. Melibiose bị thủy phân bởi α-
galactosidase cho thấy nó là một α-D-galactosyl-D-glucose.
- Oxid hóa melibiose thành melibionic acid và sau đó thủy phân tạo
thành D-galactose và D-gluconic acid cho thấy rằng đơn vị glucose
hiện diện ở dạng hemiacetal vòng và đơn vị galactose ở dạng
glycoside (Nếu điều ngược lại là đúng thì thí nghiệm này sẽ tạo
thành D-glucose và D-galactonic acid.)
Sự methyl hóa và thủy phân melibionic acid tạo thành 2,3,4,6-tetra-
O-methyl-D-galactose và 2,3,4,5-tetra-O-methyl-D-gluconic acid.
Sự tạo thành sản phẩm đầu tiên - một dẫn xuất galactose với nhóm
OH tự do ở C5 - chứng minh rằng vòng galactose có sáu cạnh; sự
tạo thành sản phảm thứ hai - dẫn xuất gluconic acid với nhóm OH
tự do ở C6 - chứng minh rằng oxygen ở C6 của đơn vị glucose tham
gia vào một cầu glycoside với đơn vị galactose.
- Sự methyl hóa và thủy phân melibiose tạo thành một dẫn xuất
glucose (2,3,4-tri-O-methyl-D-glucose) với các nhóm OH tự do ở C5
và C6 cho thấy vòng glucose cũng có sáu cạnh. Do đó, melibiose
là 6-O-(α-D-galactopyranosyl-D-glucopyranose.)

6-O-(α-D-Galactopyranosyl)-D-glucopyranose
2) Stachyose có cấu trúc như sau:

91 | Bản quyền thuộc về Tạp chí Olympiad Hóa học KEM


Raffinose có cấu trúc như sau:

Các quá trình thủy phân enzyme (như đã được chỉ ra ở trên) cho biết cấu
trúc cơ bản của stachyose và raffinose. Câu hỏi duy nhất còn lại là kích
thước vòng của đơn vị galactose đầu tiên trong stachyose. Sự methyl hóa
stachyose sau đó thủy phân tạo thành 2,3,4,6-tetra-O-methyl-D-galactose
xác thực đây là vòng pyranoside.

92 | Bản quyền thuộc về Tạp chí Olympiad Hóa học KEM


Bài 7
Disaccharide A bị thuỷ phân bởi acid loãng, tạo thành hỗn hợp D-glucose
và D-galactose. Hợp chất A là đường khử, bị oxid hoá bởi nước bromine
tạo thành acid B, chất này bị methylate hoá bởi sodium hydroxide và
dimethylsulfate, tạo thành một hợp chất octa-O-methylate. Thuỷ phân
chất này, thu được tetra-O-methylgluconic acid (C) và tetra-O-
methylgalactose (D). Oxid hoá C bằng nitric acid, tạo thành tetra-O-
methylglucaric acid. C cũng được tạo thành từ phản ứng thuỷ phân (xúc
tác acid) methyl 2,3,4,6-tetra-O-methylgalactopyranoside. Hợp chất A bị
thuỷ phân bởi enzym α-galactosidase (được chiết tách từ hạnh nhân). Xác
định cấu trúc các chất A, B, C, D.
Hướng dẫn

A là 6-O-α-D-galactopyranosyl-D-glucopyranose
(Cầu nối α bị phá huỷ bởi α-galactosidase, tạo thành glucose và galatose.
Trong cấu trúc có nhóm hemiacetal nên là đường khử)

93 | Bản quyền thuộc về Tạp chí Olympiad Hóa học KEM


94 | Bản quyền thuộc về Tạp chí Olympiad Hóa học KEM
Bài 8
Khi thủy phân tinh bột bằng axit thường chỉ tạo thành các sản phẩm có vị
ngọt (glucozơ, mantozơ...). Khi thủy phân tinh bột trong công nghiệp (30
– 40% huyền phù tinh bột trong dung dịch HCl loãng, nhiệt độ 130 – 140oC)
một vài hợp chất đắng được tạo thành. Người ta đã nghĩ rằng chất “đường
đắng“ X này được tạo thành do phản ứng thủy phân thuận nghịch. X đã
được tách ra và tinh chế. Để xác định cấu tạo của nó người ta đã tiến hành
các thí nghiệm sau:
Thí nghiệm 1: Thủy phân trong axit chỉ thu được glucozơ
Thí nghiệm 2: 24,795 mg X được hòa tan trong 25 cm3 dung dịch kali
hexaxianoferat (III) K3[Fe(CN)6] 0,01 M. Sau đó thêm 5 cm3 dung dịch KOH
0,1 M và đưa hỗn hợp lên 100oC trong 15 phút. Lượng dưu K3[Fe(CN)6]
được xác định bằng phương pháp chuẩn độ iot. Thêm một lượng dư KI và
hỗn hợp được axit hóa bằng CH3COOH. Chuẩn độ bằng Na2S2O3 0,01 M
thấy tốn hết 10,5 cm3
Thí nghiệm 3: X được metyl hóa bằng một lượng dư metyl iodua có mặt
bạc (I) oxit. Sau đó sản phẩm metyl hóa được đun nóng với HNO3 30%.
Hai axit được tách ra từ hỗn hợp là axit 2,3,4-trimetoxy-5-hydroxiadipic
HOOC – CHOH – [CH(OCH3)]3 – COOH và axit 2,3,4,6-tetrametoxy-5-
hydroxicaproic CH3OCH2 – CHOH - [CH(OCH3)]3 – COOH.
1. Xác định cấu tạo của X
2. Viết các phương trình phản ứng trong thí nghiệm nhận biết X
Hướng dẫn
1. Cấu tạo của X: 6-O--D-glucopyranosyl-D-glucozơ (gentiobiozơ)

2. Các phản ứng xảy ra:


Thí nghiệm 1:

95 | Bản quyền thuộc về Tạp chí Olympiad Hóa học KEM


Thí nghiệm 2:

2[Fe(CN)6]3- + 2I- → 2[Fe(CN)6]4- + I2


I2 + 2Na2S2O3 → Na2S4O6 + 2NaI
n(Na2S2O3) ban đầu = 0,0105.0,0100 = 0,0105 mmol
n(K3[Fe(CN)6] ban đầu = 0,250 mmol
Vậy số mol K3[Fe(CN)6] tham gia vào phản ứng oxy hóa = 0,145 mmol 
MX = 342 g/mol
Như vậy X rõ ràng là một disaccarit do M(Glu2) = 180.2 – 18 = 342 g/mol
Thí nghiệm 3:

Sự hiện diện của axit 2,3,4-trimetoxy-5-hydroxiadipic HOOC – CHOH –


[CH(OCH3)]3 – COOH trong hỗn hợp sản phẩm chứng tỏ trong disaccarit
tồn tại liên kết 1 → 6 glycozit.

96 | Bản quyền thuộc về Tạp chí Olympiad Hóa học KEM


Bài 9
Trong truyền máu thường quy, việc xác định nhóm máu là rất cần thiết.
Việc trộn lẫn các loại máu không tương thích có thể gây ra sự ngưng kết
các tế bào hồng cầu và dẫn đến tử vong. Có bốn nhóm máu đã biết: A, B,
AB và O. Phân loại nhóm máu phù thuộc vào thành phần của các
oligosaccharide liên kết với bề mặt protein của các tế bào hồng cầu.
Phần trung tâm của saccharide luôn là galactose được liên kêt với protein:

Galactose cũng liên kết với:


- Đường C bởi liên kết α-1,2’ trong máu nhóm O.
- Đường C bởi liên kết α-1,2’ và đường A bởi liên kết α-1,3’ trong máu
nhóm A.
- Đường C bởi liên kết α-1,2’ và đường B bởi liên kết α-1,3’ trong máu
nhóm B.
Xác định đường A: Đường A phản ứng với thuốc thử Tollens (thí nghiệm
tráng bạc) tạo thành hợp chất dưới đây:

Xác định đường B: Đường B phản ứng với hydroxylamine tạo thành hợp
chất X, chất này được đun nóng tiếp với acetyl anhydride khi có mặt
sodium acetate tạo thành hợp chất Y. Phản ứng thủy phân base hợp chất
Y tạo thành đường Z có ít hơn một nguyên tử carbon so với đường gốc B.
Đường Z được xử lí với các tác nhân như đường B tạo thành D-threose.

97 | Bản quyền thuộc về Tạp chí Olympiad Hóa học KEM


Các chuyển hóa như trên được mô tả theo sơ đồ dưới đây:

Hàm lượng các nguyên tố trong hợp chất Y là 49.6% C, 41.3% O, 3.6% N và
phần còn lại là hydrogen. Biết rằng khi oxid hóa đường B với HNO3 tạo
thành sản phẩm không quang hoạt
Xác định đường C: Đường C thuộc dãy L. Cấu hình tuyệt đối của nguyên
tử carbon thứ ba (C-3) là R. Khi đường C bị khử trong các điều kiện để
chuyển hóa nhóm CHO thành methyl thì tạo thành hợp chất meso. Phản
ứng giữa 1 mol C và HIO4 tạo thành 1 mol CH3CHO và 4 mol HCOOH.
Câu hỏi:
A. Vẽ công thức chiếu Fischer của:
1) Đường A.
2) Đường B và các dẫn xuất X, Y, Z.
3) Đường C.
Giải thích cấu trúc của các hợp chất B, Y, C.
B. Các monosaccharide tạo thành vòng pyranose 6 cạnh. Vẽ công thức
chiếu Haworth của:
1) disaccharide từ máu nhóm O.
2) trisaccharide từ máu nhóm A.
3) trisaccharide từ máu nhóm B.

98 | Bản quyền thuộc về Tạp chí Olympiad Hóa học KEM


Hướng dẫn
A.1) Trong phản ứng Tollens, nhóm aldehyde bị oxid hóa thành nhóm
carboxylic. Cấu trúc của sản phẩm trong thí nghiệm tráng gương đã được
biết, do đó cấu trúc của A phải là:

2) Sản phẩm của phản ứng thoái phân hai lần của đường B là D-threose,
do đó cấu trúc mảnh cuối là:

Đường B bị oxid hóa thành aldaric acid không quang hoạt khi có mặt nitric
acid. Một aldaric acid không quang hoạt khi nó có một mặt phẳng đối
xứng. Chỉ có duy nhất một cấu trúc aldaric acid với mặt phẳng đối xứng
với mảnh cuối phân tử đã cho:

Do cấu trúc của aldaric acid đã biết nên có thể xác định cấu trúc của
đường B:

99 | Bản quyền thuộc về Tạp chí Olympiad Hóa học KEM


Sản phẩm phản ứng của đường B với hydroxylamine là một oxime X:

Công thức phân tử của hợp chất Y là:

Số nguyên tử carbon và oxygen thể hiện rằng tất cả các nhóm hydroxyl đã
bị acetyl hóa trong phản ứng với acetyl anhydride. Cũng quan sát thấy
phản ứng tách nước nhóm oxime thành nitrile. Biết tất cả các dữ kiện này,
chúng ta cố thể xác định cấu trúc hợp chất Y:

Đường Z có ít hơn 1 carbon so với đường gốc B, do đó cấu trúc của nó


phải là:

100 | Bản quyền thuộc về Tạp chí Olympiad Hóa học KEM
3) Trong phản ứng khử, nhóm CHO bị chuyển thành CH3 và tạo thành hợp
chất meso. Điều này có nghĩa rằng nguyên tử carbon cuối cùng trong
đường gốc C được gắn vào nhóm CH3. Điều này được chứng minh bởi
phản ứng với HIO4 bởi CH3CHO đã được xác định là một trong các sản
phẩm. Cấu hình tuyệt đối của nguyên tử carbon thứ ba (C-3) là R và đường
thuộc dãy , do đó hợp chất phải chứ các mảnh được biểu diễn dưới đây:

Có duy nhất một cấu trúc của sản phẩm phản ứng khử với mặt phẳng đối
xứng:

Vậy cấu trúc của đường C như sau:

101 | Bản quyền thuộc về Tạp chí Olympiad Hóa học KEM
B. Sơ đồ chung về sự gắn kết saccharide:

Các saccharide trong máu nhóm O:

Các saccharide trong máu nhóm A và B:

102 | Bản quyền thuộc về Tạp chí Olympiad Hóa học KEM
Máu nhóm A: R = NHCOCH3; máu nhóm B: R= OH

103 | Bản quyền thuộc về Tạp chí Olympiad Hóa học KEM
Bài 10
Hợp chất X là một trisaccharide tồn tại chủ yếu trong bột hạt bông. X
không phản ứng với các dung dịch Benedict hoặc Fehling, nó cũng không
bị chuyển quay. Thủy phân X xúc tác acid tạo thành 3 D-hexose khác nhau:
A, B, C. Các hợp chất A và B, cũng như hợp chất 1 (dưới đây), đều tạo thành
osazone giống nhau khi phản ứng với lượng dư phenylhydrazine. Hợp chất
C phản ứng với nitric acid tạo thành hợp chất D không quang hoạt. Phương
pháp Kiliani-Fischer được sử dụng để thiết lập tương quan về cấu hình
giữa D-glyceraldehyde và C. Aldotetrose trung gian tạo thành C khi bị oxid
hóa bởi nitric acid không tạo thành hợp chất meso. Khi xử lí A với nitric
acid thì dicarboxylic acid (aldaric acid) tạo thành có tính quang hoạt. Cả
A và B đều phản ứng với 5 mol HIO4, 1 mol A tạo thành 5 mol methanoic
(formic) acid và 1 mol methanal (formaldehyde), trong khi đó 1 mol B tạo
thành 4 mol methanoic (formic) acid và 1 mol methanal (formaldehyde)
và 1 mol carbon dioxide. Cả A và B đều có liên quan với cùng một
aldotetrose là đồng phân dia của một trong các chất mà C có liên quan.
Khi methyl hóa X, sau đó thủy phân sẽ tạo thành một 2,3,4-tri-O-methyl-D-
hexose (E) (có nguồn gốc từ A), một 1,3,4,6-tetra-O-methyl-D-hexose (F)
(có nguồn gốc từ B), và một 2,3,4,6-tetra-O-methyl-D-hexose (G) (có nguồn
gốc từ C).

1) Vẽ công thức chiếu Fischer của A, B, C, D.


2) Hãy hoàn thành công thức chiếu Haworth để chỉ rõ kích thước vòng và
hóa lập thể [cấu hình] tuyệt đối của E, F, G.
3) Xác định trình tự các monosaccharide trong trisaccharide X.

104 | Bản quyền thuộc về Tạp chí Olympiad Hóa học KEM
Hướng dẫn
1) Hợp chất X là một trisaccharide không phản ứng với dung dịch
Benedict, và cũng không chuyển quay. Thông tin này cho thấy X là một
đường không-khử, do đó chỉ có các cầu acetal hoặc ketal tồn tại ở tất
cả các nguyên tử anomeric carbon. Trong số 3 monosaccharide thì A
và B tạo thành osazone giống nhau, do đó có hóa lập thể ở C3, C4, C5
(và C6) giống nhau. A và B cũng khác với hợp chất 1 (chính là D-
mannose) - chất tạo thành cùng osazone, do đó 1 trong số chúng phải
là đồng phân epimer C2 của D-mannose (nghĩa là D-glucose) và chất
còn lại là đường keto tương ứng ở C2 (nghĩa là D-fructose). (Kết luận
này được xác thực sau ở các phản ứng oxid hóa cắt mạch.) Hợp chất
C, sau phản ứng với nitric acid, tạo thành một aldaric acid D không
quang hoạt. Hai aldaric acid có thể là D là:

Aldotetrose là tiền chất của C (và cũng là của D) không tạo thành hợp chất
meso sau phản ứng với nitric acid, do đó phải là D-threose:

Theo đó thì aldaric acid D tạo thành từ C ở trên là AA1 và do đó C phải là


D-galactose. Hợp chất A phản ứng với 5 mol HI tạo thành 5 mol methanoic

105 | Bản quyền thuộc về Tạp chí Olympiad Hóa học KEM
(formic) acid và 1 mol methanal (formaldehyde), cho thấy rằng nó là
aldohexose, trong khi đó B phản ứng với 5 mol HI tạo thành 4 mol
methanoic (formic) acid, 1 mol methanal (formaldehyde) và 1 mol CO2
cho thấy rằng nó là ketohexose.
Hợp chất A và B đều liên quan đến tetrose giống nhau, chất này khác với
chất liên quan đến C (nghĩa là liên quan đến D-erythrose). Tetrose liên
quan đến A và B phải có cấu trúc như sau và theo đó thì A là D-glucose và
B là D-fructose.

2) Methyl hóa X, sau đó thủy phân, tạo thành các chất E, F, G sau đây:

106 | Bản quyền thuộc về Tạp chí Olympiad Hóa học KEM
3) Trong quá trình methyl hóa, chỉ có các nhóm hydroxyl không tham gia
tạo thành acetal/ketal (nội hoặc liên phân tử) thì mới bị ester hóa. Từ
dữ kiện methyl hóa, chỉ có E có 2 nhóm hydroxyl liên kết với các
carbohydrate khác. Do đó, A phải là carbohydrate trung tâm. Các kết
quả này chỉ ra rằng, chuỗi sắp xếp các monosaccharide trong X là C-
A-B (hoặc B-A-C).
Nếu: A5 biểu diễn furanose (vòng 5 cạnh) tạo thành carbohydrate A. A6
biểu diễn pyranose (vòng 6 cạnh) tạo thành carbohydrate A. B5 biểu diễn
furanose (vòng 5 cạnh) tạo thành carbohydrate B, … thì trisaccharide X có
thể được biểu diễn theo kiểu như là: C6-A6-B5. Có 4 biến thể như vậy và
dưới đây là một trong số đó. Chú ý: Bản chất của các liên kết anomeric
không được xác định trong bài này. Sự sắp xếp liên kết của A với B và C
cũng có thể bi nghịch đảo (nghĩa là liên kết 1,1’ giữa C và A và liên kết 1,6
giữa A và B.)

107 | Bản quyền thuộc về Tạp chí Olympiad Hóa học KEM
Bài 11
Thủy phân -chaconin (C45H73NO14) (thành phần chính trong cây họ cà và
khoai tây) bằng enzym β-glicoziđaza thu được A (C18H32O14) và alkaloid
solanidin C27H43NO). Xử lý A bằng AcOH/DMF thu được A1 chỉ chứa vòng
sáu cạnh, A1 không phản ứng với (Ag2O/NH3). Metyl hóa hoàn toàn A1
bằng MeI/Ag2O thu được A2. Thủy phân A2 bằng xúc tác axit thu được B
(C8H16O6), C1 (C9H18O5) và C2 (C8H16O5). Oxy hóa B bằng HIO4 thu được D.
Tiến hành cắt mạch D bằng dung dịch NaOH thu được HCOONa,
metoxietanal, (R)-2-hydroxi-3-metoxipropanal. Oxy hóa C1 bằng HNO3 thu
được chủ yếu axit (2R, 4R)-2,3,4-trimetoxipentadioic. Oxy hóa C2 bằng
HIO4, sau đó bằng HNO3 cho chủ yếu là axit (2R, 3R)-2,3-
dimetoxibutandioic. Biết rằng B là một dẫn xuất của glucozơ, C1 và C2 là
hai dẫn xuất của cùng một đường có cấu hình L. Hãy vẽ công thức
Haworth của tất cả các chất chưa biết có trong bài. Biết alkaloid solanidin
có cấu trúc như sau:

Hướng dẫn
Vì G là dẫn xuất của glucozơ, C1 và C2 là hai dẫn xuất của cùng một đường
có cấu hình L và S có công thức phân tử C18H32O14 có thể dự đoán S được
tạo thành từ hai liên kết glycozit (tách hai phân tử nước) từ một phân tử
glucozơ (C6H12O6) và hai phân tử đường có cấu hình L có công thức
C6H12O5.
B là dẫn xuất của D-glucozơ, oxy hóa G bằng HIO4 thu được D, cắt mạch D
bằng NaOH thu được HCOONa, metoxyetanal, (R)-2-hydroxi-3-
metoxypropanal theo phản ứng retro aldol và thủy phân este. Như vậy có
thể xác định được cấu trúc B.

108 | Bản quyền thuộc về Tạp chí Olympiad Hóa học KEM
Oxy hóa C1 bằng HNO3 thu được chủ yếu axit (2R, 4R)-2,3,4-
trimetoxypentandioic, axit meso-2,3-dimetoxybutandioic và axit axetic. Do
C1 và C2 là hai dẫn xuất của cùng một đường có cấu hình L nên cấu hình
của nhóm C3 – OH của C1 tương đồng với cấu hình nhóm C3 – OH trong
C2. Mặt khác oxy hóa C2 bằng HIO4, sau đó bằng HNO3 thu được axit (2R,
3R)-2,3-dimetoxybutandioic nên có thể xác định được lập thể các nhóm
OH và bộ khung carbon. Từ đó có thể xác định hai đường C1 và C2 đều có
chung bộ khung rhamanozơ. Như vậy cấu trúc của C1 và C2 như sau:

Từ cấu trúc của B, C1, C2 cùng với A (C18H32O14) có thể thấy rằng trong ba
liên kết glycozit chỉ có một liên kết β-glycozit giữa A và solanidin và hai
liên kết -glycozit giữa các đường. Từ vị trí các nhóm OH tự do của B, C1
và C2, cũng như A1 không cho phản ứng với Ag2O/NH3 nên nhóm OH
anome của các hợp phần A và B sẽ tạo liên kết glycozit vòng 6 khi xử lý
với AcOH/DMF. Từ đó có thể xác định được cấu trúc các hợp phần còn lại
như sau:

109 | Bản quyền thuộc về Tạp chí Olympiad Hóa học KEM
110 | Bản quyền thuộc về Tạp chí Olympiad Hóa học KEM
Bài 12
Một trisaccarit thiên nhiên X được tạo thành từ ba monosaccarit A, B, C.
Khối lượng phân tử của X là 504 g/mol. Metyl hóa X bằng metyl iodua rồi
thủy phân trisaccarit đã được metyl hóa này trong điều kiện khắc nghiệt
(1M H2SO4, 100oC) và trong điều kiện êm dịu hơn (0,01M H2SO4) thì thu
được các sản phẩm sau
Điều kiện khắc nghiệt: Một hỗn hợp gồm các dẫn xuất 2,3,4,6-tetrametyl;
2,3,4-trimetyl và 1,3,4,6-tetrametyl monosaccarit
Điều kiện êm dịu: Dẫn xuất 1,3,4,6-tetrametyl của C và disaccarit Y đã bị
metyl hóa
Disaccarit Y được xử lý tiếp với metyl iodua và thủy phân trong điều kiện
khắc nghiệt cho sản phẩm 2,3,4,6-tetrametyl và 1,2,3,4-tetrametyl
monosaccarit.
Thủy phân hoàn toàn chất X rồi cô lập từng sản phẩm thu được đem phân
tích cho kết quả monosaccarit C đồng nhất với sản phẩm thủy phân
saccarozơ. Bên cạnh đó B và C có thể chuyển hóa qua lại trong môi trường
kiềm.
Để nhận diện B thì người ta tiến hành quá trình tổng hợp cyanohydrin. Sản
phẩm thu được từ quá trình này bị oxy hóa bởi axit nitric đặc để cho một
axit dicarboxylic D quang hoạt và một axit dicarboxylic E không quang
hoạt.
Sự oxy hóa A bằng axit nitric đặc cho axit dicarboxylic không quang hoạt
F. A có thể được tổng hợp từ D-threose sau hai quá trình tổng hợp
cyanohydrin liên tiếp.
Trong trisaccarit X thì B liên kết với C, còn A được liên kết với B bằng một
liên kết -glycosid. Thứ tự liên kết đi từ trái sang phải được xác định là A
– B – C.
Xác định A, B, C, X và Y bằng cách vẽ công thức Haworth của chúng

111 | Bản quyền thuộc về Tạp chí Olympiad Hóa học KEM
Hướng dẫn
C đồng nhất với sản phẩm thủy phân D-saccarozơ nên C phải là một
hexozơ. Do C và B có thể chuyển hóa qua lại cho nhau được nên B cũng
phải là một hexozơ. Do khối lượng phân tử của một mảnh CH2O là 30 thì
số nguyên tử carbon của X sẽ là (504 + 2.18) / 30 = 18 nguyên tử C.
Do B và C đã là hexozơ nên A cũng phải có 6 nguyên tử carbon.
Xử lý X bằng MeI rồi sau đó thủy phân trong điều kiện êm dịu cho sản
phẩm 1,3,4,6-tetrametyl của chất C. Như vậy nhóm hydroxi ở vị trí C2 của
C đã liên kết với monosaccarit B, tức nhiều khả năng C là một xetozơ. Do
C đồng nhất với sản phẩm thủy phân saccarozơ nên C chỉ có thể là D-
fructozơ.
Do B và C có thể chuyển hóa qua lại lẫn nhau trong môi trường kiềm nên
B có thể là D-glucozơ hay D-mannozơ. Tuy nhiên nếu tiến hành quá trình
tổng hợp cyanohydrin thì cả hai sản phẩm từ D-mannozơ đều quang hoạt
(trái giả thiết) nên B chỉ có thể là D-glucozơ. Cấu trúc hai chất D và E sẽ là:

Tiến hành hai quá trình cyanohydrin liên tiếp D-threozơ sẽ thu được bốn
hexozơ, trong đó chỉ có duy nhất D-galatozơ khi oxy hóa bằng axit nitric
cho sản phẩm không quang hoạt.
Dựa trên cấu trúc các sản phẩm metyl hóa thì C sẽ liên kết với B ở nguyên
tử carbon thứ hai, còn A liên kết với B qua nguyên tử carbon thứ nhất. B
nối với C qua nguyên tử carbon thứ nhất, và B nối với A qua nguyên tử
carbon thứ sáu. Tức cấu trúc hai đường X và Y sẽ là:

112 | Bản quyền thuộc về Tạp chí Olympiad Hóa học KEM
113 | Bản quyền thuộc về Tạp chí Olympiad Hóa học KEM
Bài 13
Inulin, chiết xuất từ rễ cây diếp xoăn ở Bỉ và Hà Lan,
được sử dụng làm phụ gia thực phẩm do nó có tác
động hữu ích đến hệ vi sinh đường ruột. Nó cũng
được sử dụng làm nguồn fructose - chất có độ ngọt
gấp 1.9 lần sucrose, và để sản xuất mannitol - chất
được sử dụng trong kẹo cao su. Inulin là một
polymer mạch thẳng chứa các đơn vị fructose với
một đơn vị glucose ở đầu mạch. Công thức chiếu
Haworth của nó được cho ở hình bên. Trong bài này,
inulin được xét có 10 đơn vị fructose (n = 9).
1) Inulin có thể bị thủy phân trong môi trường H+.
Hãy cho biết liên kết C-O nào trong các phương
án sau đây dễ bị phân cắt nhất.

Quá trình thủy phân với nước đánh dấu đồng vị có thể cung cấp thông tin
về cơ chế thủy phân nhờ vào NMR - kĩ thuật hiện đại có thể “thấy” được
deuterium (2H) và đồng vị oxygen 17O.
2) Xác định phân tử nước đánh dấu nào là lựa chọn tốt nhất cho mục
đích này.
□ 2H2O
□ H217O
□ 2H217O

114 | Bản quyền thuộc về Tạp chí Olympiad Hóa học KEM
□ Không chất nào trong số chúng.
Khi hydrogen hóa xúc tác, glucose tạo thành sorbitol (S), trong khi đó
fructose (F) tạo thành mannitol (M) và sorbitol (S).
3) Vẽ công thức chiếu Fischer của fructose (F), sorbitol (S) và mannitol
(M).

Hướng dẫn
1) B
2) H217O
3)

115 | Bản quyền thuộc về Tạp chí Olympiad Hóa học KEM
Bài 14
1) Vẽ cấu trúc của:
(a) Polysaccharide được tạo thành bằng cách nối các đơn vị D-
mannose qua các cầu 1→4-β-glycoside;
(b) Polysaccharide được tạo thành bằng cách nối các đơn vị D-
glucose qua các cầu in 1→6-α-glycoside.
Polysaccharide ở ý (b) là dextran, một thành phần của mảng bám răng.
2) Xử lí chitin với H2O, OH- để thủy phân các cầu amide của nó, tạo thành
hợp chất gọi là chitosan. Xác định cấu trúc chitosan. Chitosan được
dùng trong các loại dầu gội, sợi để khâu và băng vết thương.
3) Vẽ cấu trúc các hợp chất sau:
a) Một polysaccharide được tạo thành bằng cách gắn các đơn vị D-
glucosamine bởi các cầu 1→6-α-glycoside.
b) Một disaccharide được tạo thành bằng cách gắn D-mannose và D-
glucose bởi cầu 1→4-β-glycoside.
c) Một α-N-glycoside được tạo thành từ D-arabinose và C6H5CH2NH2.
d) Một ribonucleoside tạo thành từ D-ribose và thymine.
Hướng dẫn
1)

116 | Bản quyền thuộc về Tạp chí Olympiad Hóa học KEM
2)

3)

117 | Bản quyền thuộc về Tạp chí Olympiad Hóa học KEM
Bài 15
1. Một nhóm các oligosaccarit có tính chất rất thú vị có thể được cô lập
từ loài vi khuẩn Bacillus macerans khi nuôi cấy chủng vi khuẩn này
trong môi trường giàu tinh bột. Tất cả các oligosaccarit này đều không
khử. Thủy phân hoàn toàn trong axit cho 6 – 8 phân tử glucozơ. Metyl
hóa hoàn toàn rồi thủy phân chỉ thu được duy nhất 2,3,6-tri-O-metyl-D-
glucozơ. Hãy đề nghị một cấu trúc tổng quát cho oliosaccarit này. Biết
nó chỉ có thể phân cắt bằng enzym α – glucozidaza.
2. Xác định cấu trúc của một pentose nếu biết rằng nó phản ứng với Ac2O
cho tetraaxetat, phản ứng được với thuốc thử Tollens và tác dụng với
phenylhydrazin cho sản phẩm không quang hoạt
Hướng dẫn
1. Việc oligosaccarit không có tính khử chứng tỏ nhiều khả năng nó có
cấu trúc vòng. Dựa trên sản phẩm metyl hóa cho thấy liên kết glycozit
tạo thành giữa C1 và C4. Mặt khác do chỉ bị thủy phân bởi α –
glycozidaza chứng tỏ liên kết là α – glycozit. Vậy cấu trúc chung của
nhóm oligosaccarit này có dạng (n: 3 – 5).

2. Khi tác dụng với phenylhydrazin cho sản phẩm không quang hoạt tức
sản phẩm cuối không có C*. Điều này chỉ xảy ra khi hai nhóm CH2OH
cùng gắn trên một cacbon. Như vậy cấu trúc của pentose sẽ như sau:

118 | Bản quyền thuộc về Tạp chí Olympiad Hóa học KEM
Glycoside
Bài 1
1) Salicin là một chất giảm đau tự nhiên, có trong vỏ cây liễu và được sử
dụng từ hàng nghìn năm nay để xử lí các vết thương và hạ sốt.

a) Salicin có phải là đường khử không?


b) Xác định các sản phẩm tạo thành khi thủy phân salicin khi có mặt
acid.
c) Salicin là α-glycoside hay β-glycoside?
d) Vẽ sản phẩm chính được tạo thành khi xử lí salicin với lượng dư
acetic acid khi có mặt pyridine.
e) Theo bạn thì salicin có thể hiện tính chất nghịch đảo đường khi hòa
tan trong nước trung tính không?
2) Trình bày cơ chế phản ứng sau:

3) Vẽ cấu trúc α-N-glycoside và the β-N-glycoside tạo thành khi xử lí D-


glucose với aniline (C6H5NH2).
4) Vẽ cấu trúc nucleoside tạo thành từ mỗi cặp hợp chất sau. Gọi tên
nucleoside:
(a) 2-Deoxy-d-ribose và adenine
119 | Bản quyền thuộc về Tạp chí Olympiad Hóa học KEM
(b) d-Ribose và guanine
5) (a) Xác định tất cả các nguyên tử O thuộc phần glycoside trong
rebaudioside A. Rebaudioside A, được bán dưới tên thương mại Truvia,
là một glycoside ngọt, thu được từ cây stevia, loại cây đã được người
Paraguay sử dụng trong nhiều thế kỉ với mục đích làm ngọt thực phẩm.
(b) Alcohol được tạo thành từ sự thủy phân của một glycoside gọi là
aglycon. Aglycol và các monosaccharide nào được tạo thành bởi sự
thủy phân rebaudioside A?

120 | Bản quyền thuộc về Tạp chí Olympiad Hóa học KEM
Hướng dẫn
1)
a) Không phải đường khử do vị trí anomer có một nhóm acetal.
b) Nhóm acetal bị thủy phân, tạo thành hai anomer của hemiacetal vòng.

c) Salicine là β-glycoside.
d)

e) Không. Khi không có xúc tác acid thì nhóm acetal không dễ bị thủy
phân.

121 | Bản quyền thuộc về Tạp chí Olympiad Hóa học KEM
2)

3)

4)

122 | Bản quyền thuộc về Tạp chí Olympiad Hóa học KEM
5)

123 | Bản quyền thuộc về Tạp chí Olympiad Hóa học KEM
Bài 2
1) Xác định sản phẩm của các phản ứng sau:

2) Trình bày cơ chế chuyển hóa β-D-glucose thành cả hai anomer của N-
ethyl glucopyranoside.

3) Aglycon và các monosaccharide nào được tạo thành khi thủy phân
mỗi chất: sacilin và solanine trong dung dịch acid?

4) Linamarin được tìm thấy trong sắn. Giải thích tại sao hợp chất này có
độc tính?

124 | Bản quyền thuộc về Tạp chí Olympiad Hóa học KEM
5) Vanillin được tìm thấy trong một β-glycoside của D-glucosoe. Vẽ cấu
trúc của glycoside.

6) Các phenyl glycoside được tạo thành từ phenol (trong tự nhiên hoặc
trong phòng thí nghiệm) như thế nào? Tại sao cấu hình của glycoside
không liên quan đến đường ban đầu?

125 | Bản quyền thuộc về Tạp chí Olympiad Hóa học KEM
Hướng dẫn
1)

2)

3)

126 | Bản quyền thuộc về Tạp chí Olympiad Hóa học KEM
4) Aglycone của linamarin là cyanohydrin của acetone. Cyanohydrin kém
bền, nó bị thủy phân thuận nghịch giải phóng HCN, là chất có độc tính cực
mạnh.
5)

6) Hemiacetal tạo thành một oxonium ion phẳng có thể cộng hợp phenol
từ mặt trên hoặc mặt dưới. Mặt dưới được ưu tiên hơn bởi hiệu ứng
anomer và sự tạo thành acetal chiu sự khống chế nhiệt động học.

127 | Bản quyền thuộc về Tạp chí Olympiad Hóa học KEM
Bài 3
Giải thích tại sao ở cùng pH thì methyl β-D-2-deoxyglucopyranoside (I) bị
thủy phân nhanh hơn methyl β-D-glucopyranoside (II)?

Hướng dẫn
Cả hai hợp chất đều tạo thành oxocarbocation trong phản ứng thủy phân.
Nguyên tử anomer carbon mang điện tích trong hợp chất I (một acetal) có
một nguyên tử hydrogen liên kết với nó, trong khi đó nguyên tử carbon
tương ứng trong hợp chất II (một ketal) lại gắn với một carbon. Do đó,
carbocation của hợp chất II bền hơn và trạng thái chuyển tiếp dẫn đến sự
tạo thành nó có năng lượng thấp hơn hợp chất I.

128 | Bản quyền thuộc về Tạp chí Olympiad Hóa học KEM
Bài 4
Rutinose là một loại đường có trong
thành phần một số bioflavonoid
(flavonoid sinh học), các hợp chất
được tìm thấy trong những loại thực
vật có giá trị y học với việc duy trì sức
khỏe tim mạch nói chung và cụ thể
là với sức mạnh của thành mạch
máu. Rutin là một bioflavonoid chứa
rutinose được tìm thấy trong kiều
mạch và bạch đàn. Hesperidin là
một chất khác, có nguồn gốc từ vỏ
chanh và cam. Mỗi chất chứa
rutinose liên kết với aglycon ba vòng.

Sử dụng các thông tin sau để suy ra cấu trúc của đường rutinose.
a) Rutinose là mộ đường khử mà khi thủy phân acid tạo thành một đương
lượng của mỗi chất D-glucose và đường A, có công thức C6H12O5.
Đường A phản ứng với bốn đương lượng HIO4 tạo thành bốn đương
lượng formic acid và một đương lượng acetaldehyde. Ở giai đoạn này,
chúng ta có thể kết luận gì về đường A?
b) Đường A có thể được tổng hợp từ L-(-)-mannose theo sơ đồ dưới. Giai
đoạn 3 (được kí hiệu bởi dấu hoa thị) là một phản ứng đặc biệt chuyển
hóa alcohol bậc một đầu mạch thành carboxylic acid. Kết quả này thể
hiện điều gì về các tâm lập thể trong đường A?

129 | Bản quyền thuộc về Tạp chí Olympiad Hóa học KEM
c) Methyl hóa hoàn toàn rutinose với lượng dư dimethyl sulfate tạo thành
dẫn xuất heptamethyl hóa. Thủy phân acid êm dịu tiếp thì thu được
một đương lượng của 2,3,4-tri-O-methyld-glucose và một đương lượng
của dẫn xuất 2,3,4-tri-O-methyl của đường A. Xác định các cấu trúc có
thể có của rutinose phù hợp với những dữ kiện này.
Hướng dẫn
a) Kết quả của thoái phân HIO4 cho ta biết điều gì? Mỗi đương lượng HIO4
phân cắt một liên kết giữa hai nguyên tử carbon nối với nhau bởi cầu
oxygen. Formic acid tạo thành hoặc từ nhóm formyl đầu mạch, hoặc
bởi nhóm hydroxy bậc hai bên trong. Acetaldehyde là một sản phẩm
thoái phân bất thường. Sự tạo thành nó cho thấy có nhóm thế methyl
đầu mạch, gắn vào nguyên tử carbon chứa hydroxy bậc hai. Cấu trúc
của A được sắp xếp logic lại như dưới đây. Cấu trúc này quả thực có
công thức phân tử C6H12O5.

130 | Bản quyền thuộc về Tạp chí Olympiad Hóa học KEM
2)

A là 6-deoxy-L-mannose.
3)

131 | Bản quyền thuộc về Tạp chí Olympiad Hóa học KEM
Bài 5
Glycoside A (C20H27NO11), được tìm thấy trong hạt Rosaceae (họ Hoa
hồng) có kết quả âm tính với các dung dịch Benedict hoặc Fehling. Thủy
phân xúc tác enzyme A tạo thành (-)B, C8H7NO và C, C12H22O11 nhưng nếu
thủy phân hoàn toàn với xúc tác acid thì tạo thành các sản phẩm hữu cơ,
(+)D, C6H12O6 và (-)E, C8H8O3. C có một liên kết β-glycosidic và có kết quả
dương tính với dung dịch Benedict hoặc Fehling. Methyl hóa C với
MeI/Ag2O tạo thành C20H38O11, chất này khi thủy phân xúc tác acid tạo
thành 2,3,4-tri-O-methyl-D-glucopyranose và 2,3,4,6-tetra-O-methyl-D-
glucopyranose. (±)B có thể được điều chế từ benzaldehyde và NaHSO3,
sau đó xử lí với NaCN. Thủy phân xúc tác acid (±)B tạo thành (±)E.
1) Xác định cấu trúc các chất A-D với hóa lập thể phù hợp trong công
thức chiếu Haworth, ngoại trừ B.
Glycoside A được phát hiện thấy có độc tính và nguyên nhân được cho là
bởi chất F kịch độc, được tạo trong các môi trường thủy phân. Sự khử độc
hợp chất F trong thực vật có thể đi kèm với các phản ứng sau (không biểu
diễn hóa lập thể):

Một lượng nhỏ hợp chất F trong cơ thể người được cho là bi khử độc bởi
phản ứng trực tiếp với cystine tạo thành L-cysteine và hợp chất I,
C4H6N2O2S - chất này bị bài tiết trong nước tiểu (không biểu diễn hóa lập
thể).

132 | Bản quyền thuộc về Tạp chí Olympiad Hóa học KEM
Hợp chất I không có hấp thụ ở 2150-2250 cm-1 trong phổ hồng ngoại,
nhưng có một dải ở 1640 cm-1. Ngoài ra cũng quan sát được các dải tương
ứng của nhóm carboxyl.
2) Xác định công thức phân tử của các chất F, G và công thức cấu tạo
của các chất H, I. Chỉ rõ hóa lập thể của H.

(Stretching Vibration Region = Vùng dao động kéo căng)


Hướng dẫn
1)

133 | Bản quyền thuộc về Tạp chí Olympiad Hóa học KEM
2) F = HCN, G = H2S

134 | Bản quyền thuộc về Tạp chí Olympiad Hóa học KEM
Bài 6
Hương liệu được chiết xuất từ rễ cây cam thảo ngọt hơn đường kính 50-
150 lần. Hợp chất quan trọng nhất và hàm lượng lớn nhất chịu trách nhiệm
cho độ ngọt và các tác động dược lí của cây cam thảo là glycyrrhizin
(C42H62O16). Glycyrrhizin cần 3 đương lượng NaOH để trung hòa. Khi
glycyrrhizin bị thủy phân acid thì thu được Glycyrrhizinic acid (A
(C30H46O4)) và B (C6H10O7) theo tỉ lệ mol 1:2 (Sơ đồ 1).

Khi methyl hóa glycyrrhizin bởi methyl iodide (MeI) tại mọi vị trí có thể
trước khi thủy phân thì quá trình thủy phân tạo thành A’ (methyl
glycyrrhizinate), C và D (Sơ đồ 2). B, C, D tồn tại ở dạng hỗn hợp của các
anomer.

Methyl hóa C và D với MeI tạo thành cùng một hỗn hợp đồng phân, J (Sơ
đồ 3).

C bị khử bởi LiAlH4 tạo thành K, khử hóa tiếp chất này thu được L. Phản
ứng oxid hóa cắt mạch vicinal diol L với NaIO4 tạo thành M và 2 đương
lượng formaldehyde. Khử hóa M tạo thành N. Cấu trúc và hóa lập thể của
N được xác nhận bởi quá trình tổng hợp N từ D-(-)-tartaric acid qua quá
135 | Bản quyền thuộc về Tạp chí Olympiad Hóa học KEM
trình methyl hóa, sau đó khử hóa (Sơ đồ 4). Phổ 1H NMR của L cho thấy 2
đỉnh riêng biệt ứng với các nhóm methyl. (Trong cấu trúc của L không có
đối xứng.)

1) Xác định cấu trúc của L, M, N.


2) C có thể có bao nhiêu cấu trúc? Hãy biểu diễn chúng.
Chuỗi phản ứng sau đã được tiến hành để xác định cấu trúc chính xác của
C. J bị khử thành E, và thủy phân acid E tạo thành F. Sự khử F tạo thành G,
khi oxid hóa chất này bởi NaIO4 tạo thành H, kèm theo đó là sự tạo thành
1 đương lượng formadehyde. Trong số các hợp chất từ A - I, chỉ có I không
quang hoạt (Sơ đồ 5).

3) Xác định cấu trúc của G và I.


4) Xác định cấu trúc chính xác của C trong số các cấu trúc đã biểu diễn
trong ý 2.
136 | Bản quyền thuộc về Tạp chí Olympiad Hóa học KEM
5) Xác định cấu trúc của B, D, J.
6) Xác định cấu trúc Glycyrrhizin.
Hướng dẫn
1)

2) Có thể có 2 cấu trúc:

3)

137 | Bản quyền thuộc về Tạp chí Olympiad Hóa học KEM
4) Cấu trúc 1 đúng.

138 | Bản quyền thuộc về Tạp chí Olympiad Hóa học KEM
5)

6)

139 | Bản quyền thuộc về Tạp chí Olympiad Hóa học KEM
Bài 7
Đảo Crete ở Hi Lạp là một trong những khu vực địa lí mà thói quen dinh
dưỡng của người dân đã thu hút sự chú ý như “hình mẫu về dinh dưỡng”.
Trong một nghiên cứu tiến hành ở 5 quốc gia, chế độ ăn kiêng của người
Crete có liên quan đến tỉ lệ bệnh tim mạch vành (CHD, coronary heart
disease) thấp. Tỉ lệ tử vong từ CHD là 7 trên 10000 đối tượng nghiên cứu,
trong khi đó tỉ lệ tương ứng ở Phần Lan, Mỹ, Hà Lan và Ý lần lượt là 566,
424, 317 và 200. Nguyên nhân chủ yếu là do sự tiêu thụ nhiều dầu olive,
có chứa nhiều oleuropein, một chất chống oxid hóa mạnh mẽ. Oleuropein
(A) có công thức cấu tạo như sau với R là nhóm alkylpolyphenolic:

1) Thuỷ phân có xúc tác acid oleuropin tạo thành glucose và hai hợp chất
khác gồm: polyphenolic (A1) và một monoterpenoid (A2). Hãy sử dụng
mũi tên để chỉ vào công thức của oleuropin.
a) Nguyên tử oxygen sẽ bị proton hóa trong phản ứng thuỷ phân xúc
tác acid để tạo ra polyphenolic A1.
b) Liên kết C–O bị phân cắt để tạo ra glucose.
Trong phổ khối của A1, peak tương ứng với ion phân tử có m/z = 154. Phổ
1
H-NMR của (A1) được cho dưới đây. Nhóm hydroxylic proton đã tham gia
phản ứng trao đổi nên không có mặt trong các peak dưới đây.

140 | Bản quyền thuộc về Tạp chí Olympiad Hóa học KEM
2) Với những thông tin trên hãy chỉ ra A1 là chất nào trong ba chất sau?

3) Từ công thức cấu tạo của A1 hãy chỉ ra proton nào của A1 phù hợp với
các tín hiệu trên phổ 1H–NMR.

141 | Bản quyền thuộc về Tạp chí Olympiad Hóa học KEM
Hướng dẫn

Công thức cấu tạo đúng là C

142 | Bản quyền thuộc về Tạp chí Olympiad Hóa học KEM
Bài 8
Axit turgorinic (X) là một hormon thực vật đóng vai trò điều khiển sự cử
động của lá cây. Chẳng hạn nó điều chỉnh sự cụp lại của lá cây xấu hổ khi
ta chạm vào. Hợp chất X không phản ứng với thuốc thử Fehling. Khi thủy
phân X bằng enzym β-glucozidaza thu được chất A (C6H12SO9) có tính axit
và phản ứng được với thuốc thử Fehling cùng với hợp chất B. Đun nóng B
quan sát thấy sự thoát ra CO2 và thu được pyrogalol. Metyl hóa hoàn toàn
X thu được C. Thủy phân hoàn toàn C bằng axit vô cơ thu được metyl 3,5-
dimetoxy-4-hydroxibenzoat và hợp chất D (C9H18SO9). Oxy hóa D bằng axit
nitric thu được sản phẩm chính là axit 2,3,4-trimetoxypentadioic. Hãy xác
định cấu trúc X dựa vào các dữ kiện trên.
Hướng dẫn
X tham gia phản ứng thủy phân xúc tác enzym β-glucozidaza chứng tỏ X
có tồn tại liên kết β-glycozit.
Đun nóng chất B thấy tách ra CO2, thu được pyrogalol (1,2,3-
trihydroxibenzen) và khi thủy phân hoàn toàn C bằng axit vô cơ thu được
metyl 3,5-dimetoxy-4-hydroxibenzoat chứng tỏ B là axit 3,4,5-
trihydroxibenzoic.
Tiến hành oxy hóa D (C9H18SO9) bằng axit nitric thu được axit 2,3,4-
trimetoxypentadioic. Như vậy công thức của D sẽ là.

Từ các dữ kiện trên có thể suy ra axit turgorinic như sau:

143 | Bản quyền thuộc về Tạp chí Olympiad Hóa học KEM
Bài 9
Photphoramidon là những chất ức chế tự nhiên của enzym thermolysin,
một enzym thuộc nhóm proteaza có trong các sinh vật đơn bào sống
trong vùng nước biển lạnh. Hãy đưa ra cấu trúc của hợp chất này dựa trên
những dữ kiện cho dưới đây:
- Hợp chất này là một muối dinatri có công thức phân tử
C23H32N3Na2O10P.
- Tiến hành phân cắt hợp chất này thu được dipeptit leuxyltryptopan,
đường L-rhamanozơ và muối NaH2PO4.
- Đầu N của dipeptit được nối với phân đoạn photphat qua liên kết
amit.
- Nhóm photphat liên kết với monosaccarit qua liên kết este.
- Photphoamidon là đường không khử.
- Metyl hóa hoàn toàn và thủy phân hoàn toàn photphoamidon thu
được 2,3,4-tri-O-metyl L-rhamanozơ (và các sản phẩm không
đường khác).
- Oxy hóa monosaccarit này bằng axit nitric loãng thu được một axit
monocacboxylic có mạch carbon không phân nhánh có công thức
phân tử C6H12O6
- Đơn vị L-rhamanozơ trong photphoamidon có cấu hình tuyệt đối
của các nguyên tử carbon bất đối giống như L-mannozơ. Cấu hình
của C1 trong rhamanozơ là 1S.

144 | Bản quyền thuộc về Tạp chí Olympiad Hóa học KEM
Hướng dẫn
Dựa vào các thông tin đã cho có thể xác định được số nguyên tử C và O
trong L-rhamanozơ như sau. Số C = 23 – 11 (Tryptopan) – 6 (leuxin) = 6.
Số O = 10 – O2 (tryptopan) – 1 (leuxin) – 2 (photphat) = 5. Bằng con đường
tương tự có thể xác định đường này không chứa nitơ và có công thức phân
tử C6H12O5.
Dựa trên các dữ kiện 6 và 7 cho thấy rằng nó là một hexozơ và nhóm
CH2OH thường thấy được thay bằng nhóm CH3 (không tạo dẫn xuất metyl
hóa ở C6 cũng như không thể tạo ra được axit dicaboxylic dưới tác dụng
của HNO3). Do đã biết cấu trúc của L-mannozơ nên cấu trúc của L-
rhamanozơ sẽ là:

Bên cạnh đó dữ kiện 6 cho thấy rhamanozơ tồn tại ở dạng vòng pyranozơ
(vắng mặt C5 trong sản phẩm metyl hóa). Cùng với việc cấu hình của
rhamanozơ của C1 là 1S chứng tỏ nhóm OH ở C1 mạch vòng sẽ là OH β.
Dựa vào những dữ kiện cắt mạch có thể xác định được cấu trúc của
photphoamidon sẽ là:

145 | Bản quyền thuộc về Tạp chí Olympiad Hóa học KEM
Bài 10
Hợp chất hữu cơ A (C17H18O9) là một chất độc được tạo ra trong một số
loài cây do cơ chế tự vệ chống lại sâu bọ và động vật có vú. Thủy phân
hợp chất A bằng enzym emulsin thu được hợp chất B (C11H6O3) và D-
Glucozơ. Hợp chất B có thể được tổng hợp theo sơ đồ sau:

Đun nóng trong nước thì A bị đồng phân hóa thành A’. A’ cũng bị thủy phân
bởi enzym emulsin thu được hợp chất B’ (C11H8O4).Đun nóng B’ với axit lại
thu được B.Xác định cấu trúc của A.
Hướng dẫn
Xác định B:

Cấu trúc của A là:

146 | Bản quyền thuộc về Tạp chí Olympiad Hóa học KEM
147 | Bản quyền thuộc về Tạp chí Olympiad Hóa học KEM
Bài 11
Axit turgorinic (PLMF1) là một hormon thực vật đóng vai trò điều khiển sự
cử động của lá cây. Chẳng hạn nó điều chỉnh sự cụp lại của lá cây xấu hổ
khi ta chạm vào. Hợp chất PLMF1 có tính axit, không phản ứng với thuốc
thử Felinh. Khi thủy phân PLMF1 bằng enzym β-glycozidaza thu được hợp
chất A (C6H12SO9, có tính axit, phản ứng được với thuốc thử Felinh) và hợp
chất B. Khi đun nóng B thấy CO2 tách ra và thu được pirogalol. Khi metyl
hóa hoàn toàn PLMF1 thu được hợp chất C. Thủy phân hoàn toàn C bằng
axit vô cơ thu được metyl 3,5-dimetoxi-4-hydroxibenzoat và hợp chất D
(C9H18SO9). Oxy hóa D bằng axit nitric thu được sản phẩm chính là axit
2,3,4-trimetoxypentandioic.
a) Hãy xác định công thức của các hợp chất A, B, D và PLMF1.
b) Trong tự nhiên, nhóm cacboxyl của PLMF1 bị este hóa bằng nhóm OH
ở vị trí số 3 của B tạo ra hợp chất E thuộc nhóm depsides (tiếng Hy Lạp có
nghĩa là có tính thuộc da). Xác định công thức của E.
Hướng dẫn
Khi thủy phân PLMF1 bằng β-glycozidaza chứng tỏ trong PLMF1 có liên
kết β-glucozit. Từ dữ kiện khi đun nóng chất B thấy tách ra CO2 cho
pirogalol (1,2,3-trihydroxibenzen) và khi thủy phân hoàn toàn chất C bằng
axit vô cơ thu được metyl 3,5-dimetoxy-4-hydroxibenzoat chứng tỏ B là
axit 3,4,5-trihydroxibenzoic.

Khi oxy hóa D (C9H18SO9) bằng axit nitric thu được axit 2,3,4-
trimetoxypentadioic. Vậy D có công thức như sau:

148 | Bản quyền thuộc về Tạp chí Olympiad Hóa học KEM
Từ dữ kiện trên suy ra công thức của PLMF1 là:

b) Công thức của E là

149 | Bản quyền thuộc về Tạp chí Olympiad Hóa học KEM
Bài 12
Avenacin A-1 là glycoside
thuộc nhóm Saponin được
tìm thấy trong yến mạch với
tác dụng ức chế sự phát triển
của một số mầm bệnh nấm.
Thủy phân Avenacin A-1 bởi
enzyme α-glycosidase tạo thành X1 (C17H30O15) và Y (cấu trúc như hình
bên). Thủy phân X1 bởi enzyme β-glycosidase thu được 2
monosaccharide quen thuộc X2 và X3 (C6H12O6, là aldohexose thuộc dãy
D).
a. Xác định công thức phân tử của X2.
Thoái phân Ruff X3 rồi oxi hóa sản phẩm bằng HNO3 cho X4 không
quang hoạt, là dẫn xuất của pentandioic acid. Methyl hóa hoàn toàn X1
bằng MeI dư/Ag2O và thủy phân sản phẩm bằng HCl thu được X5 và X6
(C6H12O5). Oxi hóa cắt mạch X5 bằng HNO3 cho 2,3,4-
trimethoxypentandioic acid (X7) không quang hoạt, (2R,3R)-2,3-
dimethoxybutandioic acid (X8) và methoxyaxetic acid.
b. Xác định công thức chiếu Fischer của X5, X7 và X8.
X6 tồn tại ở dạng vòng pyranose (đây cũng là dạng tồn tại của hợp
phần đường tương ứng trong các chất đã nêu), khi xử lí với MeI dư/Ag2O
rồi đun nóng với dung dịch HNO3 tạo thành X8 và X9 là một epimer của
X7. Nếu xử lí X6 với NaBH4 cho X10. Nếu thay đổi cấu hình tại một nguyên
tử carbon bất đối trong X10 sẽ thu được X11 không quang hoạt.
c. Xác định công thức chiếu Fischer của X6, X9, X10 và X11.
d. Vẽ công thức Haworth của hợp phần glycone trong Avenacin A-
1.
Hướng dẫn

a. X1 (C17H30O15) X2 + X3 (C6H12O6)
Do X2, X3 là hai đường đơn, mặt khác X1 chứa số lẻ carbon nên X2 có
số lẻ carbon, quá trình thủy phân 1 phân tử X1 sẽ tạo thành 2 phân tử X3

150 | Bản quyền thuộc về Tạp chí Olympiad Hóa học KEM
và 1 phân tử X2 với sự thủy phân 2 liên kết β-glycoside. Quá trình này được
mô tả đầy đủ như sau:

X1 (C17H30O15) + 2H2O X2 + 2X3 (C6H12O6)


Do đó, trong X2 có số C bằng 17 – 6.2 = 5; số H bằng 30 + 2.2 – 12.2
= 10; số O bằng 15 + 2 – 6.2 = 5.
Vậy công thức phân tử của X2 là C5H10O5.
b. + X6 chứa 5O, 6C nên là dẫn xuất mono-O-methyl của X2 còn X5 là dẫn
xuất của aldohexose X3. X3 thuộc dãy D nên C5 của X3 có cấu hình R. Sơ
đồ của quá trình hình thành X4:

X4 quang hoạt, chứng tỏ C2 của X4 và C3 của X3 đều có cấu hình S,


cùng cấu hình với C4 của X4 để khi đó X4 không có mặt phẳng đối xứng.
+ HNO3 phân cắt mạch ở vị trí nguyên tử carbon gắn với nhóm OH, mặt
khác từ X5 còn tạo ra X7 chứa ba nhóm methoxy kế nhau nên nhóm OH
tự do trong X5 gắn với C5. Từ sản phẩm phân cắt mạch X7, X8 và cấu hình
(3S, 5R) suy ra công thức chiếu Fischer của X5:

c. + X6 tồn tại ở dạng pyranose nên X2 phải là aldopentose, vòng pyranose


tạo bởi nhóm OH ở C5 với nhóm aldehyde ở C1. Sau quá trình methyl hóa
151 | Bản quyền thuộc về Tạp chí Olympiad Hóa học KEM
các nhóm OH tự do trong X6 bằng MeI/Ag2O, khi xử lí với HNO3 ban đầu
cho sản phẩm thủy phân chứa OH-hemiacetal. Sự oxi hóa sản phẩm này
tạo X8 và một epimer của X7, do đó cấu trúc của sản phẩm ở dạng mạch
hở như sau:

+ X6 tác dụng với NaBH4 cho X10 chứa 4 nhóm OH và 1 nhóm OMe. Do
thay đổi cấu hình của 1 nguyên tử carbon bất đối trong X10 tạo ra X11
không quang hoạt nên nhóm methoxy phải gắn với nguyên tử C3 trong
mạch carbon của X6 và X10. Vậy cấu trúc của X6, X10 và X11 là:

d. Từ cấu trúc của X5 và X6 suy ra 2 hợp phần glucose đều tồn tại ở dạng
pyranose và tạo liên kết β-glycoside với các nhóm OH ở C2, C4 của hợp
phần X2. Liên kết của X2 với Y là liên kết α-glycoside. Vậy công thức
Haworth của hợp phần glycone trong Avenacin A-1 như sau:

152 | Bản quyền thuộc về Tạp chí Olympiad Hóa học KEM
153 | Bản quyền thuộc về Tạp chí Olympiad Hóa học KEM
Bài 13
Albutin có công thức C12H16O7. Khi xử lý chất
này với axit hay với enzym α-glycozidaza thu
được D-glucozơ và hợp chất X có công thức
C6H6O2. Metyl hóa hoàn toàn albutin rồi thủy
phân trong môi trường axit thu được2,3,4,6-
tetra-O-metyl-D-glucozơ và hợp chất Y
(C7H8O2). Xử lý Y bằng NaOH / (CH3)2SO4 thu
được Z (C8H10O2). Biết rằng khi oxy hóa X rồi chiếu sáng thu được sản
phẩm có cấu trúc như hình bên. Hãy xác định công thức của X, Y, Z và
albutin
Hướng dẫn
Sản phẩm được tạo thành khi chiếu sáng chính là dime của quinon, như
vậy X phải là dihydroquinon, Y là monometylete và Z là dimetylete của
hydroxiquinon.
Metyl hóa hoàn toàn albutin rồi thủy phân thu được được2,3,4,6-
tetra-O-metyl-D-glucozơ, tức nhóm OH C1 đã bị thế.
Albutin chỉ bị thủy phân bằng α-glycozidaza tức liên kết là α-
glycozit.
Như vậy cấu trúc các chất như sau:

154 | Bản quyền thuộc về Tạp chí Olympiad Hóa học KEM
Bài 14
Trong những năm gần đây, phương pháp chữa trị ung thư bằng những
“viên đạn thuốc” đang rất được quan tâm. Chúng gồm 2 phần chính là
phần truyền dẫn và phần dược tính. Trong đó phần truyền dẫn có ái lực
đặc biệt với tế bào ung thư sẽ nhận trách nhiệm “mang thuốc” đến vị trí
ung thư. Phương pháp này không chỉ giúp tăng khả năng khỏi bệnh mà
còn giảm các phản ứng phụ.
Để kiểm tra khả năng truyền dẫn thì một nhóm nghiên cứu đã tiến hành
tổng hợp dẫn xuất A theo sơ đồ sau:

1. Vẽ cấu trúc các chất B, C, D ở dạng vòng phẳng


2. Cho biết tác nhân nào đã được sử dụng trong bước cuối cùng?
3. Dự đoán kết quả thu được khi thủy phân A trong kiềm loãng
Hướng dẫn
1. Cấu trúc các chất B, C, D

2. Pirole

155 | Bản quyền thuộc về Tạp chí Olympiad Hóa học KEM
3. Khi thủy phân trong kiềm sẽ tạo 16 nhóm OH nên tăng mạnh tính
tan của sản phẩm thủy phân.

156 | Bản quyền thuộc về Tạp chí Olympiad Hóa học KEM
Bài 15
Một số giống cá nóc, Fugu trong tiếng Nhật, là loại thực phẩm được đánh
giá cao ở Nhật Bản. Do nội tạng (đặc biệt là buồng trứng và gan) của loài
cá này có chứa độc tố cực mạnh (tetrodotoxin), nên các vụ ngộ độc thực
phẩm liên quan đến loài cá này thường xuyên xảy ra. Các nghiên cứu về
tetrodotoxin (1) đã được thực hiện từ đầu thế kỉ 20 và vào năm 1964, cấu
trúc của nó đã được xác định rõ:

Nhóm guanidine trong tetrodotoxin thể hiện tính base mạnh. Guanidinium
ion tạo thành từ sự proton hóa nhóm guanidine được bền hóa bởi sự tồn
tại của các cấu trúc cộng hưởng sau.

1) Vẽ các cấu trúc cộng hưởng B, C.


Nhiều phản ứng dẫn xuất hóa đã được tiến hành trong nghiên cứu cấu trúc
tetrodotoxin. Xử lí tetrodotoxin (1) (xem sơ đồ 1) với ethanolic potassium
hydroxide, đun nóng có thể tạo thành dẫn xuất quinazoline 2, chất này giúp
mang tới cái nhìn về bản chất bộ khung cơ bản của tetrodotoxin. Cơ chế
phản ứng có thể được mô tả như sau đây. Trước tiên, tetrodotoxin bị thủy
phân thành carboxylate 3. Sau đó, nhóm hydroxyl trong khung vuông bị
tách loại bởi base, tạo thành trung gian D. Phản ứng retro-aldol của D phân
cắt liên kết carbon-carbon, tạo thành các trung gian E và F. Cuối cùng,
phản ứng dehydrogen hóa và thơm hóa E tạo thành dẫn xuất quinazoline
2.

157 | Bản quyền thuộc về Tạp chí Olympiad Hóa học KEM
2) Vẽ cấu trúc các trung gian giả thiết D, E, F.

Mặc dù quá trình sinh tổng hợp tetrodotoxin vẫn còn phải làm sáng tỏ hơn,
nhưng có thể nó được sinh tổng hợp từ L-arginine và isopentenyl
diphosphate.

3) Khoanh tròn các nguyên tử carbon trong tetrodotoxin không phải có


nguồn gốc từ L-arginine.
Trong thập niên 1990, một hướng sinh tổng hợp thay thế tetrodotoxin đã
được đề xuất. Phản ứng ngưng tụ 2-deoxy-3-oxo-D-pentose và guanidine
tạo thành trung gian G với hợp phần guanidine vòng (C6H11N3O3).
Tetrodotoxin có thể sinh tổng hợp từ trung gian G và isopentenyl
diphosphate.

4) Vẽ cấu trúc của trung gian giả thiết G, chỉ rõ hóa lập thể.

158 | Bản quyền thuộc về Tạp chí Olympiad Hóa học KEM
159 | Bản quyền thuộc về Tạp chí Olympiad Hóa học KEM
Hướng dẫn
1)

2)

3)

4)

Các phương án được chấp nhận:

160 | Bản quyền thuộc về Tạp chí Olympiad Hóa học KEM
Mỗi cấu trúc ion lưỡng cực như sau cũng được chấp nhận:

161 | Bản quyền thuộc về Tạp chí Olympiad Hóa học KEM
Bài 16
Các carbohydrate là những hợp phần thiết yếu trong các tế bào sống và là
nguồn năng lượng cho động vật. Chúng có thể là các chất đường đơn giản
với phân tử nhỏ hoặc cũng có thể là những chất đại phân tử. Khi nguyên
tử oxygen trong vòng trong các chất đường bị thay thế bởi một nhóm
methylene thì hợp chất tạo thành được gọi là chất giả-đường hoặc
carbasugar. Do các carbasugar bền (khó bị thủy phân) với acid và enzyme,
nên nhiều chất được ứng dụng trong việc ứng chế glycosidase.
Các quy trình tổng hợp toàn phần của 2 carbasugar đồng phân có bộ
khung 1 được mô tả dưới đây.

Quy trình tổng hợp 1 bắt đầu với sự khử benzene bởi sodium trong
ammonia lỏng, tạo thành A. Phổ 13C NMR của A có 2 tín hiệu 124.0 và 26.0
ppm. Tiểu phân hoạt động S (xem Sơ đồ 1) được tạo thành từ
trichloroacetyl chloride khi có mặt kẽm. 1 đương lượng S tham gia phản
ứng cộng vòng [2+2] với A tạo thành sản phẩm racemic B. Phản ứng của
B với Zn trong acetic acid tạo thành C. Hợp chất C chỉ chứa carbon,
hydrogen và oxygen. Phổ 13C NMR của C có 3 tín hiệu sp2 carbon ở 210.0,
126.5 và 125.3 ppm. Phản ứng của C với 1 đương lượng m-
chloroperbenzoic acid (m-CPBA) trong methylene chloride tạo thành sản
phẩm chính là D. Phổ 13C NMR của D có 3 tín hiệu trong vùng sp2 ở 177.0,
125.8, 124.0 ppm.

Các vùng chuyển dịch hóa học 13C-NMR của các nhóm chức điển hình.

162 | Bản quyền thuộc về Tạp chí Olympiad Hóa học KEM
163 | Bản quyền thuộc về Tạp chí Olympiad Hóa học KEM
Sơ đồ 1

1) Vẽ các cấu trúc A, B, C, D và trung gian S. Sự khử của D với LiAlH4 tạo
thành E, chất này phản ứng với acetyl chloride dư trong pyridine tạo
thành F (Sơ đồ 2).

2) Vẽ cấu trúc (một đối quang) của E và F, sử dụng kí hiệu đường kẻ vạch-
in đậm. Gắn các kí hiệu cấu hình tuyệt đối (R/S) cho các carbon bất
đối trong E.
Hợp chất F (sử dụng đối quang được vẽ ở trên) phản ứng với bromine tạo
thành các đối quang G1, G2.
3) Vẽ các cấu trúc của G1, G2 sử dụng kí hiệu đường kẻ vạch-in đậm.
Hỗn hợp G1, G2 phản ứng với 2 đương lượng 1,8-diazabicyclo-
[5.4.0]undec7-ene (DBU) - một amine base mạnh - tạo thành H.
4) Vẽ cấu trúc của H sử dụng kí hiệu đường kẻ vạch-in đậm.
Phản ứng của H với singlet oxygen (tạo ra tại chỗ [in situ]) tạo thành I. Mặc
dù về mặt lí thuyết có thể có 2 đồng phân nhưng thực tế thì I được tạo
thành ở dạng đồng phân tinh khiết do sự án ngữ không gian và sự đẩy
điện tử. Phản ứng của I với LiAlH4 (dư) dẫn tới sự tạo thành J (xem Sơ đồ
3). Phổ 13C NMR của J có 8 tín hiệu, 2 trong số đó thuộc vùng sp2.
Phản ứng của J với acetyl chloride dư khi có mặt pyridine tạo thành K.
Phản ứng tiếp theo của K với OsO4 khi có mặt 4-methylmorpholine 4-oxide
(NMO) tạo thành các đồng phân lập thể L và M. Khử hóa L và M với LiAlH4
thì lần lượt tạo thành 1a và 1b.
164 | Bản quyền thuộc về Tạp chí Olympiad Hóa học KEM
5) Vẽ cấu trúc của I, J, K, L, M, 1a, và 1b sử dụng kí hiệu đường kẻ vạch-
in đậm.
Hướng dẫn
1)

165 | Bản quyền thuộc về Tạp chí Olympiad Hóa học KEM
2)

3)

4)

166 | Bản quyền thuộc về Tạp chí Olympiad Hóa học KEM
5)

6)

167 | Bản quyền thuộc về Tạp chí Olympiad Hóa học KEM
Amino acid
Bài 1
1) Vẽ công thức chiếu Fischer của các amino acid sau:
(a) L-Threonine
(b) L-Serine
(c) L-Phenylalanine
(d) L-Asparagine
2) Vẽ tất cả các đồng phân lập thể của L-isoleucine. Với mỗi đồng phân,
hãy chỉ rõ cấu hình R/S của các tâm thủ tính.
3) Arginine có tính base mạnh nhất trong số 20 amino acid tồn tại tự
nhiên. Ở pH sinh lí học, nhánh phụ của arginine bị proton hóa. Xác định
nguyên tử nitrogen nào ở nhánh phụ bị proton hóa.
4) Histidine có nhánh phụ có tính base bị proton hóa ở pH sinh lí học. Xác
định nguyên tử nitrogen nào ở nhánh phụ bị proton hóa.
Hướng dẫn
1)
a) b) c) d)

2)

168 | Bản quyền thuộc về Tạp chí Olympiad Hóa học KEM
3) Sự proton hóa ở nguyên tử nitrogen được bôi màu sẽ tạo thành acid
liên hợp được bền hóa mạnh bởi cộng hưởng (điện tích dương được giải
tỏa mạnh).

4) Dạng proton hóa dưới đây có tính thơm. Ngược lại, sự proton hóa ở
nguyên tử nitrogen khác trong vòng sẽ làm mất bền hóa thơm.

169 | Bản quyền thuộc về Tạp chí Olympiad Hóa học KEM
Bài 2
Phản ứng ngưng tụ một carboxylic acid với một amine tạo thành sản phẩm
amide. Ví dụ, sự ngưng tụ formic acid với dimethylamine tạo thành N,N-
dimethylformamide (DMF), chất này có thể mô tả bởi các cấu trúc cộng
hưởng sau đây:

1) Dự đoán trật tự giảm dần nhiệt độ nóng chảy của N,N-


dimethylformamide (hợp chất A), N-methylacetamide (CH3CONHCH3,
hợp chất B), và propionamide (hợp chất C, CH3CH2CONH2).
2) Các nhóm carbonyl thường được xác định bởi các hấp thụ mạnh đặc
trưng của chúng trong phổ hồng ngoại. Vị trí của hấp thụ phụ thuộc
vào độ bền của liên kết C=O, điều này cũng phản ánh độ dài liên kết.
Trong các amide, độ bền của các nhóm carbonyl có thể được thể hiện
bởi các cấu trúc cộng hưởng cho ở trên. Ví dụ, cyclohexanone thể hiện
một hấp thụ ở 1715 cm-1 cho các nhóm carbonyl (C=O). Dự đoán dải
hấp thụ của nhóm carbonyl trong propionamide so với cyclohexanone.
Chọn một trong các phương án sau:
(a) 1660 cm-1 bởi độ dài liên kết carbonyl ngắn hơn.
(b) 1660 cm-1 bởi độ dài liên kết carbonyl dài hơn.
(c) 1740 cm-1 bởi độ dài liên kết carbonyl ngắn hơn.
(d) 1740 cm-1 bởi độ dài liên kết carbonyl dài hơn.
3) Glycine (H2N-CH2-COOH) là một α-amino acid. 3 phân tử glycine có thể
tạo thành tripeptide Gly-Gly-Gly qua các cầu amide, kèm theo sự tách
2 phân tử nước. Vẽ cấu trúc của tripeptide này.
4) Khi một α-amino acid chứa một nhóm thế thì có thể có các đồng phân
quang học. Ví dụ, L-alanine và D-alanine là hai đối quang. Tính số lượng
khả dĩ của các tripeptide thẳng có thể được tạo thành từ 3 amino acid:
glycine, L-alanine và D-alanine từ phản ứng ngưng tụ.

170 | Bản quyền thuộc về Tạp chí Olympiad Hóa học KEM
5) Trong số các tripeptide được tổng hợp trong ý 4, có bao nhiêu chất có
tính quang hoạt?
Hướng dẫn
1) Trật tự giảm dần nhiệt độ nóng chảy: C > B > A.
Cấu trúc cộng hưởng của amide cho thấy có điện tích âm riêng phần trên
oxygen và điện tích dương riêng phần trên nitrogen. Các amide bậc một
và bậc hai cũng tham gia tạo thành liên kết hydrogen mạnh, nhưng amide
bậc ba thì không. Thực tế thì nhiệt độ nóng chảy (m.p.) của các amide là:
Propionamide, m.p. = 79 °C; N-methylacetamide, m.p. = 28 °C; N,N-
dimethylformamide, m.p. = -61 °C.
2) b) 1660 cm-1 bởi độ dài liên kết carbonyl dài hơn.
3)

4) Có thể có 27 tripeptide.
5) Có 26 tripeptide có tính quang hoạt. Trong đó H2N-GGG-OH không
quang hoạt, còn lại các tripeptide như H2N-G-GLA-OH, H2N-GGDA-OH,
H2N-GLAG-OH, H2N-LAGDA-OH, H2N-LALADA-OH, … đều quang hoạt.

171 | Bản quyền thuộc về Tạp chí Olympiad Hóa học KEM
Bài 3
Bàng giá trị pKa (ghi chú: Side chan là nhánh phụ)

1) Vẽ dạng tồn tại của amino acid chiếm ưu thế ở pH được chỉ định.
(a) Alanine ở pH 10.
(b) Proline ở pH 10.
(c) Tyrosine ở pH 9.
(d) Asparagine ở pH sinh lí học.
(e) Histidine ở pH sinh lí học
(f) Glutamic acid ở pH 3.
2) Giải thích tại sao ở pH 11 thì arginine là chất nhường proton hiệu quả
hơn asparagine?
3) Sử dụng dữ kiện ở bảng pKa ở trên để tính điểm đẳng điện pI của các
amino acid sau:
(a) Aspartic acid
(b) Leucine
172 | Bản quyền thuộc về Tạp chí Olympiad Hóa học KEM
(c) Lysine
(d) Proline
4) Tiến hành điện di một hỗn hợp chứa phenylalanine, tryptophan, và
leucine. Xác định xem amino acid nào di quyển xa nhất, giả sử rằng thí
nghiệm được thực hiện ở pH: a) 6.0; b) 5.0.
5) Các amino acid quang hoạt bị racemic hóa ở vị trí α khi xử lí với base
mạnh. Đề xuất cơ chế của quan sát này.

173 | Bản quyền thuộc về Tạp chí Olympiad Hóa học KEM
Hướng dẫn
1)

2) Arginine có nhánh phụ có tính base, trong khi đó asparagine thì không.
Ở pH 11, arginine tồn tại chủ yếu ở dạng trong đó nhánh phụ bị proton hóa.
Do đó, nó đóng vai trò như một chất nhường proton.
3)
Aspartic acid: Leucine:

Lysine: Proline:

4) pI của Phe = 5.48, pI của Trp = 6.11 và Leu = 6.00. Sử dụng các giá trị
này, chúng ta thực hiện các dự đoán sau: (a) pH = 6.0 Phe sẽ di chuyển xa
nhất; (b) ở pH = 5.0, Trp sẽ di chuyển xa nhất.

174 | Bản quyền thuộc về Tạp chí Olympiad Hóa học KEM
175 | Bản quyền thuộc về Tạp chí Olympiad Hóa học KEM
5)

176 | Bản quyền thuộc về Tạp chí Olympiad Hóa học KEM
Bài 4
1) Dự đoán sản phẩm chính giữa L-valine với:
(a) MeOH, H+
(b) Di-tert-butyl-dicarbonate
(c) NaOH, H2O
(d) HCl
2) Trình bày cơ chế của phản ứng sau:

3) Khi xử lí hợp chất sau với HCl đặc ở 100 oC trong nhiều giờ thì xảy ra
sự thủy phân, tạo thành một trong 20 amino acid tồn tại trong tự nhiên.
Xác định amino acid đó.

177 | Bản quyền thuộc về Tạp chí Olympiad Hóa học KEM
Hướng dẫn
1)
a) b)

c) d)

2)

178 | Bản quyền thuộc về Tạp chí Olympiad Hóa học KEM
3)

179 | Bản quyền thuộc về Tạp chí Olympiad Hóa học KEM
Bài 5
Các L-α-amino acid đóng vai trò quan trọng
trong tất cả các sinh vật. Histidine (dưới đây) có
giá trị pKa2 là 6.00. Nó là amino acid duy nhất
góp phần vào khả năng đệm của máu (pH =
7.40). Glutamic acid (2-aminopentane diacid)
và aspartic acid (2-aminobutan diacid) là các
chất dẫn truyền thần kinh quan trọng. Cysteine
(2-amino-3-mercapto propanoic acid, mercapto
= thiol) đóng vai trò quan trọng trong cấu trúc protein bởi nó có thể tạo
thành các cầu disulfide bền. Hai phân tử cysteine có thể liên kết với nhau
qua một cầu disulfide tạo thành dicysteine (cystine).
a) Tiểu phân nào của histidine tồn tại trong cân bằng ở các giá trị pH 1.82
và 9.17 (đó cũng là các giá trị pKa khác của histidine)? Chỉ có duy nhất
một trong hai nguyên tử nitrogen trong vòng có thể bị proton hóa, đó
là nguyên tử nào?
b) Vẽ cấu trúc L-glutamic acid theo công thức chiếu Fischer.
c) Vẽ cấu trúc 3D của S-aspartic acid và R-cysteine.
Trong các protein và peptide, các amino acid liên kết với nhau bởi các liên
kết peptide - hợp chất tạo thành được gọi là carboxylic acid amide. Trong
một số giáo trình, phản ứng được viết như sau:

Phản ứng chỉ thực sự diễn ra theo cách này trong những điều kiện khắc
nghiệt bởi carboxylic acid không đủ hoạt tính với các tác nhân nucleophile
(hoạt tính carbonyl thấp).
d) Hiệu ứng điện tử nào chịu trách nhiệm cho hoạt tính carbonyl thấp của
các carboxylic acid.

180 | Bản quyền thuộc về Tạp chí Olympiad Hóa học KEM
Để làm tăng hoạt tính của các carboxylic acid, chúng có thể được chuyển
thành các halide hoặc anhydride.
e) Trình bày cơ chế tạo thành amide giữa tác nhân hoạt động RCOX và
amine R’NH2.
f) Viết phương trình phản ứng tạo thành dicysteine từ cysteine (không
cần xét đến hóa lập thể). Phản ứng thuộc loại nào?
Hướng dẫn
a) Chỉ có nguyên tử nitrogen có liên kết đôi với nguyên tử carbon được
proton hóa. Cặp electron tự do của nguyên tử N còn lại thuộc hệ thơm
và do đó chỉ có tính base yếu.

b)

c)

d) Hợp phần OH của nhóm carboxyl gây ra hiệu ứng +M vào nguyên tử
carbonyl carbon và làm giảm tính electrophile của nó.

181 | Bản quyền thuộc về Tạp chí Olympiad Hóa học KEM
e)

f) Đây là loại phản ứng oxid hóa-khử (chính xác là: ghép cặp oxid hóa):

182 | Bản quyền thuộc về Tạp chí Olympiad Hóa học KEM
Bài 6
Amino acid là các khối cấu trúc của protein. Sự có mặt của các nhóm -
NH2 và -COOH tạo nên bản chất lưỡng tính của amino acid. Một số nhánh
amino acid trong các protein cực kì quan trọng với hoạt tính và vai trò xúc
tác của chúng. Glutamic acid (dưới đây) là một trong các amino acid như
vậy.

a) Tại sao pKa của nhóm α-COOH thấp hơn nhóm γ-COOH?
b) Tính % nhóm γ-COOH vẫn ở dạng không bị ion hóa ở pH 6.3.
c) Glutamic acid được tiêm vào giấy điện di ở pH = 3.25. Nó sẽ di chuyển
về andoe (+) hay cathode (-)? Tại sao?
Sự thủy phân các polysaccharide như chitin, cellulose và peptidoglycan là
một quá trình hóa sinh phổ biến. Quá trình này liên quan đến sự thủy phân
các liên kết glycosidic như cầu β-1,4 dưới đây.

Một phản ứng thủy phân như vậy được xúc bởi lysozyme.
d) Giả sử rằng phản ứng xúc tác bởi lysozyme được tiến hành trong nước
được làm giàu với 18O, hãy dự đoán 18O có được đưa vào sản phẩm
không? Nếu có thì ở đâu?
Đồ thị pH-hoạt tính của lysozyme được cho trong hình dưới đây.

183 | Bản quyền thuộc về Tạp chí Olympiad Hóa học KEM
e) Giải thích đồ thị này theo 2 carboxylate (Asp-52 và Glu-35) có ở tâm
hoạt động lysozyme (Chú ý: các nhóm có thể ion hóa trên chất nền
không tham gia). Viết trạng thái lí tưởng của quá trình ion hóa ở tâm
hoạt hóa lysozme tại pH 5.0.
f) pKa của Glu-35 trong tâm hoạt động lysozyme là 6.0 chứ không phải
4.3 như được tìm thấy ở amino acid tự do. Những hiệu ứng cục bộ nào
sau đây có liên quan?
1. Điện tích âm được tăng cường.
2. Điện tích dương được tăng cường.
3. Độ phân cực được tăng cường.
4. Độ phân cực suy giảm.
Các phản ứng hữu cơ kiểu mẫu đã giúp hiểu được nhiều đặc tính của các
cơ chế xúc tác enzyme. Với một phản ứng nội phân tử (giống như các xúc
tác enzyme thực hiện!) thì sự gia tăng tốc độ diễn ra khi nồng độ khả kiến
của tác nhân ở tâm tăng lên rất nhiều. Nhóm carboxylate hỗ trợ cho phản
ứng thủy phân của 3 phenylacetate và các hằng số tốc độ (k) được cho
dưới đây

184 | Bản quyền thuộc về Tạp chí Olympiad Hóa học KEM
(pseudo first order = giả bậc một)

g) Tính nồng độ cục bộ hiệu dụng của nhóm COO- trong (2) và (3) ở trên.
h) Tại sao hằng số tốc độ của (3) cao hơn (2)?
Hướng dẫn
a) Nhóm amino bị proton hóa gây ra hiệu ứng hút electron. Hiệu ứng này
làm tăng khả năng giải phóng proton từ nhóm -COOH gần đó, bởi nó
giúp bền hóa base liên hợp -COO-. Hiệu ứng này càng lớn khi -COO-
càng gần -NH3+. Do nhóm -NH3+ có mặt ở α-carbon nên hiệu ứng tác
động lên α-COOH hơn là γ-COOH. Do đó pKa của α-COOH thấp hơn γ-
COOH.
b) Tỉ lệ nhóm γ-COOH bị ion hóa/chưa bị ion hóa nhận được bằng cách
sử dụng phương trình Henderson-Hasselbalch.

pH = 6.3 và pKa của nhóm γ-COOH là 4.3. Thế các giá trị này vào phương
trình trên, ta có:

185 | Bản quyền thuộc về Tạp chí Olympiad Hóa học KEM
c) Glutamic acid có hai giá trị pKa thấp hơn 7.0 và một giá trị pKa cao hơn
7.0. Do đó, điểm đẳng điện (pI) của glutamic acid sẽ nằm giữa 2 giá
trị pKa có tính acid: pI = (2.2 + 4.3)/ 2 = 3.25. Ở pH 3.25, điện tích toàn
phần của glutamic acid sẽ bằng 0 do pH này trùng khớp với pI của
glutamic acid. Do đó, glutamic acid sẽ đứng yên ở pH 3.25.
d) Trong quá trình thủy phân liên kết glycosidic, oxygen cầu glycosidic
tách ra cùng C4 trên đường B. Sau khi phân cắt, 18O từ nước sẽ được
tìm thấy trên C1 của đường A.

Chú ý: Phản ứng diễn ra với một carbonium ion được bền hóa trên C1 của
đường A.
e) Đa số các glycosidase chứa 2 nhóm carboxylate ở tâm hoạt động của
đều rất quan trọng về mặt xúc tác. Lysozyme hoạt động chỉ khi 1 nhóm
carboxylate được proton hóa và nhóm khác bị deproton hóa. Bờ giảm
dần ở phía kiềm trên đồ thị pH là do sự ion hóa -COOH. Bờ tăng dần ở
phía acid là do sự proton hóa -COO-. Do đó, hoạt tính enzyme giảm
mạnh ở hai phía của pH tối ưu. Trạng thái lí tưởng của sự ion hóa ở pH
= 5 sẽ là

186 | Bản quyền thuộc về Tạp chí Olympiad Hóa học KEM
f) 2 và 4 đều đúng. Sự ion hóa của -COOH dẫn đến sự tạo thành tiểu phân
mang điện tích âm -COO-. Tiểu phân mang điện này bị kém bền hóa
bởi sự phân cực suy giảm và điện tích âm được tăng cường. Do đó, sự
ion hóa nhóm -COOH bị ức chế và giá trị pKa được nâng lên.
g) Tỉ lệ của hằng số giả-bậc 1 (ở CH3COO- 1 M) trong (a) với các hằng số
bậc 1 trong (b) và (c) cho biết các nồng độ cục bộ hiệu dụng. Ví dụ:
(2) (0.4) / (0.002) = 200, nghĩa là nồng độ hiệu dụng = 200 M
(3) (20) / (0.002) = 10000 nghĩa là nồng độ = 10000 M
h) Ngoài hiệu ứng nồng độ cục bộ được tăng cường, nhóm -COO- trong
(3) còn được định hướng tốt hơn để tác động trong xúc tác. Liên kết
đôi ngăn trở chuyển động của -COO- và do đó làm giảm số lượng định
hướng không phù hợp của -COO-, dẫn đến sự tăng tốc độ phản ứng.

187 | Bản quyền thuộc về Tạp chí Olympiad Hóa học KEM
Bài 7
2) Xác định các tác nhân cần thiết để tổng hợp các amino acid sau bởi
phản ứng Hell-Volhard-Zelinsky:
(a) Leucine
(b) Alanine
(c) Valine
3) Xác định các tác nhân cần thiết để tạo nên các amino acid qua tổng
hợp amidomalonate:
(a) Isoleucine
(b) Alanine
(c) Valine
4) Xác định các tác nhân cần thiết để tạo nên các amino acid sau sử
dụng phương pháp tổng hợp Strecker:
(a) Methionine
(b) Histidine
(c) Phenylalanine
(d) Leucine
5) Mỗi aldehyde sau được chuyển hóa thành α-amino nitrile sau đó thủy
phân tạo thành amino acid. Trong mỗi trường hợp, hãy vẽ công thức
và gọi tên amino acid được tạo thành.
(a) Acetaldehyde
(b) 3-Methylbutanal
(c) 2-Methylpropanal
Hướng dẫn
1)
(a) (b)

(c)

188 | Bản quyền thuộc về Tạp chí Olympiad Hóa học KEM
2)

3)
a) b)

c) d)

189 | Bản quyền thuộc về Tạp chí Olympiad Hóa học KEM
4)
a) b)

c)

190 | Bản quyền thuộc về Tạp chí Olympiad Hóa học KEM
Bài 8
Xác định các sản phẩm hữu cơ được tạo thành trong mỗi phản ứng sau:

Hướng dẫn

191 | Bản quyền thuộc về Tạp chí Olympiad Hóa học KEM
Bài 9
1) Khi điều chế leucine bởi tổng hợp amidomalonate thì thu được sản
phẩm khí phụ là isobutylene (còn gọi là 2-methylpropene). Trình bày
cơ chế phản ứng tạo thành sản phẩm phụ này.
2) Xác định các trung gian được kí hiệu bằng chữ cái sau đây. Đây là một
phương pháp thay thế để tổng hợp các amino acid, dựa vào tổng hợp
Gabriel của amine bậc 1.

3) Glutamic acid được tổng hợp theo chuỗi phản ứng sau. Trình bày cơ
chế cho các bước [1]-[3].

Hướng dẫn
1) Enolate ion (tạo thành ở giai đoạn đầu tiên) có thể đóng vai trò như một
base, thay vì nucleophile, dẫn đến phản ứng tách E2:

2)

192 | Bản quyền thuộc về Tạp chí Olympiad Hóa học KEM
3)

193 | Bản quyền thuộc về Tạp chí Olympiad Hóa học KEM
Bài 10
1) Viết sơ đồ phân giải hai đối quang của racemic lacic acid
[CH3CH(OH)COOH] sử dụng tác nhân phân giải (R)-α-
methylbenzylamine.
2) Một chiến lược khác được sử dụng để phân giải các amino acid bao
gồm việc chuyển nhóm carboxy thành ester, sau đó sử dụng
carboxylic acid thủ tính để thực hiện phản ứng acid-base ở nhóm
amino tự do. Sơ đồ tổng quát dưới đây sử dụng (R)-mandelic acid làm
tác nhân phân giải. Sử dụng một hỗn hợp đối quang alanine và (R)-
mandelic acid làm tác nhân phân giải, hãy cho biết quá trình phân giải
được thực hiện như thế nào.

3) Brucine là một alkaloid độc tính được phân lập từ Strychnos nux
vomica, một loại cây sinh trưởng ở Ấn Độ, Ski Lanka và Bắc Úc. Trình
bày sơ đồ phân giải brucine bởi hỗn hợp racemic phenylalanine.

194 | Bản quyền thuộc về Tạp chí Olympiad Hóa học KEM
Hướng dẫn
1)

195 | Bản quyền thuộc về Tạp chí Olympiad Hóa học KEM
2)

196 | Bản quyền thuộc về Tạp chí Olympiad Hóa học KEM
3)

197 | Bản quyền thuộc về Tạp chí Olympiad Hóa học KEM
Bài 10A
Amide của alanine có thể được phân giải bởi acylase trong thận lợn. Đối
quang nào của alanine bị acyl hóa với acetic anhydride nhanh hơn? Trong
phản ứng thủy phân xúc tác enzyme, đối quang nào thủy phân nhanh hơn?
Trong quá trình tách này, tại sao hỗn hợp được đun nóng trong hỗn hợp
acid và cái gì được lọc? Làm thế nào mà quá trình tách alanine tự do bằng
cách hòa tan trong ethanol diễn ra được?

Ghi chú: pig kidney = thận lợn; filter hot = lọc nóng; cool and filter = để nguội
và lọc; solid material = vật chất rắn; heat = đun nóng; filter = lọc; in solution
= trong dung dịch; crystallizes = kết tinh
Nếu quá trình acyl hóa diễn ra thiếu cẩn thận, đặc biệt nếu đun nóng quá
lâu hoặc quá mạnh thì tạo thành một sản phẩm phụ không bị thủy phân
bởi enzyme. Quá trình này diễn ra như thế nào?

(overheating = đun quá nhiệt)


Hướng dẫn
Phản ứng acyl hóa diễn ra theo cơ chế thông thường để tạo thành amide
từ anhydride, đó là tác kích nucleophile trên nhóm carbonyl và tách anion
bền nhất (acetate) từ trung gian tứ diện. Hai đồng phân alanine là đối
quang và các đối quang cần phải phản ứng với tốc độ như nhau với các
tác nhân phi thủ tính.
198 | Bản quyền thuộc về Tạp chí Olympiad Hóa học KEM
Trong phản ứng xúc tác enzyme, acylase thủy phân amide của một đối
quang nhưng với đối quang còn lại thì không. Lúc này, hai đối quang không
phản ứng với tốc độ như nhau khi tác nhân (hoặc xúc tác) là đối quang
tinh khiết của peptide lớn. Không có gì bất ngờ, enzyme phân cắt amide
của alanine tự nhiên và để lại đối quang còn lại:

Quá trình tinh chế và lọc trước tiên đòi hỏi việc loại bỏ enzyme. Nó tan ở
hệ đệm pH 8 nhưng sự acid hóa và đun nóng làm biến tính enzyme (khá
giống những gì xảy ra với lòng trắng trứng khi đun nóng) và phá hủy cấu
trúc của nó. Chất rắn được lọc tách là enzyme đã biến tính này. Quá trình
tách với ethanol diễn ra bởi các amino acid phân cực chỉ tan trong nước
trong khi đó amine lại tan trong ethanol.

Sự đun nóng quá mức dung dịch acid gây ra sự vòng hóa của nguyên tử
amide oxygen vào carboxylic acid. Phản ứng diễn ra bởi sự tạo thành vòng
năm cạnh, một “azalactone”.

199 | Bản quyền thuộc về Tạp chí Olympiad Hóa học KEM
Bài 11
1) Xác định các alkene ban đầu cần thiết để tổng hợp các amino acid
sau, sử dụng phương pháp hydrogen hóa xúc tác bất đối:
(a) L-Alanine
(b) L-Valine
(c) L-Leucine
(d) L-Tyrosine
Ví dụ về tổng hợp L-phenylalanine theo phương pháp này:

2) Giải thích tại sao việc sử dụng xúc tác bất đối không phù hợp cho tổng
hợp glycine?
3) Xác định sản phẩm của các phản ứng sau:

200 | Bản quyền thuộc về Tạp chí Olympiad Hóa học KEM
Hướng dẫn
1)
a) b)

c) d)

2) Glycin không có tâm thủ tính, do đó việc sử dụng xúc tác bất đối là
không cần thiết. Ngoài ra, không có alkene nào có thể tạo thành glycine
bằng cách hydrogen hóa.
3)

201 | Bản quyền thuộc về Tạp chí Olympiad Hóa học KEM
Bài 12
Sử dụng công thức chiếu Fischer hãy hoàn thành chuỗi phản ứng sau :

Hướng dẫn

202 | Bản quyền thuộc về Tạp chí Olympiad Hóa học KEM
Bài 13
Alanin được tổng hợp theo sơ đồ sau đây. Hãy xác định cấu trúc các chất
chưa biết:

Hướng dẫn

203 | Bản quyền thuộc về Tạp chí Olympiad Hóa học KEM
Peptide
Bài 1
1) Vẽ cấu trúc các peptide sau:
(a) Leu-Ala-Gly
(b) Cys-Asp-Ala-Gly
(c) Met-Lys-His-Tyr-Ser-Phe-Val
2) Vẽ cấu dạng s-trans của dipeptide Phe-Leu và chỉ rõ đầu N, đầu C.
3) Vẽ cấu dạng s-cis của dipeptide Phe-Phe và chỉ rõ nguồn gốc tương
tác không gian lớn liên quan đến cấu dạng này.
4) Biểu diễn cấu trúc không gian của tetrapeptide tạo thành khi nối hai
phân tử Cys-Phe bởi một cầu disulfide.
5) Biểu diễn cấu trúc peptide tương ứng với trình tự các gốc amino acid
sau và chỉ rõ các đầu N, C: Trp-Val-Ser-Met-Gly-Glu
6) Methionine enkephalin là một pentapeptide được tạo thành bởi cơ thể
để chế ngự cơn đau. Từ trình tự các gốc amino acid, vẽ cấu trúc
methionine enkephalin.

7) Từ trình tự chuỗi amino acid, hãy dự đoán dạng nào của aspartame
tồn tại chủ yếu ở pH sinh lí:

204 | Bản quyền thuộc về Tạp chí Olympiad Hóa học KEM
Hướng dẫn
1)

2)

3) Trong cấu dạng s-cis, các nhóm phenyl sẽ chịu tương tác không gian
mạnh, do đó khiến cho cấu dạng s-cis có năng lượng cực kì cao.

205 | Bản quyền thuộc về Tạp chí Olympiad Hóa học KEM
4)

5)

6)

7) Ở pH sinh lí học, nhóm carboxylic acid bị deproton hóa (và sẽ tồn tại
chủ yếu ở dạng carboxylate ion), trong khi đó nhóm amino bị proton hóa
(và sẽ tồn tại chủ yếu ở dạng ammonium ion):

206 | Bản quyền thuộc về Tạp chí Olympiad Hóa học KEM
Bài 2
1) Cystine (C6H12N2O4S2) là một diamino-dicarboxylic acid và là dimer
của L-cysteine. Dimer này có thể bị phân cắt bằng cách xử lí với một
thiol như mercaptoethanol (HOCH2CH2SH), tạo thành L-cysteine
(C3H7NO2S).
a) Xác định cấu trúc cystine. Chỉ rõ cấu hình tuyệt đối.
b) Vai trò của mercaptoethanol trong phản ứng này là gì?
Cysteine (1 mol) cũng có thể bị phân cắt khi xử lí với performic acid
(HCOO2H), tạo thành cysteic acid, C3H7NO5S (2 mol) - một acid mạnh.
c) Xác định cấu trúc của cysteic acid ở điểm đẳng điện.
d) Khi xử lí một peptide - chứa 2 chuỗi, A và B, liên kết với nhau bởi
một liên kết đơn disulfide giữa hai gốc cysteine trong mỗi chuỗi -
với perfomic acid thì thu được hai peptide biến tính ở pH 7.0, A’ và
B’, có điện tích toàn phần lần lượt là +5 và -3. Tính điện tích toàn
phần của peptide ban đầu ở pH như trên.
2) Khi peptide C (khối lượng phân tử 464.5) bị thủy phân hoàn toàn trong
dung dịch HCl thì thu được các lượng đẳng mol của glycine (Gly),
phenylalanine (Phe), aspartic acid (Asp), glutamic acid (Glu) và 1
đương lượng ammonia (NH3). Khi xử lí C với enzyme carboxypeptidase
thì thu được glutamic acid và một tripeptide. Thủy phân acid không
hoàn toàn tripeptide này thu được một hỗn hợp sản phẩm, hai trong
số đó được xác định là glycylaspartic acid (Gly-Asp) và
aspartylphenylalanine (Asp-Phe).
a) Từ thông tin ở trên, hãy suy ra trật tự chuỗi hoàn chỉnh của peptide
C.
b) So sánh điểm đẳng điện gần đúng của peptide C với pH 7.0. ươ

207 | Bản quyền thuộc về Tạp chí Olympiad Hóa học KEM
Hướng dẫn
1) a)

b) Chất khử.
c)

d) +4.
2) a) Gly-Asp-Phe-Glu
b) pI < 7.0

208 | Bản quyền thuộc về Tạp chí Olympiad Hóa học KEM
Bài 3
1) Xác định các gốc amino acid tạo nên peptide dưới đây:

2) Bacitracin A được tạo thành bởi vi khuẩn và do đó có chứa một số gốc


không thuộc về nhóm 20 amino acid tồn tại tự nhiên.
a) Xác định các gốc amino acid trong peptide này.
b) Xác định các amino acid nào có cấu hình D thay vì L.
c) Xác định tất cả các gốc không có trong nhóm 20 amino acid tồn
tại tự nhiên.

3) Các nhà nghiên cứu tin rằng các kháng sinh penicillin được tổng hợp
từ các tiền chất amino acid. Xác định hai amino acid nhiều khả năng
đã tham gia vào tổng hợp các kháng sinh penicillin:

209 | Bản quyền thuộc về Tạp chí Olympiad Hóa học KEM
4) Protein huỳnh quang xanh lục (GPP), được cô lập lần đầu tiên từ sứa
phát quang sinh học, là một protein chứa 238 gốc amino acid. Sự
khám phá GFP đã cách mạnh hóa lĩnh vực hiển vi huỳnh quang, cho
phép các nhà hóa sinh theo dõi quá trình sinh tổng hợp các protein.
Giải Nobel Hóa học năm 2008 được rao cho Martin Chalfie, Osamu
Shimomura, và Roger Tsien vì đã khám phá và phát triển GFP. Khối
con cấu trúc (subunit) của GFP chịu trách nhiệm cho sự phát huỳnh
quang, gọi là fluorophore - được tạo thành khi ba gốc amino acid trải
qua sự vòng hóa. Xác định ba amino acid cần cho quá trình sinh tổng
hợp fluorophore.

210 | Bản quyền thuộc về Tạp chí Olympiad Hóa học KEM
Hướng dẫn
1)

2)

3)

211 | Bản quyền thuộc về Tạp chí Olympiad Hóa học KEM
4)

212 | Bản quyền thuộc về Tạp chí Olympiad Hóa học KEM
Bài 4
Sự racemic hóa các α-amino acid và peptide có thể xảy ra bởi cơ chế α-
enol hóa và tác động đun nóng hay có mặt base mạnh đều thúc đẩy cho
quá trình này:

1) Vẽ công thức hóa lập thể I và II (với các liên kết đậm và kẻ vạch) cho
các hợp phần amino acid của hỗn hợp đạt tới cân bằng qua cơ chế α-
enol hóa được mô tả ở trên khi tiến hành với mỗi hydroxyamino acidA
và B sau:

2) Xác định mối liên hệ giữa các cấu trúc mà bạn vẽ trong mỗi trường
hợp A, B ở trên.
AI, II □ đối quang □ đồng phân dia
BI ,II □ đối quang □ đồng phân dia
Trong tổng hợp peptide, để tạo thành liên kết peptide mới, nhóm carboxyl
cần phải được hoạt hóa - nó cần phải mang một nhóm rời đi tốt - được
biểu diễn như trong sơ đồ đơn giản hóa sau:

213 | Bản quyền thuộc về Tạp chí Olympiad Hóa học KEM
Chính ở giai đoạn này của tổng hợp, cơ chế racemic hóa thứ hai có thể
diễn ra. Nguyên tử oxygen của nhóm amidic carbonyl nằm cách xa 5
nguyên tử so với nhóm carboxyl hoạt hóa và có thể tấn công nội phân tử
vào nhóm hoạt hóa này, tạo thành trung gian vòng 5 cạnh (một
azalactone), chất này nhanh chóng đạt tới cân bằng diễn ra tại tâm lập thể
như biểu diễn trong sơ đồ đơn giản hóa dưới đây:

3) Xác định công thức cấu tạo của trung gian C trong chuyển hóa qua lại
giữa hai azalactone và giải thích sự xáo trộn ở tâm lập thể.
Các azalactone là những hợp chất rất hoạt động, có thể phản ứng với
nhóm amino của một amino acid. Do đó, phản ứng ghép cặp có thể diễn
tới hoàn toàn dẫu cho có các sản phẩm racemic hóa hoặc epimer hóa.
4) Nếu N-benzoyl glycine, C9H9NO3, được đun tới 40 oC với acetic
anhydride thì nó có thể chuyển thành hợp chất hoạt động mạnh,
C9H7NO2 (P1).
a) Đề xuất cấu trúc của P1.
b) Viết các sản phẩm tạo thành từ phản ứng của P1 với S-alanine ethyl
ester (P2) (mạch nhánh R của amino acid alinine là nhóm methyl)
sử dụng các công thức hóa lập thể (với các liên kết đậm và kẻ
vạch) cho cả chất phản ứng và sản phẩm.

214 | Bản quyền thuộc về Tạp chí Olympiad Hóa học KEM
Hướng dẫn
1)

2) AI, II là đối quang; BI, II là đồng phân dia.


3) Trung gian C

4)

215 | Bản quyền thuộc về Tạp chí Olympiad Hóa học KEM
216 | Bản quyền thuộc về Tạp chí Olympiad Hóa học KEM
Bài 5
Protein hiện diện trong tất các các tế bào sống và thực hiện vô số các
chức năng khác nhau trong hóa học của sự sống. Chúng được tạo bởi các
α-aminocarboxylic acid. Peptide là các protein “nhỏ” với lượng tương đối
ít các amino acid. Liên kết peptide là liên kết amide tạo bởi tương tác của
nhóm amine trong amino acid. với nhóm carboxylic acid của phân tử kế
cận.
1) Có những peptide nào được hình thành từ phenylalanine F và alanine
A? Viết cấu trúc của chúng.

Trong phân tích cấu trúc của peptide các gốc ở đầu N và đầu C có vai trò
quan trọng.
Phương pháp Sanger để xác định gốc ở đầu N (là gốc amino acid trong
peptide với nhóm NH2 tự do) bao gồm xử lí với 2,4-dinitrofluorobenzene
trong môi trường kiềm, tiếp theo là sự thủy phân hoàn toàn các liên kết
peptide trong môi trường acid. Amino acid đầu N khi ấy có màu vàng có
thể xác định dễ dàng trong phương pháp phân tích sác kí trên giấy. Sanger
nhận giải Nobel năm 1958 và 1980.
2) Phản ứng nào diễn ra với thuốc thử Sanger (để đơn giản, có thể viết
phía đầu N của peptide là H2NR). Viết phương trình phản ứng.
Gốc ở đầu C, có chứa nhóm COOH trong peptide, được xác định bằng sự
thủy phân chọn lọc xúc tác enzyme gốc amino acid đầu C bằng
carboxypeptidase (có trong tuyến tụy). Với một tetrapeptide tạo bởi các
amino acid F, A, Glyxine G và Leucine L, gốc ở đầu C được xác định là F.
Phương pháp Sanger xác định gốc ở đầu N là G.
3) Suy ra các cấu trúc có thể có của tetrapeptide này. Viết các cấu trúc
tương ứng.

217 | Bản quyền thuộc về Tạp chí Olympiad Hóa học KEM
Hướng dẫn
1) Công thức của các peptide nhận được:
Ph Ph
O O O O
H H H H
N N N N
H2 N OH H 2N OH H 2N OH H 2N OH
O O O O
Ph Ph
FA AF FF AA

2) Phản ứng xảy ra:


NO 2 NO 2

O 2N F + 2H 2NR O 2N NHR + RNH 3+F-

3) Không thể biết được thứ tự hai amino acid giữa là AL hay LA nên peptide
ban đầu có có thể có cấu tạo như sau: GALF hoặc GLAF.

218 | Bản quyền thuộc về Tạp chí Olympiad Hóa học KEM
Bài 6
Giải thích một số vấn đề sau đây
1. Peptit Gly-Ala-Arg-Ala-Glu dễ dàng bị cắt mạch bằng dung dịch nước
chứa enzym trypsin ở pH = 8, nhưng lại hoàn toàn trơ với dung dịch
trypsin trong ure ở cùng pH
2. Một đoạn peptit chứa cystin (có cầu nối disunfua) sau khi xử lý với
HSCH2CH2OH rồi xử lý tiếp với aziridin lại có thể bị cắt mạch bằng
enzym trypsin
3. Oxy hóa L-methionin bằng H2O2 thu được hai sản phẩm
4. Benzamidine clorua có thể làm mất hoạt tính của enzym trypsin (Cho
rằng chất này không làm biến tính peptit ở điều kiện đang xét)
5. Axit Polyglutamic tồn tại ở dạng xoắn ở pH < 3. Tuy nhiên khi tăng dần
pH đến pH = 5 thì xảy ra sự tháo xoắn.
Hướng dẫn
1. Trong môi trường ure ở cùng pH thì đầu guanidin của araginin đã phản
ứng với ure cho sản phẩm biến tính. Do cấu trúc chuỗi peptit thay đổi
nên sự cắt mạch bằng trypsin không xảy ra

2. Phản ứng xảy ra với chuỗi peptit như sau:

Cấu trúc sản phẩm cuối cũng tương tự như cấu trúc của phần mạch nhánh
lysine nên có thể bị cắt mạch bởi trypsin
3. Thu được sunfoxit và có khả năng có cả sunfon.
4. Do benzamidin clorua có nhóm chức guanidin nên có khả năng cạnh
tranh tâm hoạt động với trypsin. Tâm hoạt động của trypsin đã bị
chiếm nên enzym mất đi hoạt tính.

219 | Bản quyền thuộc về Tạp chí Olympiad Hóa học KEM
5. Trong axit polyglutamic vẫn còn một nhóm COOH tự do. Khi tăng pH
thì nhóm COOH này sẽ chuyển thành dạng COO-. Các nhóm này ở gần
nhau trong dạng xoắn sẽ đẩy lẫn nhau gây ra sự tháo xoắn.
Bài 7
Glutathione, kí hiệu GSH, là một peptide nhỏ, hiện diện trong hầu hết mô
của các loài động vật. GSH đáp ứng các chức năng sinh học quan trọng,
như giải độc các hóa chất có ái lực điện tử (electrophilic) và khử các
peroxide (hữu cơ) trong máu. Một hợp chất ái điện tử có phản ứng bất
thuận nghịch với GSH, đặc biệt là trong gan, tạo thành sản phẩm chính
được biến đổi bởi một loạt sinh chuyển hóa, tạo thành một mercapturic
acid - chất này được bài tiết qua nước tiểu. Các chất oxid hóa phản ứng
với GSH tạo thành disulfide GSSG, chất này có thể được hoàn nguyên
enzyme về GSH bởi các men khử. Tỉ lệ GSH/GSSG trong hầu hết các tế
bào là ≥ 500.

1)
a) Có bao nhiêu gốc amino acid trong GSH?
b) Vẽ cấu trúc của các amino acid tương ứng và đánh dấu các tâm
chiral bởi dấu hoa thị.
Mercapturic acid A được phân lập từ nước tiểu của một người đã tiếp xúc
với acrylonitrile (H2C=CH-CN) có công thức phân tử là C8H12N2O3S. Phổ 1H
NMR của A trong (CD3)2SO được cho trong hình 1. Khi sản phẩm này được
xử lí trước với D2O thì các tín hiệu ở δ 12.8 và δ 6.8 không xuất hiện nữa,
còn tín hiệu 3 thì được đơn giản hóa.

220 | Bản quyền thuộc về Tạp chí Olympiad Hóa học KEM
2)
a) Các tín hiệu NMR tương ứng với các proton trong các nhóm sau:
CH, CH2, CH3, OH và NH. Hãy chỉ ra nhóm proton thích hợp với các
tín hiệu 1-7.

b) Có bao nhiêu nguyên tử carbon trong hợp chất A mà không liên kết
với bất kì proton nào?
c) Vẽ cấu trúc hợp chất A.
Vitamin C (ascorbic acid) phản ứng với các chất oxid hóa tạo thành
dehydroascorbic acid D

221 | Bản quyền thuộc về Tạp chí Olympiad Hóa học KEM
3) Việc ăn rau và hoa quả tươi rất tốt cho sức khỏe, bởi vì
□ Vitamin C tạo thành phức chất với GSH.
□ Vitamin C phản ứng với các hợp chất ái điện tử.
□ Vitamin C loại bỏ các chất oxid hóa và ngăn sự suy giảm không
mong muốn của GSH.
□ Nhiều lý do, nhưng không liên quan đến GSH.
Hướng dẫn
1) a) 3 gốc amino acid.
b)

2) a)

b) 3
c)

3) Vitamin C loại bỏ các chất oxid hóa và ngăn sự suy giảm không mong
muốn của GSH.

222 | Bản quyền thuộc về Tạp chí Olympiad Hóa học KEM
Bài 8
1) Thủy phân không hoàn toàn octapeptide angiotensin II tạo thành
produces Pro-Phe, Val-Tyr-Ile, Asp-Arg-Val, và Ile-His-Pro. Xác định
trình tự peptide.
2) Xử lí somatostatin với tác nhân Edman tạo thành một dẫn xuất của
alanine. Thủy phân không hoàn toàn polypeptide này tạo thành các
oligopeptide sau đây. Xác định cấu trúc polypeptide.
I: Phe-Trp II: Lys-Thr III: Thr-Ser-Cys
IV: Thr-Phe-Thr-Ser-Cys V: Asn-Phe-Phe-Trp-Lys
VI: Ala-Gly-Cys-Lys-Asn-Phe
3) Thành phần amino acid của một peptide là (Arg2, Gly, Phe2, Pro3, Ser).
Xử lí peptide này với tác nhân Edman tạo thành dẫn xuất PTC của
arginine. Thủy phân không hoàn toàn tạo thành nhiều mảnh peptide
gồm các oligopeptide sau. Xác định trình tự amino acid của peptide
này.
I: Gly-Phe-Ser II: Arg-Pro-Pro-Gly III: Phe-Arg-Ser-Pro-Phe
4) Các giai đoạn cuối trong thoái phân Edman dẫn đến sự chuyển vị
thiazolinone thành mộ N-phenylthiohydantoin. Trình bày cơ chế từng
bước cho phản ứng xúc tác acid này.

223 | Bản quyền thuộc về Tạp chí Olympiad Hóa học KEM
Hướng dẫn
1)

2)

3) Xử lí với tác nhân Edman giúp xác định arginine là amino acid đầu N.

224 | Bản quyền thuộc về Tạp chí Olympiad Hóa học KEM
4)
c

225 | Bản quyền thuộc về Tạp chí Olympiad Hóa học KEM
Bài 9
1) Pentapeptide met-enkephalin bị thủy phân bởi chymotripsin tạo thành
Met, Tyr, và Gly-Gly-Gly. Thông tin này đã đủ giúp thiết lập trình tự các
amino acid trong met-enkephalin chưa?
2) Tetrapeptide tuftsin bị thủy phân bởi trypsin tạo thành Pro-Arg và Thr-
Lys. Thông tin này đã đủ giúp thiết lập trình tự các amino acid trong
tuftsin chưa?
Hướng dẫn
1) Chưa đủ. Chymotrypsin phân cắt các peptide ở đầu C của amoni acid
thơm phenylalanine, tyrosine và trytophan. Sản phẩm thủy phân có hai
amino acid thơm và chúng có thể dẫn đến một trong hai cấu trúc sau. Các
tâm bị phân cắt được chỉ rõ bằng cách in đậm.
Tyr-Gly-Gly-Phe-Met
Gly-Gly-Phe-Tyr-Met
2) Chưa đủ. Trypsin phân cắt các peptide ở đầu C của các amino acid có
tính base lysine và arginine. Cả hai dipeptide đều có các amino acid có
tính base đầu C. Tetrapeptide có thể là một trong hai cấu trúc sau. Các
tâm bị phân cắt được chỉ rõ bằng cách in đậm.
Pro-Arg-Thr-Lys
Thr-Lys-Pro-Arg

226 | Bản quyền thuộc về Tạp chí Olympiad Hóa học KEM
Bài 10
1) Xác định các amino acid và các mảnh peptide tạo thành khi xử lí
decapeptide A-P-F-L-K-W-S-G-R-G với mỗi tác nhân hoặc enzyme sau:
(a) chymotrypsin; (b) trypsin; (c) carboxypeptidase; (d) C6H5N=C=S.
2) Angiotensin là một octapeptide làm thu hẹp các mạch máu, do đó làm
tăng huyết áp. Các chất ức chế ACE là một nhóm thuốc được sử dụng
để điều trị cao huyết áp bằng cách ngăn quá trình tổng hợp
angiotensin trong cơ thể. Angiotensin chứa các amino acid Arg (2
đương lượng), His, Ile, Phe, Pro, Tyr, và Val. Xác định trình tự
angiotensin biết các mảnh sau được tạo thành bởi quá trình thủy phân
không hoàn toàn với acid: Ile-His-Pro-Phe, Arg-Arg-Val, Tyr-Ile-His, and
Val-Tyr
3) Sử dụng các dữ kiện thực nghiệm dưới đây để suy ra trình tự
octapeptide chứa các amino acid sau: Ala, Gly (2 đương lượng), His (2
đương lượng), Ile, Leu, và Phe. Thoái phân Edman phân cắt Gly từ
octapeptide, còn carboxypeptidase tạo thành Leu và một
heptapeptide. Thủy phân không hoàn toàn thu được các mảnh sau: Ile-
His-Leu, Gly, Gly-Ala-Phe-His, và Phe-His-Ile.
4) Một octapeptide chứa các amino acid sau: : Arg, Glu, His, Ile, Leu, Phe,
Tyr, và Val. Xử lí octapeptide trên với carboxypeptidase tạo thành Phe
và một heptapeptide. Xử lí octapeptide với chymotrypsin tạo thành hai
tetrapeptide, A và B. Xử lí A với trypsin tạo thành hai dipeptide, C và D.
Thoái phân Edman phân cắt các amno acid từ mỗi peptide: : Glu
(octapeptide), Glu (A), Ile (B), Glu (C), và Val (D). Thủy phân không hoàn
toàn tetrapeptide B tạo thành Ile-Leu cùng với các sản phẩm khác. Suy
ra cấu trúc của octapeptide và các mảnh A-D.
Hướng dẫn
1)

2)

227 | Bản quyền thuộc về Tạp chí Olympiad Hóa học KEM
3)

4)

228 | Bản quyền thuộc về Tạp chí Olympiad Hóa học KEM
Bài 11
Một trong số những neuropeptit (những peptit ảnh hưởng trực tiếp đến hệ
thần kinh) chính là β-neoendorphin phân lập từ não lợn. Kết quả phân tích
sơ bộ cho thấy nó gồm 9 aminoaxit liên kết trực tiếp với nhau. Để xác định
trình tự sắp xếp các aminoaxit này thì đầu tiên người ta tiến hành cắt mạch
bằng enzym dipeptidiaminopeptidaza (cắt mạch từ aminoaxit thứ hai tính
từ đầu N). Tiếp theo cắt mạch nonapeptit này bằng enzym
dipeptidicacboxipeptidaza (cắt mạch từ aminoaxit thứ 2 tính từ đuôi C)
thu được kết quả như sau:
- Dipeptidiaminopeptidaza: Gly-Phe; Leu-Arg ; Tyr-Gly; Lys-Tyr; Pro
- Dipeptidicacboxipeptidaza: Phe-Leu; Gly-Gly; Arg-Lys; Tyr-Pro; Tyr
1. Xác định trật tự sắp xếp các aminoaxit trong β-neoendorphin
2. Trong dãy các dipeptit trên thì dipeptit nào
a) Không quang hoạt?
b) Có điểm đẳng điện > 7
c) Khó bị thủy phân để tạo aminoaxit tự do nhất
Giải thích
Hướng dẫn
1. Tyr – Gly – Gly – Phe – Leu – Arg – Lys – Tyr – Pro
2. a) Gly – Gly. Vì Gly là aminoaxit duy nhất không có tính quang hoạt
b) Leu – Arg; Lys – Tyr; Arg – Lys. Vì tất cả chúng đều còn một
nhóm NH2 tự do
c) Tyr – Pro. Do liên kết amit trong dipeptit này được tạo thành từ
amin bậc 2 nên khó bị thủy phân hơn nhiều.

229 | Bản quyền thuộc về Tạp chí Olympiad Hóa học KEM
Bài 12
Thủy phân toàn phần một hexapeptit X có hoạt tính sinh học thu được hỗn
hợp 3 aminoaxit Arg, Trp và Phe với tỉ lệ tương ứng 3:2:1. Tiến hành thủy
phân X bằng trypsin rồi xác định bằng DNFB thu được DNB-Trp, Phe, Trp,
DNB-Arg và Arg với tỉ lệ tương ứng 1:1:1:2:1. Còn nếu tiến hành tương tự
với chymotrypsin thu được DNB-Arg, Arg, DNB-Phe và Trp với tỉ lệ tương
ứng 1:2:1:2.
Vẽ công thức cấu tạo của X nếu biết rằng MX = 987 g/mol
Hướng dẫn
Tổng 3Arg + 2 Trp + 1Phe = 1095, chênh 108 đơn vị so với giá trị MX thực
tế, ứng với 6 phân tử nước. Hay nói cách khác một hexapeptit tạo được 6
liên kết peptit chứng tỏ nó có cấu trúc vòng.
Dựa trên cấu trúc sản phẩm thủy phân từng phần có thể kết luận
hexapeptit X có cấu trúc: xiclo – (Trp – Arg – Arg – Arg – Trp – Phe).
X không phản ứng với DNFB nên X có công thức cấu tạo như sau:

230 | Bản quyền thuộc về Tạp chí Olympiad Hóa học KEM
Bài 13
1) Bradykinin có trình tự như sau:
(đầu N) Arg-Pro-Pro-Gly-Phe-Ser-Pro-Phe-Arg (đầu C)
Xác định tất cả các mảnh sẽ được tạo thành khi xử lí bradykinin với:
a) Trypsin
b) Chymotrypsin
2) Xác định gốc đầu N của peptide tạo thành dẫn xuất PTH
(phenylthiohydantoin) khi thoái phân Edman:

3) Xử lí một tripeptide với phenyl isothiocyanate tạo thành hợp chất A và


một dipeptide. Xử lí dipeptide với phenyl isothiocyanate tạo thành hợp
chất B và glycine. Xác định cấu trúc tripeptide ban đầu.

Hướng dẫn
1) (a) Trypsin xúc tác cho sự thủy phân liên kết peptide ở phía carboxyl
của arginine, tạo thành hai mảnh sau:
Arg + Pro-Pro-Gly-Phe-Ser-Pro-Phe-Arg
(b) Chymotrypsin xúc tác cho sự thủy phân các liên kết peptide ở phía
carboxyl của phenylalanine, tạo thành ba mảnh sau:
Arg-Pro-Pro-Gly-Phe + Ser-Pro-Phe + Arg

231 | Bản quyền thuộc về Tạp chí Olympiad Hóa học KEM
2) Nhóm R (bôi màu) trong dẫn xuất PTH giúp xác định gốc đầu N. Do gốc
R là nhóm benzylic (-CH2Ph) nên đó phải là phenylalanine.

3)

232 | Bản quyền thuộc về Tạp chí Olympiad Hóa học KEM
Bài 14
1) Đề xuất hai cấu trúc có thể có của một tripeptide chứa glycine, L-
alanine, and L-phenylalanine nhưng không phản ứng với phenyl
isothiocyanate.
2) Vẽ cấu trúc dẫn xuất PTH (phenylthiohydantoin) đầu tiên được tạo
thành khi tripeptide Ala-Phe-Val trải qua thoái phân Edman.
3) Một peptide với 22 gốc amino acid được xử lí với trypsin tạo thành bốn
mảnh (fragment), trong khi đó xử lí với chymotrypsin thì tạo thành sáu
mảnh. Xác định chuỗi 22 gốc amino acid trong peptide ban đầu.

4) Xét cấu trúc octapeptide vòng sau. Phân cắt peptide này bởi trypsin
có tạo thành các mảnh khác với phân cắt bởi chymotrypsin không?
Giải thích.

5) Glucagon là một peptide hormone được tạo thành bởi tuyến tụy mà nó
cùng với insulin giúp điều hòa mức độ đường huyết. Glucagon được
tạo thành bởi 29 gốc amino acid. Xử lí với trypsin tạo thành bốn mảnh,
trong khi đó xử lí với chymotrypsin tạo thành sáu mảnh. Xác định trình
tự của các gốc amino acid trong glucagon và chỉ rõ bất kì cầu disulfide
nào nếu có.
Các mảnh trypsin:
His-Ser-Gln-Gly-Thr-Phe-Thr-Ser-Asp-Tyr-Ser-Lys
Ala-Gln-Asp-Phe-Val-Gln-Trp-Leu-Met-Asn-Thr
Tyr-Leu-Asp-Ser-Arg
Arg

233 | Bản quyền thuộc về Tạp chí Olympiad Hóa học KEM
Các mảnh chymotrypsin:
His-Ser-Gln-Gly-Thr-Phe
Thr-Ser-Asp-Tyr
Leu-Met-Asn-Thr
Ser-Lys-Tyr
Leu-Asp-Ser-Arg-Arg-Ala-Gln-Asp-Phe
Val-Gln-Trp
Hướng dẫn
1) Nếu một tripeptide không phản ứng với phenyl isothiocyanate thì nó
phải có đầu N tự do. Nó phải là một tripeptide vòng. Dưới đây là hai
cấu trúc có thể có của tripeptide vòng này:

2) Thoái phân Edman sẽ loại bỏ gốc amino acid đầu N và trong tripeptide
Ala-Phe-Val thì đó là Ala. Do đó, aline bị loại bỏ, tạo thành dẫn xuất
PTH sau:

3) Chỉ có một trong các mảnh trypsin có đầu C không phải là arginine
hoặc lysine. Mảnh này, kết thúc với valine, phải là mảnh cuối cùng
trong chuỗi peptide. Ba mảnh trypsin còn lại có thể được đặt vào vị trí
phù hợp bằng cách phân tích các mảnh chymotrypsin. Trình tự peptide
đúng là:
Ala-Val-Met-Phe-Val-Ala-Tyr-Lys-Pro-Val-Ile-Leu-Arg-Trp-His-Phe-Met-
Cys-Arg-Gly-Pro-Phe-Ala-Val

234 | Bản quyền thuộc về Tạp chí Olympiad Hóa học KEM
4) Phân cắt với trypsin sẽ tạo thành Phe-Arg, trong khi phân cắt với
chymotrypsin sẽ tạo thành Arg-Phe. Hai peptide này không giống nhau,
chúng là đồng phân cấu tạo của nhau.
5) His-Ser-Gln-Gly-Thr-Phe-Thr-Ser-Asp-Tyr-Ser-LysTyr-Leu-Asp-Ser-Arg-
Arg-Ala-Gln-Asp-Phe-Val-GlnTrp-Leu-Met-Asn-Thr
(không có cầu disulfide)

235 | Bản quyền thuộc về Tạp chí Olympiad Hóa học KEM
Bài 15
Quy trình xác lập trật tự các aminoaxit trong một heptapeptit Y
(Met2 , Asp, Lys, Arg, Phe, Gly) có hoạt tính sinh học được tiến hành như
sau:
Phản ứng với DNFB cho DNP – Met.
Cắt mạch bằng cacboxipeptidaza cũng thu được Met.
Cắt mạch heptapeptit bằng BrCN giải phóng 1 mol homoserin
lacton
Cắt mạch bằng chymotrypsin thu được một pentapeptit và một
dipeptit. Khi cho pentapeptit phản ứng với 5-(dimetylamino)naphtalene-1-
sunfonyl clorua rồi cắt mạch thu được dẫn xuất của methionin.
Cắt mạch bằng trypsin thu được hai tripeptit có chứa Met và Arg
tự do.
Cắt mạch bằng pepsin cho tetrapeptit (Met, Arg, Lys và Asp) và
tripeptit (Met, Phe và Gly).
Từ những dữ kiện đã cho xác định trật tự sắp xếp các aminoaxit
trong peptit.
Hướng dẫn
Aminoaxit đầu N lẫn đầu C đều là Met.
BrCN đóng vai trò cắt mạch peptit sau Met, và biến đoạn Met bị
cắt ra thành homoserin lacton theo cơ chế:

Chymotrypsin cắt mạch sau aminoaxit thơm nên cấu trúc


pentapeptit phải là (Met - ? -? - ? – Phe).

236 | Bản quyền thuộc về Tạp chí Olympiad Hóa học KEM
Việc cắt bằng trypsin cho Arg tự do và hai tripeptit có chứa Met
chứng tỏ Arg phải là aminoaxit thứ 4.
Dựa trên dữ liệu cắt mạch bằng pepsin có thể suy ra được peptit
phải có trật tự sắp xếp như sau: Met – Asp – Lys – Arg – Phe – Gly – Met.

237 | Bản quyền thuộc về Tạp chí Olympiad Hóa học KEM
Bài 16
1) Feline gastrin, một loại hormone kích thích tiết dịch dạ dày ở mèo, có
thành phần amino acid là Ala2, Asp, Gly2, Glu5, Leu, Met, Phe, Pro, Trp2,
Tyr. Kết quả phân tích nhóm đầu cho thấy các amino acid đầu C và N
lần lượt là Phe và Glu. Thủy phân với chymotrypsin tạo thành bốn
peptide sau. Viết hai trình tự amino acid có thể có của feline gastrin.
I: Gly-Trp
II: Met-Asp-Phe
III: Glu-Gly-Pro-Trp
IV: Leu-Glu-Glu-Glu-Glu-Ala-Ala-Tyr
2) Corticotropin, một hormone tuyến yên, kích thích vỏ thượng thạn. Khi
thủy phân chymotrypsin tạo thành sáu peptide sau:

Thủy phân không hoàn toàn bởi trypsine tạo thành lysine, arginine và năm
peptide:

Xác định trình tự amino acid của corticotropin.


Hướng dẫn
1)

2) Cả peptide IV và V từ sự thủy phân chymotrypsin đều chứa peptide V


từ sự thỷ phân trypsin. Cả hai peptide II và III từ sự thủy phân trypsin đều
có trong peptide VI từ thủy phân chymotrypsin. Cả hai peptide III và II từ
sự thủy phân chymotrypsin đều có trong peptide IV từ sự thủy phân
trypsin. Do đó, ba peptide sau chứa phần lớn các amino acid:

238 | Bản quyền thuộc về Tạp chí Olympiad Hóa học KEM
Tryptophan chỉ có trong hai mảnh. Do đó, dipeptide I từ sự thủy phân bởi
chymotrypsin và peptide I từ sự thủy phân trypsin có thể được kết hợp
thành một chuỗi amino acid Arg-Trp-Gly-Lys. Một phần của chuỗi này
được nhân đôi theo hai amino acid ở các vị trí đầu N của peptide B. Xếp
chồng phù hợp thu được:
Arg-Trp-Gly-Lys-Pro-Val-Gly-Lys-Lys-Arg-Pro-Lys-Val-Tyr
Amino acid đầu N của mảnh này chung với arginine của peptide C, không
khớp với arginine của peptide B. Peptide tạo thành với sự xếp chồng phù
hợp là:
Ser-Tyr-Ser-Met-Glu-His-Phe-Arg-Trp-Gly-Lys-Pro-Val-Gly-Lys-Lys-Arg-Pro-
Lys-Val-Tyr
Trình tự amino acid của peptide ban đầu là:
Ser-Tyr-Ser-Met-Glu-His-Phe-Arg-Trp-Gly-Lys-Pro-Val-Gly-Lys-Lys-Arg-Pro-
Lys-Val-Tyr-Pro-Asp-Ala-Gly-Glu-Asp-Gln-Ser-Ala-Glu-Ala-Phe-Pro-Leu-Glu-
Phe

239 | Bản quyền thuộc về Tạp chí Olympiad Hóa học KEM
Bài 17
Khi thủy phân hoàn toàn một peptit X thu được: Arg, Asp, Cys2 , Gly,
Ile, Leu, Lys, Met, Phe, Pro, Ser. Kết quả phân tích theo phương pháp
Edman cho Leu và Ser, trong khi đó thủy phân bằng enzym
cacboxipeptidaza cho Asp và Ile.
Xử lý X với HOCH2CH2SH sau đó cho phản ứng với ICH2COOH rồi
phân cắt mạch bằng trypsin thu được ba dipeptit (Arg, Ser), (Asp, Met),
(Cys, Lys) và một Hexapeptit (Cys, Gly, Ile, Leu, Phe, Pro).
Xử lý X với HOCH2CH2SH sau đó cho phản ứng với
BrCH2CH2NH3+Br- rồi phân cắt mạch bằng trypsin thu được hai dipeptit
(Arg, Ser), (Asp, Met), hai tripeptit (Cys, Gly, Leu) và (Ile, Phe, Pro) cùng với
hai aminoaxit tự do là Cys và Lys.
Cắt mạch bằng pepsin thu được nonapeptit (Arg, Asp, Cys2 , Gly,
Leu, Lys, Met, Ser) và một tripeptit (Ile, Phe, Pro).
Từ các thông tin trên hãy xác định trật tự sắp xếp các aminoaxit
trong peptit X.
Hướng dẫn
Việc cắt mạch bằng Edman thu được hai aminoaxit đầu N là Leu và Ser
chứng tỏ chuỗi peptit có khả năng gồm hai chuỗi peptit nhỏ riêng rẽ được
nối lại với nhau bằng liên kết disunfua của hai phân tử Cys. Trong đó một
chuỗi peptit nhỏ có Asp là aminoaxit đầu C, chuỗi còn lại không có nhóm
COOH tự do.
Hợp chất HOCH2CH2SH cắt liên kết disunfua thành hai nhóm SH
riêng rẽ, sau đó xử lý tiếp với ICH2COOH rồi cắt bằng trypsin thu được một
hexapeptit và các dipeptit riêng rẽ. Trypsin cắt sau Arg và Lys nên từ đó
suy ra trật tự ba dipeptit sẽ là Ser – Arg, Cys – Lys và Met – Asp. Như vậy
trật tự sắp xếp của một chuỗi peptit nhỏ sẽ là Ser – Arg – Cys – Lys – Met
– Asp.
Việc xử lý bằng BrCH2CH2NH3+Br- khiến cho mạch alkyl nối với Cys
tích điện dương làm cho peptit trở nên nhạy hơn với trypsin nên sự cắt
mạch xảy ra dữ dội hơn. Lúc này đoạn hexapeptit phía trên đã bị cắt thành
hai tripeptit nhỏ. Do Leu được xác định là aminoaxit đầu N nên một
tripeptit sẽ có thứ tự Leu – Gly – Cys.

240 | Bản quyền thuộc về Tạp chí Olympiad Hóa học KEM
Trypsin cắt mạch peptit ở đầu NH của aminoaxit thơm nên thứ tự
các aminoaxit trong tripeptit còn lại phải là Phe – Pro – Ile.
Như vậy cấu trúc của chuỗi peptit ban đầu sẽ là:

241 | Bản quyền thuộc về Tạp chí Olympiad Hóa học KEM
Bài 18
Peptide A có công thức phân tử 1007. Thủy phân xúc tác acid tạo thành
các amino acid sau với lượng đẳng mol: Asp, Cystine, Glu, Gly, Ile, Leu, Pro,
và Tyr (xem Bảng 1). Oxid hóa A với HCO2OH chỉ tạo thành B, chất này có
2 gốc cysteic acid (Cya là dẫn xuất cysteine với nhóm thiol bị oxid hóa
thành sulfonic acid).
1) Có bao nhiêu nhóm sulfonic acid được tạo thành từ phản ứng oxid hóa
một liên kết disulfide?
Thủy phân không hoàn toàn B tạo thành một số di- và tripeptide (B1-B6).
Trình tự chuỗi của mỗi sản phẩm thủy phân được xác định theo các cách
sau đây. Amino acid đầu N được xác định bằng cách xử lí peptide với 2,4-
dinitrofluorobenzene (DNFB), tạo thành DNP-peptide. Sau khi thủy phân
acid hoàn toàn DNP-peptide thì một DNP-amino acid sẽ được tạo thành,
chất này có thể được xác định nhanh chóng bằng cách so sánh với các
DNP-amino acid chuẩn.
2) B1, khi xử lí với DNFB, sau đó thủy phân acid tạo thành một sản phẩm
là DNP-Asp. Thông tin này gợi ý rằng B1 có một gốc aspartic acid ở
đầu N. Biểu diễn cấu trúc của DNP-Asp tại điểm đẳng điện (không yêu
cầu hóa lập thể).
Tiếp theo, amino acid đầu C được xác định bằng cách đun nóng peptide
ở 100 oC với hydrazine, làm phân cắt toàn bộ các liên kết peptide và
chuyển tất cả, ngoại trừ amino acid đầu C, thành các amino acid
hydrazide, còn nhóm carboxyl đầu C giữ nguyên. Theo cách này, các
amino acid đầu N và C được xác định và chuỗi hoàn chỉnh của B1-B6 được
cho dưới đây:
B1 Asp-Cya B4 Ile-Glu
B2 Cya-Tyr B5 Cya-Pro-Leu
B3 Leu-Gly B6 Tyr-Ile-Glu
Thủy phân B bởi enzyme từ trực khuẩn Bacillus tạo thành B7-B9 với công
thức như sau:
B7 Gly-NH2 (Glycinamide)
B8 Cya, Glu, Ile, Tyr

242 | Bản quyền thuộc về Tạp chí Olympiad Hóa học KEM
B9 Asp, Cya, Leu, Pro
3) Xác định trình tự chuỗi của B8, biết DNP-Cya được tạo thành khi xử lí
B8 với DNFB, sau đó thủy phân acid hoàn toàn.
4) Biết các amino acid đầu N và C của B9 được xác định lần lượt là Asp
và Leu, hãy viết trình tự chuỗi của B9.
5) Xác định cấu trúc hoàn chỉnh của A, sử dụng các kí hiệu viết tắt trong
Bảng 1, chỉ rõ vị trí của liên kết disulfide.
Tuy nhiên, khối lượng mol tính được của A dựa vào chuỗi ở trên lại lớn hơn
2 đơn vị so với giá trị thực nghiệm. Khi quan sát cẩn thận hỗn hợp từ quá
trình thủy phân acid hoàn toàn A thì thấy 3 đương lượng mol ammonia
cũng được tạo thành ngoài các amino acid được xác định ban đầu.
6) Hãy đề xuất cấu trúc được sửa đổi của A và khoanh tròn (các) tâm
trong cấu trúc là nguồn tạo ra ammonia.
7) Sử dụng thông tin trong Bảng 2, hãy tính điểm đẳng điện của A.
Bảng 1: Kí hiệu và công thức của một số amino acid phổ biến tại điểm
đẳng điện.

243 | Bản quyền thuộc về Tạp chí Olympiad Hóa học KEM
Bảng 2: pKa của một số nhóm quan trọng trong các amino acid

244 | Bản quyền thuộc về Tạp chí Olympiad Hóa học KEM
Hướng dẫn
1) 2 nhóm sulfonic acid được tạo thành khi oxid hóa một liên kết
disulfide.
2) Cấu trúc hoàn chỉnh ở DNP-Asp ở điểm đẳng điện là

3) Trình tự chuỗi B8 là: Cya-Tyr-Ile-Glu


4) Trình tự chuỗi B9 là: Asp-Cya-Pro-Leu
5) Cấu trúc hoàn chỉnh của A là:

6) Cấu trúc hoàn chỉnh của A đã hiệu chỉnh là:

7) Điểm đẳng điện của A là pI = 9.

245 | Bản quyền thuộc về Tạp chí Olympiad Hóa học KEM
Bài 19
Depsipeptit là những peptit mà một hoặc nhiều liên kết amit (CO-NH) được
thay thế bằng liên kết este (CO-OR). Do có cấu tạo khác lạ nên chúng có
hoạt tính sinh học lý thú. Một số depsipeptit tách được từ nấm đã được
dùng để chữa trị ung thư và HIV.
Khi cho depsipeptit P (C24H36N4O6S2) phản ứng với 2-mercaptoetanol
(HOCH2CH2SH) dư, thu được (HOCH2CH2S)2 và Q (C24H38N4O6S2). Q phản
ứng với MeI trong dung dịch K2CO3 tạo thành R. Cho R phản ứng với dung
dịch nước của imidazole (1,3-diazole, C3H4N2) thu được T (C26H44N4O7S2).
Thủy phân T bằng enzym cacboxipeptidaza lần lượt thu được L-valin (L–
H2NCH(CHMe2)COOH), cis – dehydrobutyrin (axit (E)-2-aminobut-2-
enoic), L-metylcystein (L – H2NCH(CH2SMe)COOH) và U. Thủy phân U
bằng axit thì tạo ra D-valin và hợp chất V. Ozon hóa V rồi chế hóa với
Zn/HCl thu được axit 3-hydroxi-4-oxobutanoic, 3-(metylsunfinyl)propanal
(MeS(O)CH2CH2CHO) và 3-(metylsunfonyl)propanal (MeS(O2)CH2CH2
CHO).
Hãy xác định cấu tạo của Q, R, T, U, V và depsipeptit P.
Hướng dẫn
Hãy xác định cấu tạo của Q, R, T, U, V và depsipeptit P
Các dữ kiện đầu bài được tóm tắt từng bước và phân tích như sau:
• P (C24H36N4O6S2) + HOCH2CH2SH dư → Q (C24H38N4O6S2) +
(HOCH2CH2S)2 (1)
Phản ứng (1) cho thấy cầu disunfua (-S-S-) ở P đã bị khử thành hai
nhóm -SH ở Q (phân tử tăng thêm 2H).

• Q (C24H38N4O6S2) ⎯⎯⎯⎯→
MeI / K 2CO3
(2)
R ⎯⎯⎯⎯⎯
H 2O /Im idazole
(3)
→ T (C26H44N4O7S2)

Phản ứng (2) và (3) cho thấy 2 nhóm -SH ở Q đã chuyển thành 2
nhóm -SMe ở R rồi cộng thêm 1 H2O thành T (phân tử tăng thêm 2C, 6H
và 1O).
• Thủy phân T nhờ enzym cacboxipeptidaza thì lần lượt thu được L-
valin (L– H2NCH(CHMe2)COOH), cis – dehydrobutyrin (axit (E)-2-

246 | Bản quyền thuộc về Tạp chí Olympiad Hóa học KEM
aminobut-2-enoic), L-metylcystein (L – H2NCH(CH2SMe)COOH) và
U.
Dữ kiện này cho thấy thứ tự các hợp phần chính trong T như sau:
U – HNCH(CH2SMe)CO-HNC(=CHMe)CO-HNCH(CHMe2)COOH.
• Thủy phân U nhờ xúc tác axit sinh ra D-valin và hợp chất V. Ozon
hóa V rồi chế hóa với Zn/HCl thu được axit 3-hydroxi-4-
oxobutanoic, 3-(metylsunfinyl)propanal (MeS(O)CH2CH2CHO) và
3-(metylsunfonyl)propanal (MeS(O2)CH2CH2CHO).
Dữ kiện này cho thấy V liên kết với D-valin bởi liên kết amit CO-NH.
Ở T có 26C, tổng số C ở bốn hợp phần tách ra là 18 → Ở V có 8C. Do đó V
có cấu tạo là:
MeS – CH2CH2CH=CHCH(OH)CH2COOH.
Cấu tạo của U như sau:
MeS – CH2CH2CH=CHCH(OH)CH2CO-NHCH(CHMe2)COOH
Suy ra T có cấu tạo như sau:

Viết gọn là:


MeS – CH2CH2CH=CHCH(OH)CH2CO(D-Val)-(metylCys)-(cis-
dehydrobutyrin)-(Val)
• R (C26H42N4O6S2) + H2O/Imidazole → T (C26H44N4O7S2) (3)
So sánh công thức của R và T thì thấy ở phản ứng (3) R đã cộng
thêm một phân tử H2O để tạo ra T. Theo đầu bài ở depsipeptit phải có liên
kết este nên phản ứng (3) là sự thủy phân một este vòng nhờ xúc tác
imidazole (vì không tạo ra ancol và axit riêng rẽ và thành phần tăng thêm
2H và 1O).
Đối với T (C26H44N4O7S2) độ không no ( + v) = 7 phù hợp với 7 liên
kết  của 5 nhóm C=O và 2 nhóm C=C. Đối với R (C26H42N4O6S2) độ không
no ( + v) = 8, phù hợp với 7 liên kết  của 5 nhóm C=O, 2 nhóm C=C và 1
vòng este. Liên kết este ở R chỉ có thể được tạo thành từ nhóm COOH của
L-valin và nhóm OH ancol của axit V.
247 | Bản quyền thuộc về Tạp chí Olympiad Hóa học KEM
Như vậy R có cấu tạo như sau:

Q có cấu tạo như sau:

P có cấu tạo như sau:

248 | Bản quyền thuộc về Tạp chí Olympiad Hóa học KEM
Bài 20
*opioid peptide là các peptide liên kết với các thụ thể opioid trong não.
Các hợp chất A, B, C – những chất tương tự với các peptide giảm đau nội
sinh – đã được tiến hành phân tích. Biết rằng tất cả các hợp chất này đều
là những peptide ngắn (chúng có cùng số gốc amino acid) và được tạo
thành chỉ bởi các amino acid thơm và iminoacid. Trong chuỗi của hai
peptide, amino acid tạo protein đã bị thay thế bởi amino acid phi tự nhiên.
Bằng phổ khối lượng, xác định được rằng khối lượng mol của các peptide
A và B lớn hơn 14 g/mol so với khối lượng mol của peptide C. Tất cả các
peptide này đều dương tính với phép thử ninhydrin.
Kết quả của thủy phân chymotrypsin là nhận được ba mảnh cấu trúc từ
các peptide A và C, và chỉ có hai mảnh từ peptide B. Trong trường hợp
của hai trong số các peptide được phân tích, khối lượng mol của một
mảnh bị phân cắt là 262 g/mol, ngoài ra một trong các peptide này cũng
nhận được một mảnh với khối lượng mol 165 g/mol.
Thoái phân Edman (một chu trình) của mỗi peptide tạo thành dẫn xuất
giống nhau với khối lượng mol 298 g/mol. Các peptide bị rút ngắn (từ thoái
phân Edman) không tạo màu tím trong phép thử ninhydrine.
Amino acid phi tự nhiên có thể được tổng hợp theo chuyển hóa sau:

a) Trình bày sơ đồ tổng quát của thoái phân Edman và vẽ cấu trúc của
dẫn xuất tạo thành.
b) Vẽ cấu trúc các hợp chất X và Z. Sản phẩm của các chuyển hóa này
có tính quang hoạt không?
c) Xác định chuỗi các peptide A, B, C và giải thích ngắn gọn lựa chọn của
bạn (sử dụng các kí hiệu ba-chữ cái, với amino acid phi tự nhiên thì kí
hiệu là U).
d) Mô tả ngắn gọn tác động của amino acid phi tự nhiên trong các chuỗi
peptide đến tính bền của chúng trong các điều kiện sinh lí.

249 | Bản quyền thuộc về Tạp chí Olympiad Hóa học KEM
Hướng dẫn
a) Thoái phân Edman là phương pháp xác định trình tự các amino acid
trong một peptide. Nó cũng cho phép xác định amino acid đầu N. Sơ đồ
phản ứng diễn ra như sau:

Hoặc ngắn gọn hơn là:


R S

NCS peptide H +
+ H2N N NH
- peptide
O
O R

Biết rằng khối lượng mol của dẫn xuất tạo thành là 298 g/mol. Mảnh
isothiocyanate có khối lượng mol là 135 g/mol. Có thể dễ dàng tính được
khối lượng mol của gốc amino acid:
298 -135= 163,
163+18(H2O) = 181 g/mol – tyrosine (Tyr).
Cấu trúc của dẫn xuất tạo thành là:

250 | Bản quyền thuộc về Tạp chí Olympiad Hóa học KEM
S

N NH

HO

b) Sơ đồ tổng hợp amino acid phi tự nhiên:

Hợp chất X là nitrile không no, được tạo thành từ phản ứng Knoevenagel.
Sự khử với NaBH4 khi có mặt CoCl2 của nitrile như vậy tạo thành ester của
amino acid phi tự nhiên (Z). Phản ứng tạo thành hỗn hợp đẳng mol của
các đối quang (racemic), có nghĩa là sản phẩm tổng hợp không có tính
quang hoạt.
c) Biết rằng trong trình tự tất cả các peptide chỉ có các amino acid thơm
và iminoacid. Tất cả các peptide được phân tích đều tạo màu tím với
ninhydrin nhưng các peptide được rút ngắn sau một chu trình thoái phân
Edman thì không. Điều này có nghĩa là gốc thứ hai là iminoacid. Proline
(Pro) là imioacid tự nhiên duy nhất được biết tới. Do đó, đầu N của các
peptide là Tyr-Pro.
Khối lượng mol của các peptide A và B lớn hơn C 14 g/mol - gợi ý về sự
hiện diện của thêm một nhóm CH2 nữa. Amino acid phi tự nhiên là β-hPhe,
vậy nó có thể được tìm thấy trong chuỗi các peptide A và B.
Chymotrypsin là enzyme phân cắt các peptide ở phía carboxyl của các
amino acid thơm. Sau thủy phân, nhận được ba mảnh từ các peptide A và
C và chỉ có haai mảnh từ peptide B. Một trong các mảnh được phân cắt
là tyrosine đầu N. Trong trường hợp hai peptide được phân tích thì mảnh
262 g/mol cũng bị phân cắt. Nó có thể chứa Pro, vậy: 262 – 115(Pro) +
18(H2O) =165 g/mol – khối lượng mol của phenylalanine (Phe). 165 g/mol
cũng là khối lượng mol của mảnh khác được tạo thành từ thủy phân
enzyme của một peptide.
251 | Bản quyền thuộc về Tạp chí Olympiad Hóa học KEM
Như đã đề cập ở trên, C được tạo thành chỉ từ các amino acid thơm tạo
protein và iminoacid. Chuỗi peptide của C là:

Trong thủy phân chymotrypsin của các peptide A và C, nhận được ba


mảnh, sự khác nhau giữa hai peptide này phải là phần đầu C. Do đó chuỗi
peptide A là:

Sự khác nhau giữa các peptide A và B phải là vị trí thứ ba (thủy phân
enzyme). Do đó, chuỗi peptide B là:

d) Các peptide với amino acid phi tự nhiên trong chuỗi thường chống chịu
tối hơn trước tác động thủy phân của enzyme. Liên kết peptide trên phía
carboxylic của amino acid phi tự nhiên không bị thủy phân.

252 | Bản quyền thuộc về Tạp chí Olympiad Hóa học KEM
Bài 21
Việc xác định trình tự của một protein (polypeptide) gồm các giai đoạn
sau: a) tinh chế; b) xác định amino acid đầu N; c) phân cắt chuỗi
polypeptide bằng phương pháp hóa học hoặc enzyme; d) cô lập các mảnh
peptide và (e) xác định trình tự của chúng bằng một máy giải trình tự tự
động (sequenator). Cũng có thể xác định trình tự hỗn hợp các mảnh
peptide mà không cần phân giải. Trình tự chuỗi cuối cùng có thể được xác
định bằng cách xếp chồng các chuỗi sau khi phân tích thông tin về vị trí
của các amino acid trong các mảnh khác nhau.
Một protein nhỏ, tạo nên từ 40 gốc amino acid được xác định trình tự chuỗi
như sau:
- Tiến hành thoái phân Edman bằng cách xử lí với phenyl isothiocyanate,
sau đó thủy phân và xác định bằng quang phổ amino acid biến tính.
Quá trình này xác định được aspartic acid (Asp) là gốc đầu N.
- Protein được xử lí với CNBr (cyanogen bromide) - chất này phân cắt
liên kết peptide giữa methionine và bất kì amino acid nào khác ở đầu
C của nó. Mảnh peptide thu được không được tách riêng. Hỗn hợp các
peptide được phân tích trên máy giải trình tự protein. Do đó, máy giải
này có thể phát hiện nhiều amino acid ở vị trí xác định. Kết quả được
cho ở Bảng 1(a).
- Protein được xử lí với một enzyme trypsin phân giải protein. Enzyme
này phân cắt liên kết peptide giữa amino acid có tính base (Arg hoặc
Lys) và gốc đầu C tiếp theo. Hỗn hợp peptide thu được cũng được tiến
hành phân tích như trên. Kết quả được cho ở Bảng 1(b).

253 | Bản quyền thuộc về Tạp chí Olympiad Hóa học KEM
Bảng 1: Dữ liệu từ máy giải trình tự protein.

Dựa vào các thông tin trên:


a) Suy ra trình tự chuỗi amino acid chung với mảnh đầu tiên (đầu N) nhận
được bởi các xử lí CNBr và trypsin.
b) Suy ra trình tự chuỗi của mảnh đầu tiên được tạo ra bởi xử lí CNBr.
c) Suy ra trình tự toàn chuỗi của polypeptide ban đầu. Chỉ rõ các tâm bị
phá hủy bởi CNBr và trypsin ở chuỗi này.
d) Tính % các gốc là amino acid có tính base.
e) Biết polypeptide tồn tại ở dạng chuỗi xoắn α, tính độ dài cấu trúc chuỗi
xoắn α này.
f) Tính kích thước của doạn DNA (exon) mã hóa cho polypeptide 40
amino acid này. Xác định kích thước theo các cặp base, cũng như theo
Dalton. (Xét khối lượng phân tử trung bình của một nucleotide trong
DNA là 330.)
g) Giả sử rằng DNA tương ứng với exon chứa cùng số Adenine và
Cytosine, hãy tính số liên kết hydrogen để giữ chuỗi xoắn kép này.

254 | Bản quyền thuộc về Tạp chí Olympiad Hóa học KEM
Hướng dẫn
Trình tự các amino acid trong một protein hoặc polypeptide được biểu
diễn từ amino acid đầu N. Từ phương pháp thoái phân Edman, amino acid
đầu N là Asp. Do đó, trong mảnh đầu N được tạo ra bởi trypsin hoặc CNBr,
amino acid này phải ở vị trí 1. Tất cả các peptide khác được tạo thành bởi
phân cắt CNBr sẽ được đặt trước bởi Met trên phía đầu N. Tương tự như
vậy, tất cả các peptide được tạo thành bởi trypsine sẽ được đặt trước bởi
Arg hoặc Lys. Khi chúng ta tiến hành từ amino acid đầu N đến đầu C,
chúng ta xem xét cẩn thận các amino acid ở mỗi vị trí được đưa ra trong
Bảng 1(a) và 1(b).
Từ mảnh đầu tiên bắt đầu từ Asp đầu N ở vị trí 1, chúng ta tìm các gốc
chung ở mỗi vị trí với các peptide bị phân cắt CNBr và trypsin. Thu được:

Ở vị trí 6, Arg sẽ khiến polypeptide nhạy cảm với trypsin. Do đó, gốc thứ 7
của mảnh CNBr này (Bảng 1(a)) sẽ giống như gốc 1 trong mảnh peptide
khác được tạo ra bởi trypsin và gốc thứ 8 của mảnh CNBr này sẽ giống
như gốc 2 trong Bảng 1(b). Do đó ta có

Do 8 sẽ là Tyr, Pro sẽ được gắn với vị trí 2 của polypeptide …..(3)


Gốc 9 trong polypeptide sẽ ở vị trí thứ 3 trong Bảng 1(b) và các gốc 10, 11,
12, 13 và 14 sẽ phải lần lượt ở các vị trí 4, 5, 6, 7 và 8 trong Bảng 1(b). Các
gốc tương tự phải ở các vị trí từ 1 trở đi trong Bảng 1(a). Không gốc nào
trong các gốc ở vị trí 3 (Bảng 1(b)) giống như vị trí 1 trong Bảng 1(a). Tuy
nhiên, các vị trí 4 đến 8 trong Bảng 1(b) có chung gốc với các vị trí 1 đến
5 trong Bảng 1(a). Hơn nữa, Glu ở vị trí 1 (Bảng 1(a)) sẽ được đặt trước bởi
Met (do nó là một phần của peptide phân cắt CNBr). Và vị trí 3 trong Bảng
1(b) có Met. Do đó, ta có:

255 | Bản quyền thuộc về Tạp chí Olympiad Hóa học KEM
Vị trí 5 trong polypeptide bây giờ có thể được gán chắc chắn cho Ilu …..(5)
Các vị trí 15 và 16 trong polypeptide sẽ vượt quá gốc 8 trong peptide phân
cắt trypsin (không đưa ra ở đây). Bây giờ, chúng ta cố gắng xây dựng các
mảnh trypsin và CNBr còn lại.
Bảng 1(a) cho thấy Arg ở vị trí 1. Gốc này được đặt trước bởi một Met. Kết
hợp các gốc chưa được gán ở vị trí 2 trong Bảng 1(a) với các gốc ở vị trí 1
trong Bảng 1(b) và với các vị trí tiếp sau theo cách đã trình bày ở trước, ta
có:

(2 gốc cuối là các gốc chưa được gán ở vị trí 1 và 2 trong Bảng 1(b)).
Khi xét (2), (5), (6) cùng nhau thì có thể chắc chắn gán vị trí 7 của
polypeptide với Gly …..(7)
a)

b)

Để hoàn thành chuỗi polypeptide chúng ta cần xây dựng trình tự chuỗi của
mảnh trypsin khác. Bắt đầu từ vị trí 4-(Arg) trong Bảng 1(a), ta có trình tự
chuỗi:

Ở bước này, ta lại xem xét các gốc chưa được gán. Arg ở (8) sẽ phải được
đặt trước bởi Asn, Gln, Gly và Met (chúng là các gốc chưa được gán ở các
vị trí tương ứng trong Bảng 1(a)). Sau đó, ta có được trình tự chuỗi

Và sau Ala ở (9)

Từ (9) và (10), ta có trình tự chuỗi


256 | Bản quyền thuộc về Tạp chí Olympiad Hóa học KEM
Do mảnh nhỏ nhất là một dipeptide (Bảng 1(b)) và (6) cho thấy rằng nó
theo sau Lys nên nó sẽ ở đầu C. Do đó, chuỗi cục bộ ở (6) sẽ ở sau chuỗi
cục bộ ở (11). Ta có:

Đã có một Met ở vị trí 9 trong polypeptide. Met tiếp theo chỉ có thể đến
trước tiên ở vị trí 17 do mảnh CNBr có ít nhất 8 amino acid. Do đó, gốc bắt
đầu của (12) có thể được gán cho vị trí 17 - để lại các vị trí 15 và 16 sẽ
được điền bởi các gốc chưa được gán Val và Al trong mảnh CNBr ở các vị
trí 6 và 7 (Bảng 1(a)).
c) Do đó, chuỗi cuối là:

Mũi tên (↓) chỉ các tâm bị phá hủy bởi CNBr và trypsin.
d) Có 6 gốc amino acid có tính base trong peptide. 6/40 = 15%.
e) Một chuỗi xoắn α helix có 3.6 gốc amino acid trên mỗi vòng 5.4Å. Do
đó, chiều dài của polypeptide trong cấu dạng xoắn α sẽ là:
40/3.6 × 5.4 = 59.4 Å.
f) Polypeptide có 40 amino acid. Do mỗi amino acid được mã hóa bởi
một bộ ba nucleotide nên tổng số cặp nucleotide trong DNA mạch kép
của exon sẽ là: 40 x 3 = 120 cặp base.
Khối lượng phân tử DNA tạo thành exon là = 330 x 2 x 120 = 79200 Da
257 | Bản quyền thuộc về Tạp chí Olympiad Hóa học KEM
g) Nếu exon chứa 120 cặp base và A và C có số lượng bằng nhau thì sẽ
có 60 cặp A-T và 60 cặp G-C. Mỗi cặp A-T được giữ bởi 2 liên kết
hydrogen và mỗi cặp G-X được giữ bởi 3 liên kết hydrogen. Do đó tổng
số liên kết hydrogen để giữ chuỗi xoắn kép này là:
(60 x 2) + (60 x 3) = 300

258 | Bản quyền thuộc về Tạp chí Olympiad Hóa học KEM
Bài 22
a) Dự đoán sản phẩm tạo thành khi butanoic acid phản ứng với methyl
alcohol khi có mặt hydrochloric acid? Viết phương trình phản ứng và
gọi tên sản phẩm.
b) Trình bày sự tấn công của alcohol trong giai đoạn tốc định. Đó là sự
tấn công nucleophile hay electrophile? Vẽ trạng thái trung gian.
c) Sản phẩm chính nào được tạo thành khi acid hóa 5-hydroxypentanoic
acid?
Chuyển hóa của các amine RNH2 thành amide RCONH2 có thể được tiến
hành với acetic anhydride:

Trong tổng hợp peptide, thường sử dụng di-tert-butyl dicarbonate, tạo


thành tert-butoxycarbonylamid, hoặc BOC-Amid.
d) Biểu diễn cấu trúc của BOC-Ala.

Nhóm BOC có thể bị tách loại bằng cách xử lí với acid hữu cơ như
CF3COOH.
e) Trình bày quy trình tổng hợp peptide H-Leu-Ala-
OH. Mô tả các giai đoạn - chưa cần viết phương
trình phản ứng. Cho biết chỉ khi tiến hành bảo
vệ các nhóm chức hoạt động của amino acid thì
mới có thể đạt hiệu suất cao.
f) Viết các phương trình phản ứng.

259 | Bản quyền thuộc về Tạp chí Olympiad Hóa học KEM
Hướng dẫn
a)

b) Tấn công nucleophile: Trung gian:

c) Phản ứng nội phân tử:

d)

e) Quy trình tổng hợp H-Leu-Ala-OH:


1. Đưa nhóm bảo vệ vào leucine (nhóm NH2) và alanine (nhóm
COOH).
2. Ghép cặp hai amino acid đã được bảo vệ.

260 | Bản quyền thuộc về Tạp chí Olympiad Hóa học KEM
3. Loại nhóm bảo vệ khỏi peptide tạo thành.
f)
1. Đưa các nhóm bảo vệ vào

2. Ghép cặp các amino acid đã được bảo vệ:

261 | Bản quyền thuộc về Tạp chí Olympiad Hóa học KEM
3. Loại nhóm bảo vệ
Thủy phân acid

Thủy phân base

262 | Bản quyền thuộc về Tạp chí Olympiad Hóa học KEM
Bài 23
1) Trình bày tất cả các giai đoạn và tác nhân cần thiết để tổng hợp mỗi
dipeptide sau từ các amino acid tương ứng của chúng:
(a) Trp-Met
(b) Ala- le
(c) Leu-Val
2) Trình bày tất cả các giai đoạn và tác nhân cần thiết để tổng hợp
tripeptide với trình tự Ile-Phe-Gly.
3) Trình bày tất cả các giai đoạn và tác nhân cần thiết để tổng hợp
pentapeptide với trình tự Leu-Val-Phe- le-Ala.
Hướng dẫn
1) a)

b)

c)

2) Hai gốc amino acid đầu tiên là Ile và Phe. Do đó, chúng ta cần bắt đầu
với các amino acid này. Trước tiên, chúng ta phải thiết lập các nhóm bảo
263 | Bản quyền thuộc về Tạp chí Olympiad Hóa học KEM
vệ phù hợp. Sau đó, xử lí với DCC, các amino acid được bảo vệ sẽ được
ghép cặp. Nhóm bảo vệ ở đầu C sau đó được loại bỏ và đầu C loại nhóm
bảo vệ tạo thành được ghép cặp với amino acid được bảo vệ phù hợp
(glycine, được bảo vệ đầu C) bởi DCC. Và cuối cùng, loại các nhóm bảo vệ:

3)

264 | Bản quyền thuộc về Tạp chí Olympiad Hóa học KEM
265 | Bản quyền thuộc về Tạp chí Olympiad Hóa học KEM
Bài 24
Ngoài cách tạo thành amino acid được bảo vệ Fmoc sử dụng Fmoc-Cl,
nhóm bảo vệ Fmoc cũng có thể được thêm vào nhóm amino bằng cách
sử dụng tác nhân A.
a) Trình bày cơ chế phản ứng thêm nhóm bảo vệ Fmoc vào amino acid.

b) Trình bày cơ chế phản ứng loại nhóm Fmoc khỏi amino acid sử dụng
các điều kiện sau:

266 | Bản quyền thuộc về Tạp chí Olympiad Hóa học KEM
Hướng dẫn
a)

b)

267 | Bản quyền thuộc về Tạp chí Olympiad Hóa học KEM
Bài 25
Peptide là các polyamide mạch thẳng, được tạo thành bởi các liên kết đầu-
đến-đuôi của các α-amino acid, chủ yếu là ở cấu hình L (hay S).
1) Các dipeptide nào có thể được tạo thành bởi sự ngưng tụ L- alanine
and L-phenylalanine? Chỉ rõ hóa lập thể.
2) Sự kéo dài từng phần mạch peptide hầu hết luôn bắt đầu từ nguyên tử
C của các amino acidbậc ba (sử dụng ở dạng ester) liên kết với mỗi
đơn vị amino acidkế tiếp (sử dụng ở dạng dẫn xuất thế ở nitrogen) dẫn
đến sự thay thế nguyên tử N - thế trước khi đơn vị kế tiếp được gắn
vào. Dẫn xuất thế thường được sử dụng là các nhóm alkoxycarbonyl
ROCO - và dẫn xuất carbamatecủa nó. Hãy giải thích lý do tại sao sự
hiện diện của nhóm thế (nhóm bảo vệ) của nguyên tử nitrogen amine
làm trở ngại việc tạo liên kết amide với nhóm cacboxyl.
a) Vì nitơ bây giờ chỉ còn có 1H.
b) Vì nhóm bảo vệ có mật độ electron ít hơn nguyên tử nitrogen.
c) Vì nhóm bảo vệ chắn sự tấn công của nhóm carbonyl.
d) Vì sự kháng tĩnh điện.
e) Vì nó vốn đã là một amide.
3) Vẽ các công thức cộng hưởng của một nửa nhóm amide. Sử dụng các
ký hiệu lập thể và các mũi tên để chỉ rõ sự chuyển dịch electron.
4) Tác nhân nào dưới đây sẽ được sử dụng để gắn nhóm benzyl
carbamate vào một amine (nhóm Bergmann-Zervas). Viết phản ứng.
a) C6H5CH2OCONH2
b) C6H5CH2OCO2CH3
c) C6H5CH2OCO2C(CH3)3
d) C6H5CH2OCOCl
e) C6H5OCOCl
5) Việc loại nhóm bảo vệ alkoxycarbonyl thường kèm theo phản ứng cắt
mạch dưới tác dụng của các acid theo sơ đồ:

Xếp theo chiều tăng dần tính hoạt động của các carbamate sau đây dưới
tác dụng của acid:
268 | Bản quyền thuộc về Tạp chí Olympiad Hóa học KEM
Hướng dẫn
1) Công thức cấu tạo các peptide có thể có:

Các dipeptide vòng (diketo piperazine) cũng được chấp nhận:

2) Câu trả lời tốt nhất là 5 và 2


3)

269 | Bản quyền thuộc về Tạp chí Olympiad Hóa học KEM
4) Tác nhân 4 (Bbenzyl chloroformate) sẽ phản ứng với amine theo sơ
đồ sau: C6H5CH2OCOCl + H2NR ⎯⎯
base
→ C6H5CH2OCONHR + HCl
5) Nếu chúng ta giả thiết trạng thái chuyển tiếp có tạo thành carbonium
ion thì chất nào tạo thành carbonium ion dễ dàng nhất thì tính bền
cũng tỉ lệ thuận với khả năng đó. Trong chất D thì có sự giải toả
electron mạnh nhất:

và khó nhất ở A:

Giải thích tương tự ta thấy cation tạo thành từ B bền hơn C. Như vậy thứ
tự sẽ là: D>B>C>A

270 | Bản quyền thuộc về Tạp chí Olympiad Hóa học KEM
Bài 26
Dẫn ra quy trình tổng hợp từng bước cho Val-Leu-Val từ 3-methylbutanal
[(CH3)2CHCH2CHO] là nguồn nguyên liệu đầu hữu cơ duy nhất. Bạn có thể
dùng thêm bất kì tác nhân vô cơ, hữu cơ nào khác.
Hướng dẫn

271 | Bản quyền thuộc về Tạp chí Olympiad Hóa học KEM
Bài 27
1) Xác định cấu trúc amino acid được bảo vệ phải neo vào chất trợ rắn
để dùng cho tổng hợp Merrifield để điều chế leucine enkephalin.
(Đầu N) Try-Gly-Gly-Phe-Leu (Đầu C)
2) Xác định trình tự tripeptide có thể được tạo thành theo trật tự sau của
các tác nhân. Chỉ rõ đầu C và đầu N của tripeptide:

3) Xác định tất cả các giai đoạn cần thiết để tổng hợp mỗi peptide sau
theo phương pháp Merrifield:
(a) Phe-Leu-Val-Phe
(b) Ala-Val-Leu- le

272 | Bản quyền thuộc về Tạp chí Olympiad Hóa học KEM
Hướng dẫn
1) Leucine được bảo vệ (bởi nhóm Boc):

2) Trước tiên, gốc valine được bảo vệ được gắn vào polymer. Sau khi loại
nhóm bảo vệ, gốc alanine đã bảo vệ được gắn vào. Sau đó, lại loại nhóm
bảo vệ rồi gắn gốc phenylalanine đã bảo vệ vào. Loại nhóm bảo vệ, sau
đó tách khỏi polymer, tạo thành tripeptide sau:
(Đầu N) Val-Ala-Phe (Đầu C)

3) a) Trước tiên, chúng ta gắn gốc phù hợp, được bảo vệ bởi Boc vào
polymer:

Sau đó, nhóm bảo vệ Boc được loại bỏ và một liên kết peptide mới được
tạo thành với amino acid được bảo vệ bởi Boc, sử dụng DCC. Quá trình hai
giai đoạn này (loại bỏ nhóm bảo vệ Boc, sau đó tạo liên kết peptide mới)
được lặp lại để bổ sung thêm mỗi gốc, cho đến khi sắp xếp được trình tự
mong muốn. Cuối cùng, loại bỏ nhóm bảo vệ Boc và peptide mong muốn
được tách khỏi polymer:

273 | Bản quyền thuộc về Tạp chí Olympiad Hóa học KEM
b) Tương tự như trên:

274 | Bản quyền thuộc về Tạp chí Olympiad Hóa học KEM
Bài 28
Quá trình sinh tổng hợp peptide của kháng sinh vancomycin trong các vi
sinh vật bắt đầu với việc xây dựng một chuỗi peptide bởi các enzyme có
khả năng sử dụng các amino acid phi-tiêu chuẩn [không chính tắc] và có
thể đồng phân hóa chúng trong quá trình tổng hợp peptide. Các enzyme
này gồm các module [khối] - mà mỗi module chịu trách nhiệm cho việc
cộng thêm một amino acid. Module gồm một chuỗi các domain với chức
năng cụ thể: Domain A - chọn lọc amino acid, T - gắn kết amino acid hoặc
chuỗi peptide với enzyme, C - tạo thành liên kết amino acid tiếp theo và
phát triển mạnh, E - thay đổi cấu hình R/S trên nguyên tử α-carbon của một
amino acid, Te - tách chuỗi peptide khỏi enzyme.
a) Mỗi amino acid sau có cấu hình R hay S?

Hpg, Dpg, và β-OH-Cl-Tyr là các amino acid phi-tiêu chuẩn. Hpg có cấu trúc
tương tự như Tyr, nhưng M(Tyr) - M(Hpg) = 14 g/mol. Dpg có cấu trúc
tương tự Dpg với khối lượng lớn hơn 16 g/mol. Tất cả các nhóm thế của
Dpg trên vòng phenyl đều có khoảng cách với nhau bằng nhau. Cấu trúc
của β-OH-Cl-Tyr dựa trên cấu trúc Tyr, với nhóm hydroxyl gắn vào nguyên
tử β-carbon và vòng phenyl bị chlorine hóa ở vị trí ortho tương ứng với
nhóm OH.
b) Vẽ cấu trúc của Hpg, Dpg, và β-OH-Cl-Tyr.
Hình dưới đây biểu diễn chuỗi các module của enzyme tham gia vào quy
trình tổng hợp chuỗi peptide vancomycin. Chuỗi amino acid của peptide
được tổng hợp theo các A domain. Quy trình tổng hợp peptide diễn ra theo
chiều từ N → C (nhóm Leu amino không tham gia tạo thành liên kết
peptide). Nếu module có E-domain thì amino acid được thêm vào theo cấu
hình R. Còn nếu không có thì theo cấu hình S. (Trong hình: domeen =
domain.)

275 | Bản quyền thuộc về Tạp chí Olympiad Hóa học KEM
c) Vẽ cấu trúc phẳng của peptide tạo thành và chỉ rõ cấu hình với mỗi
amino acid.

276 | Bản quyền thuộc về Tạp chí Olympiad Hóa học KEM
Hướng dẫn
a) Asn - R; Leu - R; Thr - 2S, 3R và Tyr - S
b)

c)

277 | Bản quyền thuộc về Tạp chí Olympiad Hóa học KEM
Bài 29
1. Một peptit P có hoạt tính sinh học được cô lập từ một loài cóc ở
Nam Mỹ. Thứ tự sắp xếp các aminoaxit trong peptit P có thể được
xác định dựa trên các thông tin sau
- Heptapeptit P có thành phần gồm 6 aminoaxit thiên nhiên và một
aminoaxit ở dạng D. Peptit P chứa đầu C amit.
- Phản ứng giữa P và 2,4-DNFB rồi thủy phân trong axit sinh ra sản
phẩm có khối lượng phân tử 347.
- Cắt mạch peptit P bằng chymotrypsin thu được các phân đoạn có
khối lượng phân tử lần lượt là R 181, S 201, T 236 và U 238 g/mol.
Phân tách các peptit nhỏ hơn này rồi cho phản ứng với DNFB sau
đó thủy phân trong axit thu được DNP – Pro (S); DNP – Ala (T) và
DNP – Gly (U).
- Cắt mạch peptit P bằng enzym thermolysin thu được hai mảnh W
(252 g/mol) và X (568 g/mol). Biết rằng thermolysin là enzym cắt
mạch peptit ở đầu N của một aminoaxit kị nước (mạch ankyl phân
nhánh hay chứa vòng thơm).
- Peptit P không phản ứng với enzym dipeptidylpeptidaza. Biết rằng
enzym dipeptidylpeptidaza cắt mạch peptit ở nhóm carbonyl của
aminoaxit thứ hai.
Hãy xác định cấu trúc các mảnh chưa biết, từ đó suy ra cấu trúc
peptit P.

2. Một heptapeptit (2 Met, Asp, Arg, Lys, Phe và Gly) được phân lập
từ một loài sinh vật được cho là có hoạt tính sinh học. Các kết quả
xác định trật tự sắp xếp của aminoaxit trong peptit này được cho
như sau. Từ đó hãy cho biết trật tự sắp xếp của các aminoaxit
trong peptit này.
- Thủy phân peptit bằng enzym cacboxipeptidaza cho Met.
- Tiến hành phân cắt heptapeptit này bằng CNBr thu được 1 mol
homoserin lacton. Còn khi phân cắt bằng chymotrypsin thu được
một pentapeptit và một dipeptit.
- Phân cắt bằng trypsin cho hai aminoaxit đều có chứa Met và Arg
tự do.
- Khi phân cắt bằng pepsin cho một tetrapeptit (Met, Asp, Lys, Arg)
và một tripeptit.

278 | Bản quyền thuộc về Tạp chí Olympiad Hóa học KEM
3. Các nhà nghiên cứu vừa mới phân lập một peptit thiên nhiên có
cấu trúc lý thú. Xử lý peptit này với axit mạnh thu được nhiều sản
phẩm, trong đó có dẫn xuất có cấu trúc vòng như hình dưới. Dẫn
xuất này được cho tham gia phản ứng Edman. Đề nghị sản phẩm
của phản ứng này và giải thích bằng cơ chế phản ứng (tạo ra dẫn
xuất thiazoline).

4. Các nhà khoa học đã cố gắng để xác định được enzym  -


lactamaza từ Staphylococcus aureus. Khi enzym tinh khiết được
đánh dấu bằng đồng vị 32P thì ta chỉ đánh dấu một mình Serin.
Trong phân tích thì serin chỉ chiếm 0,35% về khối lượng của enzym
 - lactamaza. Biết MSerin = 105 g/mol.
a) Ước lượng Menzym.
b) Để đánh dấu đầu hoạt động,  - lactamaza được phân cắt bởi
tripsin. Hexapeptit ban đầu (P1) chứa các aminoaxit sau: Glu, Leu,
Lys, Met, Phe và Ser
• Sử dụng phương pháp Edman ta biết được aminoaxit “đầu N” là
Phe và dãy peptit P2.
• Sử dụng BrCN để cắt mạch P1 thu được dipeptit P4 và tetrapepetit
P3
• Sử dụng 1 – flo – 2,4 – dinitrobenzen sau đó thuỷ phân hoàn toàn
peptit P2 ta thu được N–2,4–dinitrophenyl–Glu .
Biết P1; P2; P3 đều chứa Ser. Hãy xác định vị trí các aminoaxit trong
P1; P2; P3; P4.
5. Hoàn chỉnh sơ đồ phản ứng sau đây:

279 | Bản quyền thuộc về Tạp chí Olympiad Hóa học KEM
Hướng dẫn
1. Các mảnh chưa xác định:
S: H – Pro – Ser – NH2 T: H – Ala – Phe – OH
U: H – Gly – Tyr – OH X: H – Phe – Gly – Tyr – Pro
– Ser – NH2
Dựa trên các kết quả phân tích khối lượng và sản phẩm phân cắt
từ các enzym trypsin và chymotrypsin cũng như việc peptit P không bị cắt
mạch bởi enzym dipeptidyl cho thấy Aminoaxit có cấu hình D là Ala
Như vậy cấu trúc của peptit cần xác định sẽ là:

2. Dựa trên kết quả phân cắt bằng enzym cacboxipeptidaza thì Met
là aminoaxit đuôi C.
Việc cắt mạch bằng CNBr thu được homoserin cho thấy aminoaxit
đầu N cũng là Met (CNBr cắt mạch sau Methionin, và bản thân nó chuyển
thành homoserin lacton theo cơ chế như sau)

Chymotrypsin cắt mạch sau aminoaxit thơm, như vậy aminoaxit


thứ năm của dãy (vì chia thành pentapeptit và dipeptit) sẽ là Phe.
Do aminoaxit cần phân tích có đầu N và C đều là Met, thế nên việc
cắt bằng trypsin cho ra Arg tự do cho thấy nó chính là aminoaxt thứ tư

280 | Bản quyền thuộc về Tạp chí Olympiad Hóa học KEM
trong thứ tự. Điều này dẫn đến kết quả Lys là aminoaxit thứ ba (Trypsin
cắt mạch sau Lysin và araginin).
Khi tiến hành cắt mạch bằng pepsin thu được tetrapeptit Met, Asp,
Lys, Arg tức thứ tự của nó sẽ là Met – Asp – Lys – Arg. Tripeptit còn lại
gồm ba aminoaxit Phe, Gly, Met sẽ có thứ tự Phe – Gly – Met.
Như vậy trật tự sắp xếp của các aminoaxit trong heptapeptit sẽ là
Met – Asp – Lys – Arg – Phe – Gly – Met.
3. Cơ chế phản ứng:

4. Do Mserin = 105 Da và Serin chiếm 0,35% khối lượng peptit nên có


thể dự đoán gần đúng Mpeptit = 105.100/0,35 = 30000 Da.
Có thể xác định được Phe là aminoaxit đầu N bằng phương pháp
Edman
BrCN chỉ cắt sau Met nên với việc thu được một dipeptit và một
tetrapeptit có thể xác định được Met là aminoaxit thứ tư.
Khi cho P2 phản ứng với DNFB thu được dẫn xuất của Glu chứng
tỏ nó là aminoaxit đầu mạch P2, tức nó nằm ngay sau Phe trong
hexapeptit.
281 | Bản quyền thuộc về Tạp chí Olympiad Hóa học KEM
Peptit có thể bị phân cắt bởi trypsin tức Lys không thể là aminoaxit
cuối mạch.
Do tất cả các peptit P1, P2 và P3 đều chứa Ser nên thứ tự sắp xếp
của các aminoaxit trong peptit sẽ như sau:
P1: Phe – Glu – Ser – Met – Lys - Leu
P2: Gly – Ser – Met – Lys - Leu
P3: Phe – Glu – Ser – Met.
P4: Lys – Leu

5. Sơ đồ phản ứng tổng hợp dipeptit Gly – Ala (PS là gốc polysyren)

282 | Bản quyền thuộc về Tạp chí Olympiad Hóa học KEM
Phụ lục: Các chuyên đề hóa sinh
& polymer
(Trích từ đề thi Olympiad Hóa học các quốc gia trên thế giới những năm
gần đây)

283 | Bản quyền thuộc về Tạp chí Olympiad Hóa học KEM
Hóa sinh
Bài 1
BPTI là một polypeptide chứa 58 amino axit (AA).
Hình bên cho thấy một phần của cấu trúc xoắn  và
cấu trúc lớp -sheet, và cũng có phần có cấu trúc
khung sườn không được ưu tiên. Điểm quan trọng
trong cấu trúc của BPTI là sự có mặt của ba liên kết
cầu disunfua.

A. Câu hỏi:
a) Threonin là một AA có hai nguyên tử C bất đối. Hãy
vẽ tất cả các cấu dạng có thể có của Threonin và gọi tên chúng theo
danh pháp hệ thống.
b) Đánh số các cấu trúc trên và hãy chỉ các cặp chất là đồng phân lập
thể của nhau .
c) Tính điếm đẳng điện của AA glutamic và vẽ công thức của chất đó
ứng với điểm đẳng điện này.
pK (-COOH) = 2.2 pK (-NH3+) = 9.7 pK (mạch nhánh) = 4.3

B. Cấu trúc sơ cấp:


Sự sắp xếp các aminoaxit của BPTI từ 1-37 được cho ở trang bên.
Hãy tìm sự sắp xếp các AA 38 đến AA 58 khi sử dụng các thông tin sau:
Sau khi bẻ gãy các liên kết disunfua, trong một thí nghiệm thì BPTI bị
phân cắt bởi enzym (cắt ở nhóm cacboxyl của lysin và arginin). Trong
một thí nghiệm khác thì BPTI bị cắt mạch bởi 2-Nitro-5-thioxyanobenzoat
(cắt ở nhóm amino của cystein). Sự sắp xếp các AA trong các chuỗi bị
cắt trên đã được xác định
Như vậy arginin và các dãy peptit sau đã được xác định:
(Hướng dẫn: AA 38 và 39 chỉ được tìm thấy một lần trong peptit. Nó là một
phần của các mảnh dài hơn đã được xác định ở trước đó.)
(1) Thr – Cys – Gly – Gly – Ala
284 | Bản quyền thuộc về Tạp chí Olympiad Hóa học KEM
(2) Cys – Arg – Ala – Lys – Arg – Asn – Asn – Phe – Lys – Ser – Ala –
Glu –Asp
(3) Cys – Met – Arg – Thr
(4) Ser – Ala – Glu – Asp – Cys – Met – Arg
(5) Ala – Lys
(6) Asn – Asn – Phe – Lys
(7) Cys – Gly – Gly -Ala

Arg-1 Pro-2 Asp-3 Phe– Cys-5 Leu-6 Glu-7 Pro-8 Pro-9 Tyr-
4 10
Thr- Gly- Pro- Cys- Lys- Ala- Arg- Ile- Ile-19 Arg-
11 12 13 14 15 16 17 18 20
Tyr- Phe- Tyr- Asn- Ala- Lys- Ala- Gly- Leu- Cys-
21 22 23 24 25 26 27 28 29 30
Gln- Thr- Phe- Val- Tyr- Gly- Gly- -38 -39 -40
31 32 33 34 35 36 37
-41 -42 -43 -44 -45 -46 -47 -48 -49 -50
-51 -52 -53 -54 -55 -56 -57 -58

Câu hỏi
a) Gọi tên các mảnh sinh ra khi phân cắt mạch (chỉ đối với AA 38 – 58).
b) Đặt các AA bị mất vào bảng trên.
c) Vẽ CTCT của dipeptit Ala-Lys.
d) Có bao nhiêu mảnh sinh ra khi cắt mạch bằng BrCN?
C. Cấu trúc bậc 2:

285 | Bản quyền thuộc về Tạp chí Olympiad Hóa học KEM
Cho giản đồ Ramachandranplot và các giá
trị sau:

Phi  Psi 

Tyr - 21 - 96,66 149,81

Arg - 53 - 67,79 - 36,00

Câu hỏi:
a) Nêu định nghĩa về góc xoắn  sử dụng công thức Newman
b) Loại cấu trúc bậc hai nào mà bạn có thể tìm thấy đối với vùng
1. Tyr – 21?
2. Arg – 53?
Hướng dẫn
A. Aminoaxit

COO- COO- COO- COO-


a) +H N
3 C H H C NH3+ +H N
3 C H H C NH3+
H C OH HO C H HO C H H C OH
CH3 CH3 CH3 CH3
(1) (2) (3) (4)

(1) Axit 2S-Amino-3R-hydroxybutyric (2) 2R-3S (3) 2S- 3S (4) 2R- 3R

b) (1)(2) và (3)(4) Enantiome (1)(3) ; (1)(4) ; (2)(3) và (2)(4)


Đồng phân dia
COO-
+H N C H
3

c) pH = (2.2 + 4.3)/2 = 3.25 CH2


CH2
COOH

286 | Bản quyền thuộc về Tạp chí Olympiad Hóa học KEM
B. Cấu trúc sơ cấp
a) Arg / (1) / (4) / (5) / (6)
b) Cys-Arg-Ala-Lys-Arg-Asn-Asn-Phe-Lys-Ser-Ala-Glu-Asp-Cys-Met-
Arg-Thr-Cys- Gly-Gly-Ala

O
+H N CH C NH CH COO-
3
c)
CH3 (CH2)4
NH3+

d) 2

C. Cấu trúc bậc hai


H O C'
a)
N C' C'
C' C C
O R H R

1. gấp  2. xoắn 

287 | Bản quyền thuộc về Tạp chí Olympiad Hóa học KEM
Bài 2
Tiêu hóa là một quá trình xử lí cơ học, hóa học và xúc tác enzyme của thực
phẩm, dẫn đến sự tạo thành các chất dinh dưỡng có thể được hấp thụ vào
hệ tiêu hóa (GIT).
1) Trong Phiếu trả lời, hãy xác định mối liên hệ giữa các enzyme với các
kiểu chất dinh dưỡng trong quá trình thủy phân xúc tác enzyme các
tiền chất. (Bỏ qua)
Các chất chuyển hóa phân tử khối thấp được tạo thành sẽ cung cấp, cùng
với các chất khác, năng lượng cần cho cơ thể.
2) Viết các phản ứng đốt cháy hoàn toàn palmitic acid (acid béo giới hạn
có chứa 16 nguyên tử carbon), glucose, glycine (α-amino acid không
quang hoạt) và glycerin.
Trong các tế bào đặc biệt của hệ tiêu hóa, các quá trình liên quan đến sự
tổng hợp chất X có trong quá trình tiêu hóa. Cấu trúc của tế bào GIT được
cho trong hình. Giản đồ biểu diễn 3 kênh ion liên màng.

3) Xác định các chất X1-X4 và đề xuất cấu trúc của X tạo thành từ X3,
biết rằng: a) hàm lượng carbon trong X2 là 19.4 %; b) hoạt động của
mỗi kênh ion không dẫn đến sự biến đổi điện tích tổng trong vùng
không gian ngoại tế bào; c) sự vận chuyển liên màng các phân tử và
hạt mang điện chỉ diễn ra qua các kênh ion.
95 % các trường hợp loét dạ dày có liên quan đến vi khuẩn Helicobacter
Pylori. Để xác định thực trạng nhiễm trùng, một thí nghiệm kiểm tra hơi
thở sẽ được sử dụng. Bệnh nhân được yêu cầu uống dung dịch A (21.29
% C và 6.61 % H về khối lượng) hoặc B (22.56 % C và 6.51 % H về khối
lượng), sau đó xác định bằng máy đo phổ tỉ lệ khí C và D (trong trường hợp
A) hoặc lượng khí E (trường hợp B) trong hơi thở của bệnh nhân.
4) Xác định các chất A-E, biết rằng A và B:

288 | Bản quyền thuộc về Tạp chí Olympiad Hóa học KEM
- trong thành phần chỉ có đồng vị hydrogen thông thường.
- có tỉ lệ số nguyên tử C và H là số nguyên.
- chứa C, N, H và O và có khối lượng mol < 100 gram/mol.
- không phải là các chất điện li.
5) Viết phương trình phản ứng tạo thành D từ A (1) và E từ B (2).
6) Vì lý do gì mà Helicobacter Pylori tổng hợp một enzyme xúc tác cho
chuyển hóa A thành D? Bình thường thì enzyme này không có trong dạ
dày con người.
Hướng dẫn
1) Bỏ qua.
2)

3) Х1 - СО2, Х2 - Н2СО3, Х3 - Н3О+, Х4 - HCO3-, X - HCl.


4) A là 13C-urea, B là 14C-urea, C là 12CO2, D là 13CO2, E là 14CO2.
5)

6) Vi khuẩn tổng hợp urease, để thủy phân urea chuyển đến từ máu thành
ammonia, chất này tạo liên kết với HCl trong môi trường gần nhất của vi
khuẩn, do đó làm tăng pH. Nếu không, vi khuẩn không tồn tại được trong
môi trường khắc nghiệt của dạ dày. Chú ý rằng urea chỉ độc với các động
vật có vú (đa bào), nhưng không độc với vi khuẩn.

289 | Bản quyền thuộc về Tạp chí Olympiad Hóa học KEM
Bài 3
Sỏi niệu là căn bệnh khá phổ biến và các loại sỏi thường có thành phần
hóa học đa dạng. Đa số chúng là các muối calcium A1 và B1 của các acid
đa chức A và B.
Để trung hòa dung dịch chứa 1.00 gram A cần dùng 44.4 mL dung dịch
KOH 0.5 M, có mặt chỉ thị phenolphthalein. Tiến hành thí nghiệm tương tự
với dung dịch B cần 40.8 mL dung dịch KOH 0.5 M. Biểu đồ nhiệt trọng (sự
phụ thuộc của biến thiên khối lượng mẫu theo nhiệt độ khi nung) cho thấy
A1 (1.37 % H về khối lượng), có 3 peak và bã rắn còn lại sau khi nung ở
nhiệt độ cao có khối lượng bằng 38.4 % mẫu ban đầu. B1 có chứa 23.3 %
Ca về khối lượng và phản ứng với cả dung dịch acid lẫn base.
1) Xác định A, A1, B, B1. Viết các phương trình phản ứng.
Để hòa tan các viên sỏi tạo thành từ A1, B1 cần sử dụng các dung dịch
sodium và potassium citrate.
2) Đánh dấu vào Phiếu trả lời để xác định mục đích sử dụng các citrate
trong điều trị sỏi niệu.
Chất C là sản phẩm quan trọng của quá trình chuyển hóa nitrogen. Vi
phạm chuyển hóa C dẫn tới sự hình thành sỏi. Quá trình sinh tổng hợp C
từ adenine và guanine diễn ra theo sơ đồ sau:

3) Xác định các cấu trúc C, C1, C2 biết sự oxid hóa không ảnh hưởng đến
các nguyên tử nitrogen.
Các viên sỏi thuộc một kiểu khác được tạo thành trong vi phạm quá trình
lọc thận của α-amino acid D. Đốt cháy hoàn toàn 1000 mg sỏi loại này rồi
dẫn sản phẩm cháy qua dung dịch nước vôi dư, thu được 3500 mg kết tủa
trắng và còn lại 101.4 mL khí (20 oC và 100 kPa).
4) Xác định chất D, chỉ rõ hóa lập thể.
5) pH bình thường của nước tiểu là khoảng 7. Hãy chọn trong Phiếu trả
lời đồ thị mô tả sự phụ thuộc độ tan của amino acid này vào pH. Hãy
đề xuất giải pháp ngăn chặn sự hình thành các loại sỏi tương ứng.
290 | Bản quyền thuộc về Tạp chí Olympiad Hóa học KEM
291 | Bản quyền thuộc về Tạp chí Olympiad Hóa học KEM
Hướng dẫn
1)

A1 - CaC2O4 ⋅ H2O
B1 - CaHPO4 ⋅ 2H2O
Phương trình phản ứng:
H2C2O4 + 2OH- → C2O42-+ 2H2O
H3PO4 + 2OH- → HPO42- + 2H2O
CaC2O4·H2O → CaC2O4 + H2O
Ca2O4 → CaCO3 + CO
CaCO3 → CaO + CO2
CaHPO4 + H+ → Ca2+ + H2PO4-
3CaHPO4 + 2OH- → HPO42- + Ca3(PO4)2 + 2H2O
2) Việc bổ sung các citrate (muối của acid ba chức citric) chỉ có thể dẫn
tới sự tăng pH và do đó chuyển các acid A và B thành những anion chỉ có
thể tạo thành muối calcium ít tan. Sự tham gia của các citrate vào chu
trình Krebs không ảnh hưởng đến độ tan của sỏi. Sự hòa tan sỏi là bởi sự
tạo thành các phức vòng càng tan tốt của calcium citrate.
3) Uric acid được tạo thành trong cơ thể theo chuyển hóa này:

292 | Bản quyền thuộc về Tạp chí Olympiad Hóa học KEM
4)

5) Các amino acid là hợp chất lưỡng tính. Trong môi trường trung hòa của
dung môi phân cực, chúng chủ yếu tồn tại ở dạng ion lưỡng cực. Hàm
lượng cực đại của dạng này (và độ tan cực tiểu) đạt được ở điểm đẳng
điện. Do trong cystine thì số nhóm carboxyl và amino bằng nhau nên điểm
đẳng điện sẽ nằm trong khoảng môi trường acid yếu do ảnh hưởng tương
hỗ của chúng (giá trị tham chiếu là pI 5.0). Với sự gia tăng tỉ lệ của các
dạng muối, độ tan của cystine sẽ tăng và trạng thái thực tế được mô tả
bởi đồ thị dạng chữ U. Do pH 7 tương ứng với nhánh tăng dần của đồ thị
nên cần giảm độ acid để tránh sự tạo thành sỏi.

293 | Bản quyền thuộc về Tạp chí Olympiad Hóa học KEM
Bài 4
Tìm kiếm các hợp chất thiên nhiên có khả năng kháng ung thư là một trong
những nhánh phát triển mạnh của khoa học hiện đại. Các kết quả nghiên
cứu gần đây được xem xét dưới đây.
X là một dược phẩm antineoplatic tiềm năng. Để nghiên cứu cơ
chế tạo thành nó từ các tiền chất khác nhau, một hỗn hợp của ba chất
được tổng hợp trong phòng thí nghiệm A, B và C được cho phép tiêm vào
chuột theo liều lượng 63,5, 58,5 và 39,6 g trên kg cơ thể cân nặng tương
ứng.
A và B là các -amino axit bền trong tự nhiên. Phần dư của 1 trong những
hợp chất này được tìm thấy trong protein. Thông tin về A, B và C được
tổng hợp ở bảng sau:
Hàm lượng, khối lượng (%) Số các
Số các nguyên
ngyên tố
Chất tử bất đối
C H O trong hợp
quang học
chất
A 31,09 5,74 16,57 5 1
B 26,67 5,04 17,77 5 1
Tìm thấy
C 9,24 3,10 4 0
trong C
Biết rằng
- A, B và C có khối lượng phân tử nhỏ hơn 250 g/mol
- A, B và C có chứa C, H, N và O (không bắt buộc tất cả các nguyên tố này)
theo các tỉ lệ đồng vị thông thường.
- Số lượng nguyên tử N tuân theo bất đẳng thức sau: Nnitơ(B)  Nnitơ(A).
1. Xem xét tất cả các khả năng với số nguyên tử nitơ trong A và B, hãy xác
định thành phần nguyên tố của chúng.
2. Nếu bạn không trả lời được câu 1, hãy xem xét thêm gợi ý: A và B có
cùng số lượng nguyên tử nitơ.
3. Hãy vẽ tất cả các công thức có thể của B (không kể công thức lập thể
cụ thể).
294 | Bản quyền thuộc về Tạp chí Olympiad Hóa học KEM
4. Nếu các dữ kiện cung cấp là đủ, hãy chỉ ra cấu hình tuyệt đối (R hoặc S)
ở các trung tâm lập thể của các cấu trúc trong câu 3.

Trong khi tiến hành thí nghiệm, các mẫu không khí xả ra từ động vật được
thu thập lại ở thời điểm xác định giữa các thí nghiệm. Các chất sau đây
(bên cạnh các chất vi sinh khác) tìm thấy được:
Các chất khí tìm thấy Tỉ trọng tương đối với H2 Hợp chất tiền chất
A1 53 A
B1 53,5 B
C1 56 C
5. Hãy vẽ cấu trúc của A1 và B1. Nếu chúng ta biết A1 chỉ có một loại
nguyên tử và hiđro và không có chứa liên kết .
Sự tạo thành C1 từ C trong chuột thực hiện qua hai giai đoạn enzym: khử
C cho chất trung gian X kèm theo sự chuyển hóa thành C1.
6. Xác định cấu trúc của C, C1 và antieoplastic trao đổi chất X nếu chúng
ta biết C không có chứa các liên kết C-O.
Sự tạo thành A1 và B1 từ A và B tương ứng cũng xảy ra qua hai giai đoạn,
giai đoạn sau cũng sử dụng cùng xúc tác enzym giống như sự chuyển
dạng của X thành C1.
7. Hãy xác định cấu trúc của A và B.
8. Hãy biện luận về lựa chọn khối lượng của A, B và C trong hỗn hợp áp
dụng cho chuột.
Một trong những amino axit được thảo luận ở trên có thể được thấy ở
trong protein. Người ta có thể nói rằng amino axit này không có trong RNA
vận chuyển của nó (tRNA).
9. Hãy quyết định phần còn lại của amino axit nào (A hoặc B) có thể tìm
thấy trong protein? Từ các biến số liệt kêt dưới đây, chọn một để giải thích
xem nó xuất hiện như thế nào trong các protein.
TT Biến đổi

295 | Bản quyền thuộc về Tạp chí Olympiad Hóa học KEM
1 A, Bởi vì nó được tạo thành như là kết quả của một bước biến đổi
chuyển dịch sau của một aminoaxit canonical
2 A, bởi vì nó có cấu trúc tương tự như aminoaxit canonical, thứ
thỉnh thoảng dẫn đến sự chèn sai trong quá trình dịch mã
3 A, bởi vì nó có thể liên quan đến sinh tổng hợp protein ở ribosom
mà không trải qua quá trình tiền tạo thành aminoaxyl-tRNA.
4 B, bởi vì nó có cấu trúc tương tự như amino axit canonical, thứ
thỉnh thoảng dẫn đến sự chèn sai trong quá trình dịch mã
5 B, bởi vì nó có thể liên quan đến sinh tổng hợp protein ở ribosom
mà không trải qua quá trình tiền tạo thành aminoaxyl-tRNA.
Hướng dẫn
1. Tính toán tỉ lệ mol của cacbon, hiđro, oxi trong A-C cho phép xác định
khối lượng phân tử nhỏ nhất ứng với công thức (chú ý rằng tỉ số đồng vị
của C, H, N, và O là tự nhiên):
Hợp Tỉ lệ tính toán Tỉ lệ KLPT nhỏ nhất,
chất g/mol
A 31,09 5,74 16,57 60,05.100
nC:nH:nO= : : M= = 193,1
12,01 1,008 16,00 31,09
= 5:11:2
B 26,67 5,04 17,77 48,04.100
nC:nH:nO= : : M= = 180,1
12,01 1,008 16,00 26,67
= 4:9:2
C 9,24 3,10 12,01.100
nC:nH = : = 1:2,25= M= = 130,0
12,01 1,008 9,24
1:4
Với thông tin cung cấp giới hạn (M < 250 g/mol), khối lượng phân tử thực
và nhỏ nhất là trùng khớp. KLPT dư còn lại thuộc về hai nguyên tối còn lại
(bên cạnh C , H, và O) trong A và B là 90,0 và 91,0 g/mol tương ứng. Có hai
lý do đằng sau sự khác biệt này về KLPT dự kiến của A và B (90,1-90.0 = 1 g/mol).
Sự khác biệt của khối lượng nguyên tử của nguyên tố thứ năm trong A và
B và/ hoặc số nguyên tử nitơ khác nhau trong các hợp chất này. Tất cả

296 | Bản quyền thuộc về Tạp chí Olympiad Hóa học KEM
các biến số của số nguyên tử nitơ (không quá 6) trong A được xem xét
dưới đây:
Số nguyên Khối lượng phân tử Các biến số
Hợp chất hóa
tử nitơ dư còn lại của nguyên của nguyên
sinh có thể đúng
trong A tố thứ 5 trong A tố thứ 5
1 76 - -
2 62 2P Có thể xem xét
1 Ti ? Không thể
2 Mg? Không thể
3 48
3 O? Không thể
4 C? Không thể
4 34 - -
5 20 1 Ne? Không thể
6 6 6 H? Không thể
1. Với thông tin không cân bằng đã cho của đề bài, biến số 2 của nguyên
tử nitơ trong A tương ứng với 1 hoặc 2 nguyên tử nitơ trong B, và 75 hoặc
63 g/mol còn lại cho nguyên tố thứ 5 trong hợp chất sau, tương ứng.
Không có biến số nào đúng với các giá trị ở trên. Vì thế, chúng ta lại gặp
phải bức tường gạch.
2. Sự khác nhau 1 g/mol trong KLPT của nguyên tố thứ 5 trong A và B còn
lại như là lý do duy nhất. Điều này có thể đúng trong trường hợp đồng vị
(chú ý rằng đồng vị tự nhiên được đề cập chỉ cho bốn nguyên tố). Nếu như
thế, các đồng vị có thể bền (khả năng bền của tất cả các hợp chất đầu) và
giống như một và cùng nguyên tố (A, B, và C là tiền chất của cùng một
hợp chất X). Với số các nguyên tử nitơ bằng nhau trong A và B, các tập
hợp sau đâu của tổ hợp đồng vị có sẵn: 20-21; 34-35; 48-49; 62-63; 76-77.
Tiếp đó, sự khác nhau của 1 g/mol được đề nghị duy nhất một nguyên tử
của nguyên tố trong mỗi A và B.
Hai tổ hợp đồng vị bền (48Ti-49Ti và 76Se-77Se) phù hợp tốt. Khi không có
amino axit tự nhiên chứa titan, thành phần nguyên tố của A và B cuối cùng
là C, H, N, O và Se.

297 | Bản quyền thuộc về Tạp chí Olympiad Hóa học KEM
3. Như đã xác định ở trên, công thức phân tử của B là C4H9SeNO2. Bốn cấu
trúc có thể đề xuất cho -amino axit này. Cấu trúc bên phải nhất có chứa
2 nguyên tử quang học, như thế là không có giá trị, trái lại công thức bên
trái nhất là không bền. Như thế chỉ còn hai cấu trúc ở giữa còn lại là câu
trả lời đúng.

4. Cả cấu hình amino axit- R- và –S tìm thấy trong tự nhiên. Vì đã chưa đề


cập trong bài tập công thức chính xác của A và B trong protein, điều này
không thể gán một cách chính xác cấu hình cacbon- nếu không có thêm
các thông tin khác.
5. Các khí A1, B1 và C1 có KLPT là 106, 107 và 112 g/mol tương ứng. Nếu
như xem rằng sự khác nhau trong KLPT của A và B (1 g/mol) được duy trì
đối với các tế bào trao đổi chất của chúng. Như thế A1 và B1 giống như
các phân tử đồng vị. Bên cạnh sele, A1 và có chứa các nguyên tố với tổng
KLPT còn lại là 106 -76 = 30 g/mol. Khi các tế bào trao đổi chất có khí
chứa nguyên tố hiđro, có hai khả năng biến số của công thức phân tử của
chúng: C2H6Se hoặc CH2SeO. Với việc chuẩn bị xác định nguyên tử hiđro
trong A, công thức cấu trúc sau là có khả năng:

Trong số đó, chỉ đimetylselenua là không chứa các liên kết . Cuối cùng,
A1 là (CH3)276Se và B1 là (CH3)277Se.
6. Khối lượng nguyên tử của đồng vị selen trong C1 là 76 + (112-106) = 82
a.u. (Chú ý rằng tế bào chất cuối cùng giống nhau với ba chất nguyên bản
ban đầu). KLPT dư còn lại với nguyên tố thứ 4 trong C (nó chỉ chứa 4
nguyên tố) là 130- 16 – 82- 32 g/mol, chất mà tương ứng với hai nguyên
tử oxi. Như thế, công thức phân tử của C là CH4O82Se.
Sự có mặt nhóm metyl C1 cũng như sự thiếu các liên kết C-O trong cấu
trúc cho phép kết luận cuối cùng của cấu trúc phân tử của C (công thức
bên trái dưới đây với 82Se) là:

298 | Bản quyền thuộc về Tạp chí Olympiad Hóa học KEM
Sau đó, X là metylselenua CH3SeH và C1 là (CH3)2Se được tạo ra như là
kết quả của sự metyl hóa X (phản ứng chéo hóa).
7. Như đã xác định trong mục 6, sự metyl hóa là bước thứ hai của các quá
trình xem xét. Với xu hướng đặc biệt cao của enzym, tất cả các chất nền
đều có thể metyl hóa phải tương tự nhau. Như thế, các phân tử đồng vị X
(CH376SeH và CH377SeH) là các chất trung gian có thể bắt nguồn từ A và B
thành A1 và B1 tương ứng. Các chất trung gian này có thể bắt nguồn trực
tiếp chỉ từ các hợp chất có chứa CH3Se- dư. Như thế selenonmethion và
metylselenocysteine có thể gán cho A và B:

8. Khi thí nghiệm ở điều kiện thảo luận ở đây mục đích là tìm ra con đường
của selenum tế bào chất, người ta có thể kiểm tra khối lượng của selen
trong mỗi mẫu đã cho. Các tính toán liên quan đến KLPT của A, B và C và
khối lượng của các hợp chất này trong hỗn hợp cung cấp kết quả tuyệt vời:
hỗn hợp có chứa 25 g mỗi đồng vị selen.
9. Biến số 2 là lựa chọn đúng. Selenomethionine có cấu trúc tương tự
methionine (so với cấu trúc dưới đây), mà thi thoảng dẫn tới nhầm lẫn
trong chuyển dịch và chèn sai một amino axit chứa selen thay cho amino
axit chứa lưu huỳnh.

299 | Bản quyền thuộc về Tạp chí Olympiad Hóa học KEM
8. Đồng vị 76Se cũng tìm thấy trong tự nhiên (khoảng 1% tổng selen) nên
sự dư của selenothionine với 76Se có thể tìm thấy (mặc dù rất hiếm) trong
protein.
Biến thể 1 là không thể vì các biến đổi hậu dịch mã dẫn đến A phải liên
quan đến sự metyl hóa selenothionine dư, amino axit sau cũng thiếu trong
tRNA.

Biến số 3 và 5 cũng không thể, vì protein sinh tổng hợp chỉ chấp nhận một
cách kéo dài mạch polipeptit mà liên quan đến một amino axit dư chuyển
từ aminoaxyl-tRNA.
Biến số 4 là không thể với lý do giống như biến số 2. Metylselenocysteine
có cấu trúc tương tự như S-methylcysteine (so sánh cấu trúc dưới đây),
mà không chứa một amino axit ghi nhận, như thế cũng thiếu tRNA của nó:

300 | Bản quyền thuộc về Tạp chí Olympiad Hóa học KEM
Bài 5
Hẳn bạn đã biết rằng quá tình sinh tổng hợp protein trong cơ thể được mã
hoá. Một bộ ba nucleotide (codon) trong mRNA chịu trách nhiệm đưa vào
protein một amino acid nhất định. Điều ít được biết tới hơn là sự tổng hợp
nucleotide trong cơ thể sống có sự tham gia của các amino acid. Trình tự
các giai đoạn sinh tổng hợp pyrimidine nucleotide có tên uridine-5’-
phosphate (UMP, hợp chất F) được cho như sau:

Trong đó NAD+ và NADH là các dạng oxid hoá và khử hoá của coenzyme
nicotine amide adenine dinucleotide; ADP là adenosine diphosphate; ATP
là adenosinetriphosphate.
1) Xác định công thức các hợp chất B - F. Biết rằng C chứa vòng 6 cạnh.
2) Gốc uracil (U) là một phần của uridine-5’-phosphate. Ngoài uracil, các
gốc còn lại của hai pyrimidine base có thể được tìm thấy trong các
nucleic acid. Các gốc này là thymine (T), cytosine (C). Các nucleic acid
chứa U, T, C thuộc loại nào?

301 | Bản quyền thuộc về Tạp chí Olympiad Hóa học KEM
Hướng dẫn
1)

2) U chỉ có trong RNA; T chỉ có trong DNA; C có trong cả DNA và RNA.

302 | Bản quyền thuộc về Tạp chí Olympiad Hóa học KEM
Bài 6
Vấn đề này liên quan đến HI-Virus, thuộc nhóm retroviruses. Một
retrovirus sử dụng RNA làm thông tin di truyền. Tất cả các virus đều đặc
biệt để có thể sinh sản trong tế bào chủ. Vì vậy, rất cần thiết để sao chép
RNA của virus sang DNA. Virus cung cấp các enzyme cho quá trình phiên
mã. Vấn đề này liên quan đến một loại thuốc kháng virus, ngăn ngừa sự
sinh sản của virus trong tế bào chủ.
Sử dụng các công thức sau để hoàn thành các bài tập.

1) Nêu tên ba thành phần cấu tạo một nucleotide.


2) Vẽ cấu trúc của một 3-Deoxy-“purine nucleotide”.
3) Viết phương trình cho sự chuyển hóa (2S, 3R, 4 R)-2,3,4,5-
Tetrahydroxypentanal thành α-D-Arabinofuranose. Biểu diễn chất
phản ứng bằng công thức chiếu Fischer và sản phẩm bằng công thức
chiếu Haworth.
4) Phản ứng chuyển hóa từ dạng mạch hở sang mạch vòng thuộc loại
nào?
Phân tích thành phần nguyên tố trong nucleoside A cho kết quả như sau
(w/w):
44.I % C 4.9 % H 26.I % O 17.I % N 7.7 % F
Nucleoside này chứa một base thế fluorine. Chú ý rằng có một liên kết C-
F thay cho một liên kết C-H. Nguyên tử fluorine có khoảng cách tối đa với
liên kết N-glycosidic. Nucleoside thay thế này có khả năng hình thành
phosphoric acid diester, như bình thường.
5) Tính công thức tổng quát của A.
6) Vẽ công thức cấu tạo của A.

303 | Bản quyền thuộc về Tạp chí Olympiad Hóa học KEM
Bắt đầu với nucleoside A sơ đồ phản ứng sau thể hiện quy trình tổng hợp
hợp kháng virus G.
Đó là những thay đổi nhỏ về cấu trúc của A, duy nhất gây nên hoạt tính
kháng virus của G. Cho rằng các bước cần thiết để phản ứng chọn lọc ở
một nhóm chức, các nhóm chức khác cần được bảo vệ.

Lưu ý:
• Ph = phenyl
• Kết hợp sự cản trở không gian của xúc tác (Ph)3CCl trong phản ứng
B tạo ra C.
• Bước chuyển E thành F không tuân theo quy tắc Saytzeff.
• Phổ khối lượng của sản phẩm G dẫn đến các ion phân tử m/z (M+.)
7) Viết công thức cấu tạo của các hợp chất từ B đến G. Viết tắt Phenyl là
Ph như thông thường. Nếu base không tham gia phản ứng thì kí hiệu
là ‘base’. Sử dụng nucleoside Guanine và một đường phổ biến có một
nhóm hydroxyl sơ cấp nếu bạn không thể tìm ra công thức của A.
Các chất như G thể hiện đặc điểm cấu trúc đặc biệt. Điều này gây ra các
hoạt động ở mức độ phân tử tương ứng như một chất kháng virus.
8) Viết Đ (cho phát biểu đúng) và S (cho phát biểu sai) vào các ô trống.
Cách điền ngẫu nhiên sẽ bị trừ điểm.
A. Sự vắng mặt của nhóm 2´ hydroxyl gây ra các hoạt động ở mức độ
phân tử.
B. Các nucleoside tương tự G cạnh tranh với dCTP.
C. Các nucleoside tương tự G cạnh tranh với CTP.
D. Các nucleoside tương tự G cạnh tranh với TTP.
E. Chấm dứt sự kéo dài chuỗi DNA là yếu tố quan trọng của tác động
ức chế.

304 | Bản quyền thuộc về Tạp chí Olympiad Hóa học KEM
F. Việc phiên mã ngược một DNA phụ thuộc vào DNA polymerase.
Enzyme này sử dụng sai nucleotide cho phiên mã.
G. Việc phiên mã ngược một RNA phụ thuộc vào DNA polymerase.
Enzyme này sử dụng sai nucleotide cho phiên mã.
Hướng dẫn
1. base hữu cơ dị vòng; saccharide (thường là pentose), phosphoric acid
2.

3.

4. Cộng nucleophile (AN).


5. C3.67H4.90O1.63N1.22F0.41 → C9H12O4N3F
6.

305 | Bản quyền thuộc về Tạp chí Olympiad Hóa học KEM
7.
B: C:

E:
D:

F: G:

306 | Bản quyền thuộc về Tạp chí Olympiad Hóa học KEM
8. Đúng : A, C, D, F; Sai: B, G

307 | Bản quyền thuộc về Tạp chí Olympiad Hóa học KEM
Bài 7
Codycepin (E) được tách ra từ nấm đông trùng hạ thảo có tác dụng ngăn
ngừa ung thư do trong quá trình tổng hợp AND tế bào ung thư nhầm lẫn
adenosin với codycepin, vì thế tế bào có nguy cơ ung thư sẽ chết.
Codycepin được tổng hợp theo sơ đồ sau:

a) Xác định cấu trúc các chất A, B, C, D và cordycepin, biết enzym


reductase có vai trò khử hóa.
b) Dưới xúc tác của enzym adenosin deaminaza, codycepin chuyển
thành deoxynosin F (C10H14N4O4) không còn hoạt tính. Dưới xúc tác
của enzym transaminaza, deoxynosin F chuyển về codycepin. Cho biết
cấu tạo của F và mô tả cơ chế chuyển hóa E thành F.
c) Mô tả cấu trúc một đoạn ADN lỗi được hình thành từ 1 phân tử
cordycepin và 1 phân tử thymidin (liên kết qua este của axit photphoric
giống trong chuỗi ADN bình thường).

308 | Bản quyền thuộc về Tạp chí Olympiad Hóa học KEM
Hướng dẫn
Xác định cấu trúc các chất A, B, C, D và codycepin (E).

b) Cho biết cấu tạo của F và cơ chế chuyển hóa E thành F.

c) Mô tả cấu trúc đoạn ADN lỗi

309 | Bản quyền thuộc về Tạp chí Olympiad Hóa học KEM
Bài 8
Các hydrocarbon là những nguồn nguyên liệu thô rất quan trọng. Cách
thức hình thành nguồn tích trữ chúng vẫn đang là đề tài tranh cãi trong
cộng đồng học thuật. Các vi sinh khuật (vi khuẩn cổ Methanococcus), có
thể tổng hợp methane từ carbon dioxide và hydrogen, đã được tìm thấy
gần đây.
1) Viết phương trình phản ứng tổng của quá trình tổng hợp methane bởi
vi khuẩn cổ.
Trong thực tế, tổng hợp này là một quá trình nhiều giai đoạn, diễn ra theo
sơ đồ sau:

(Q - coenzyme; R, Y, Z - những phần không biến đổi trong phân tử)


2) Xác định công thức cấu tạo của S1, S3 và số oxide hoá của các nguyên
tử carbon có nguồn gốc từ carbon dioxide.
Dưới đây là mảnh quan trọng trong cấu trúc Y2N-H

3) Xác định công thức cấu tạo của S4.


Methane là khí nhà kính nguy hiểm, mỗi năm có 6.00∙1014 gam methane
phát thải vào khí quyển, 70 % trong đó có nguồn gốc hữu sinh.

310 | Bản quyền thuộc về Tạp chí Olympiad Hóa học KEM
4) Giả sử rằng methane tạo thành một lớp vỏ đặc, bao quanh Trái đất ở
độ cao xác định trong điều kiện áp suất 0.50 bar và nhiệt độ -20 oC.
Tính độ dày tăng lên hàng năm của lớp vỏ. Cho biết bán kính Trái đất
là 6370 km.
Hợp chất X đóng vai trò chủ yếu (ước lượng cỡ 40 - 70 %) gây nên hiệu
ứng nhà kính trên Trái đất. X có thể nhận được từ phản ứng của hai đơn
chất. X bị hấp thụ hoàn thoàn bởi dung dịch NaOH 0.1 M, khi dẫn X qua
than nung đỏ tạo thành chất khí có tỉ khối so với helium bé hơn 5.
5) Xác định X và viết các phương trình đã đề cập ở trên.

311 | Bản quyền thuộc về Tạp chí Olympiad Hóa học KEM
Hướng dẫn
1) CO2 + 4H2 = CH4 + 2H2O
Trong thực tế, tổng hợp này là một quá trình nhiều giai đoạn, diễn ra theo
sơ đồ sau:

(Q - coenzyme; R, Y, Z - những phần không biến đổi trong phân tử)


2)

Số oxid hóa của C tạo thành từ CO2 là +2 trong S3 và S4, 0 trong S5, -2
trong S6.
3)

312 | Bản quyền thuộc về Tạp chí Olympiad Hóa học KEM
4) Khoảng 3.1∙10-3 m
5) X là hơi nước.
2H2 + O2 → 2H2O (1)
С + H2O → CO + H2 (2)

313 | Bản quyền thuộc về Tạp chí Olympiad Hóa học KEM
Bài 9
Các deoxyribonucleic acid (DNA), môi trường chứa thông tin di truyền có
cấu trúc polymer được tạo thành từ các monomer nucleotide. Thành phần
nucleotide trong DNA bao gồm ba phần: purine hay pyrimidine base; 2’-
desoxy-D-ribose và orthophosphoric acid. Nitrogen base liên kết với
nguyên tử carbon 1’ của pentose bằng liên kết N-glycoside.
Phosphoric acid gắn với carbon 5’ của pentose bằng liên kết ester. Trong
DNA có 4 loại nitrogen base. 2 trong số chúng (adenine và guanine) thuộc
nhóm purine và 2 chất lại (thymine và cytosine) là các pyrimidine. Trong
trường hợp các purine nucleotide, để tạo liên kết với gốc pentose thì phải
dùng nguyên tử ở vị trí số 9 của purine; còn trong trường hợp của
pyrimidine nucleotide là nguyên tử số 1. Công thức của các base và
desoxyribose ở dạng công thức chiếu Fischer được cho dưới đây:

Trong DNA, các nucleotide liên kết với nhau qua cầu phosphate. Nhóm
hydroxyl 5’ của pentose thuộc một nucleotide sẽ gắn với nhóm hydroxyl 3’
của pentose thuộc nucleotide khác qua một “cầu” là gốc orthophosphoric
acid. Như vậy chuỗi polynucleotide sẽ có hai đầu 5’ và 3’.
1) Vẽ cấu trúc dạng vòng của D-ribose và 2’-desoxy-D-ribose.
2) Vẽ cấu trúc của mảnh 3-nucleotide DNA 5’-ATG-3’ (A, T, G là kí hiệu
của các đơn vị nucleotide chứa adenine, thymine, guanine. Vẽ gốc
pentose ở dạng vòng.
Trong các điều kiện sinh lý, DNA tồn tại ở dạng chuỗi xoắn kép trong đó
các base ở bên trong chuỗi còn các gốc phosphoric acid thì ở bên ngoài
chuỗi. Các mạch DNA định hướng ngược nhau và duy trì dạng xoắn kép
bởi liên kết hydrogen giữa các base. Erwin Chargraff chỉ ra có hai cặp liên
kết hydrogen trong phân tử DNA: A-T và G-C.
3) Dựa vào quy tắc của Chargaff, hãy chọn ra những tương quan đúng
▪ A+T=G+C

314 | Bản quyền thuộc về Tạp chí Olympiad Hóa học KEM
▪ A+G=T+C
▪ A+C=T+G
▪ A/T = G/C
▪ A/G = T/C
4) Trong các cặp A-T và G-C, số liên kết hydrogen đạt giá trị lớn nhất có
thể có. Vẽ cấu trúc các cặp A-T và G-X trong chuỗi xoắn này và biểu
diễn các liên kết hydrogen bằng đường nét đứt.
Tái tạo là quá trình nhân đôi các phân tử DNA trước khi phân bào. Trong
quá trình này, mỗi dải của phân tử DNA ban đầu (phân tử mẹ) đóng vai trò
như một khuôn để tổng hợp nên chuỗi mới (chuỗi con). Quá trình này tuân
theo quy tắc của Chargraff.
Có hai cơ chế khả thi của quá trình tái tạo đã được thảo luận:
a) Bảo toàn: kết quả của quá trình tái tạo chính là một chuỗi xoắn kép
chứa hai dải gốc và một chuối khác chứa hai dải sao chép.
b) Bán bảo toàn: mỗi một trong hai chuỗi xoắn kép chứa một dải gốc
và một dải sao chép.
Câu trả lời đã được tìm thấy bằng cách sử dụng phương pháp ly tâm cân
bằng mẫu DNA trong CsCl gradient1. Các DNA nó sẽ lắng xuống theo tỷ
trọng. Phương pháp này cho phép phân biệt theo khối lượng riêng các
chuỗi xoắn kép của phân tử DNA chứa đồng vị 14N (bên trái) và 15N (bên
phải) (xem hình dưới)

Trong một thí nghiệm, vi khuẩn được nuôi cấy khi có mặt 15N. Kết quả là
các phân tử DNA được làm tăng hàm lượng đồng vị này. Ngay tại một thời
điểm xác định, vi khuẩn được chuyển vào môi trường chỉ chứa 14N và sau

1
ly tâm này tạo nên một gradient tỉ trọng, tăng dần từ miệng đến đáy.
315 | Bản quyền thuộc về Tạp chí Olympiad Hóa học KEM
quá trình sinh sản vi khuẩn, các chuỗi xoắn kép DNA được đưa vào ly tâm
cân bằng. Kết quả là cơ chế bán bảo toàn đã được xác thực.
5) Vẽ các dải được tìm thấy trong ống nghiệm khi nghiên cứu DNA trích
từ
a) Thế hệ vi khuẩn thứ nhất;
b) Thế hệ vi khuẩn thứ hai.
Hướng dẫn
1)

2)

3) Các cặp 2, 4, 5.
4)

5)

316 | Bản quyền thuộc về Tạp chí Olympiad Hóa học KEM
317 | Bản quyền thuộc về Tạp chí Olympiad Hóa học KEM
Bài 10
Tính acid của các acid hai chức được mô tả bởi mô hình sau:

Các quá trình diễn ra


Quá trình ΔSi ΔHi
Giai đoạn phân li thứ -Rln(Ω1/Ω0) + const1 ΔHdiss (ΔHdiss =
nhất const2)
Giai đoạn phân li thứ -Rln(Ω2/Ω1) + const1 ΔHdiss + ΔHcoulo
hai
ΔHcoulo = NAe2/(4
πεaqε0r)
Mô tả nhiệt động lực học
Trong đó Ωj là số trạng thái tương đương của các phân tử (ion) với điện
tích bằng -j; NA = 6.022·1023 mol-1; ε0 = 8.85·10-12 F/m; εaq = 83; e = 1.6·10-
19
C; π ≈ 3.1416; R = 8.314 J/(mol·K).
ΔGi = -RTln(Ki) = ΔHi - TΔSi
Hãy xét một acid hai chức không có tương tác giữa các nhóm chức acid,
nghĩa là trường hợp khoảng cách r vô cùng lớn (r → ∞).
1) Tính giá trị chính xác của tỉ lệ K1/K2 khi r → ∞, chỉ xét đến ΔS1 và ΔS2
tương ứng với hai trạng thái: 1 và 2 nhóm chức acid bị phân li.
2) Tính khoảng cách giữa các điện tích dương trong anion S2- ở 25 oC,
nếu biết tỉ lệ K1/K2 của H2S bằng 107; có tính đến các thành phần
entropy (ý 1) và enthalpy (điều kiện) của ΔG.
3) Xác định cấu tạo của hợp chất C4H2O4, có tỉ lệ K1/K2 nhỏ hơn giá trị
tính được trong ý 1. Vẽ cấu tạo các anion.
4) Các đồng phân cis- và trans- ứng với công thức C4H4O4 có tỉ lệ K1/K2
lần lượt là 10-4.31 và 10-1.36. Hãy vẽ cấu tạo dạng phân li của các acid
đồng phân này. Chỉ ra những đặc trưng cấu trúc, gây ra sự khác biệt
đáng kể giữa (K1/K2)cis và (K1/K2)trans.

318 | Bản quyền thuộc về Tạp chí Olympiad Hóa học KEM
5) Các dimethyl ether của các acid này bị thủy phân trong kiềm theo hai-
giai đoạn bất thuận nghịch, với hằng số tốc độ là k1 và k2. Giá trị
ln(k1/k2) với các đồng phân cis và trans ngược dấu nhau. Trình bày cơ
chế thủy phân các đồng phân để giải thích cho sự ngược dấu ln(k1/k2).
6) Đồ thị dưới đây là đường cong chuẩn độ acid-base của DNA chứa các
đơn vị C-G.

a) Giải thích độ trễ của các đường cong chuẩn độ acid và base.
b) Đường cong nào ứng với sự chuẩn độ dạng acid của DNA và đường
cong nào của dạng base?
c) Sẽ có thay đổi nào xảy ra với: i) diện tích vòng trễ S; ii) vị trí các
đường cong A, B - khi thay thế tất cả các cặp C-G trong DNA ban
đầu bởi các cặp A-T (tương tác giữa các cặp C-G gồm 3 liên kết
hydrogen, còn cặp A-T là 2).
Hướng dẫn
1) Các dạng tương đương: Ω0 = 1; Ω1 = 2; Ω2 =1
Từ các công thức cho ở trên, ta có:
ΔS2 - ΔS1 = Rln(Ω0Ω2/Ω12) = -RTln4
nhưng -RTln(K2/K1) = -TΔS12, do (ΔH12)∞ = 0 nên (K1/K2)entropy = 4.
2) ΔG12 = -RTln(K1/K2) = ΔH12 - TΔS12, sử dụng giá trị ΔS2 - ΔS1 tính được
ở trên:
ΔH12 = -RTln(K1/4K2)
Nhưng ΔH12 = -(NAe2)/(4πεaqε0r)
Do đó r = (NAe2)/[4πεaqε0RTln(K1/4K2)] ≈ 0.469 Å
319 | Bản quyền thuộc về Tạp chí Olympiad Hóa học KEM
3)
4)

Trong Z-monoanion, giá trị ΔH12 tăng do sự xuất hiện của liên kết hydrogen
(kết hợp với các tính toán như ở ý 1, 2 để giải thích.)
5) Sự khác biệt về động học các quá trình thủy phân trong kiềm có liên
quan tới khả năng xảy ra sự thế nucleophile liên phân tử với sự trợ giúp
của nhóm kề của đồng phân Z , khi k2 chủ yếu được quyết định bởi tốc độ
thủy phân maleic anhydride. Đồng phân E bị thủy phân ở giai đoạn thứ hai
qua diaion, làm chậm đáng kể quá trình.

6) a) Độ trễ của các đường cong liên quan đến quá trình phân li của acid
dianion. Tại thời điểm này, xảy ra sự phá vỡ các liên kết hydrogen giữa các
nitrogen base. Các phản ứng acid-base diễn ra nhanh hơn phản ứng phân
li và kết hợp, do đó đường cong bị trễ.
b) Đường cong A tương ứng với sự chuẩn độ muối DNA bằng acid, đường
cong B tương ứng với sự chuẩn độ bằng base.
c) Do 3 liên kết hydrogen bị thay thế bởi 2 liên kết hydrogen nên tổng điện
tích cần để phá vỡ liên kết sẽ giảm. Cả hai điểm tương đương sẽ dịch
chuyển về môi trường có tính acid hơn. Vùng trễ sẽ giảm.

320 | Bản quyền thuộc về Tạp chí Olympiad Hóa học KEM
321 | Bản quyền thuộc về Tạp chí Olympiad Hóa học KEM
Bài 11
Nitrogen là nguyên tố quan trọng bậc nhất trong các hệ số và cố định đạm
(cố định nitrogen) khí quyển là một vấn đề then chốt trong phát triển hóa
học. Trong công nghiệp, các quá trình như vậy diễn ra trong điều kiện khắc
nghiệt, tốn nhiều năng lượng và nguồn lực. Trong tự nhiên, sự cố định đạm
diễn ra trong những cơn dông hoặc được thực hiện bởi tác động sống của
các vi khuẩn.
1) Viết các phương trình phản ứng kèm theo điều kiện cho quá trình cố
định đạm trong: a) công nghiệp và b) tự nhiên.
2) Lượng N2 được cố định bởi vi khuẩn trong đất là khoảng 1.8∙1011 kg
mỗi năm (lớn hơn toàn ngành công nghiệp!). Hãy ước lượng số mol
nitrogen nguyên tử đi vào mỗi m2 đất mỗi tháng, biết bán kính Trái đất
là 6400 km.
Cố định đạm diễn ra với hệ nitrogenase enzyme (tâm hoạt tính chứa các
nguyên tử Mo và Fe) trong điều kiện kị khí.

3) Xác định công thức các tiểu phân X1 - X4.


4) Tiềm năng cố định đạm của các tế bào được xác định bởi khả năng
khử hydrocarbon khí (trong điều kiện thường) Y1 thành Y2. Xác định
Y1, Y2 nếu biết hàm lượng của carbon trong Y2 ít hơn 1.077 lần so với
Y1.
Hệ nitrogenase cũng có khả năng khử các anion độc với cơ thể ngời theo
phản ứng sau: X5 + 2e- + 4H+ → X6 + X4. X6 cũng bị khử bởi các enzyme
và chỉ tạo thành X4. Xác định X5, X6.
Các vi khuẩn thuộc chi Nitrosomonas có thể sử dụng các hợp chất
nitrogen để cung cấp năng lượng cho tế bào:

322 | Bản quyền thuộc về Tạp chí Olympiad Hóa học KEM
(Trong tiếng Nga, B là V, kí hiệu của volt)
5) Xác định Z1 - Z3 và tính năng lượng tự do của chuyển hóa NH4+ → Z3
(sử dụng biểu thức r G = −F nE 0
i i ). Chú ý rằng Z2 là chất kém bền.

Hướng dẫn
1) a) N2 + 3H2 2NH3 (500 oC, 30 MPa, xúc tác Fe)

b) N2 + O2 → 2NO (sét đánh)


2) 0.014 mol nguyên tử.
3) X1 - N2H2, X2 - N2H4, X3 - NH3, X4 - NH4+.
4) Do hệ nitrogenase thực hiện phản ứng khử 2-electron, phản ứng có thể
được viết ở dạng: CxH2y + H2 → CxH2y+2. Khi đó:
12.01x + 1.01  (2y + 2)
1.077 =  12x + y = 26.0 - do vậy hai hydrocarbon
12.01x + 1.01  2y
khí là Y1 và Y2 là acetylene và ethylene.
5) Do X6 bị khử sẽ tạo thành chỉ NH4+, nên tiểu phân này có thể chỉ chứa
nitrogen (và, có thể, là cả hydrogen - có nghĩa, nó là một phần của chu kì
được xét trước đó). Hơn nữa, theo phương trình thì so với X5, X6 có chứa
ít hơn 1 nguyên tử nitrogen và điện tích cao hơn 1 giá trị. Khả năng duy
nhất là X5 - N3- và X6 - N2.
6) Z1 - NH2OH, Z2 - NOH, Z3 - NO2-.

rG = -FnE
i i = -96500  ( 2  0.899 + 4  0.069 ) = -2  10 kJ/mol
5

323 | Bản quyền thuộc về Tạp chí Olympiad Hóa học KEM
Bài 12
Trong cơ thể thực vật, ankaloit được sinh tổng hợp từ các amino axit dưới
tác dụng của enzym. Con đường sinh tổng hợp norcoclaurin từ L-tyrosin
(G) dưới tác dụng của 4 loại enzym E1, E2, E3, E4 (trong đó có enzym
decarboxylaza) theo sơ đồ sau:

a) Trên cơ sở vai trò của các enzym, xác định công thức cấu tạo của các
chất H, I, K và L.
b) Thực vật họ Cactaceae chứa ankaloit Mescalin. Con đường sinh tổng
hợp Mescalin theo sơ đồ sau

Trong đó quá trình metyl hóa được thực hiện bởi tác nhân sinh học SAM
(S-adenosylmethionin có vai trò giống như tác nhân hóa học MeI/Ag2O).
Xác định cấu tạo của các chất M, N, P, Q, S và mescalin
c) Norcoclaurin còn được chuyển hóa thứ cấp tạo thành papaverin. Viết
sơ đồ mô tả quá trình sinh hóa chuyển norcoclaurin thành papaverin.

d) Papaverin có tác dụng làm giảm sự co thắt của cơ trơn theo cơ chế ức
chế enzym chuyển hóa nucleotit X1 thành X2. Trong tế bào X1 tồn tại
ở dạng monoanion (photpho chiếm 9,44% khối lượng), X2 tồn tại ở
dạng dianion (photpho chiếm 8,98% khối lượng). Nucleotit X1 được
hình thành theo con đường thủy phân ribonucleotit triphotphat.
Xác định cấu trúc của X1 và X2, biết trong phân tử X1 chứa bazơ nitơ
điển hình và không chứa vòng lớn hơn 6 cạnh.

324 | Bản quyền thuộc về Tạp chí Olympiad Hóa học KEM
Cho biết cấu trúc các bazơ nitơ và nucleotit điển hình sau:

Z: bazơ nitơ

Hướng dẫn X: H, OH

a) Công thức cấu tạo các chất H, I, K và L


Để hình thành norcoclaurin phải có hai giai đoạn decarboxyl hóa, vì vậy
enzym E1 xúc tác cho phản ứng decarboxyl hóa, E3 là enzym xúc tác cho
phản ứng oxy hóa để đưa nhóm OH vào vị trí ortho so với nhóm -OH ban
đầu của vòng thơm. Enzym E2 xúc tác cho phản ứng deamin hóa do sản
phẩm norcoclaurin cuối cùng chỉ chứa một nguyên tử N. Enzym E4 xúc
tác cho phản ứng đóng vòng imin. Từ đó xác định được cấu tạo:

b) Cấu tạo các chất M, N, P, Q, S và mescalin

c) Sinh chuyển hóa norcoclaurin thành papaverin

d) Một nucleotit điển hình có dạng dianion như sau:

325 | Bản quyền thuộc về Tạp chí Olympiad Hóa học KEM
Z: bazơ nitơ
X: H, OH

MX1 = (31 x 100)/9,44 = 328


MX2 = (31 x 100)/8,98 = 345
Khối lượng phân tử của X1 và X2 chênh nhau 17 g/mol chứng tỏ có sự
loại bỏ một nhóm -OH
X2 → X1 + HO-
Vì vậy X2 chuyển hóa sang X1 là quá trình đóng vòng nội phân tử, chuyển
từ X2 từ dạng dianion sang X1 ở dạng monoanion. Trong tế bào X1 hình
thành theo con đường thủy phân ribonucleotit triphotphat X3 chứng tỏ X1
và X2 đều chứa hợp phần đường ribozơ (X = OH) chứ không phải hợp phần
dehydroribozơ.
Khối lượng phân tử của bazơ nitơ được xác định như sau:
M(bazơ nitơ) = 31x100/9,44 – M(ribozơ) + 1 = 135. Do đó bazơ nitơ Z là
adenin.
Do trong X1 còn 1 nhóm -OH tự do nên xảy ra hai trường hợp nucleotit
chứa vòng 3,5’-monophotphat hoặc 2,5’-monophotphat. Tuy nhiên theo
bài X1 không chứa vòng nhiều hơn 6 cạnh nên X1 chứa vòng 6 cạnh ở vị
trí 3,5’-monophotphat (dạng 2’,5-monophotphat tồn tại ở vòng 7 cạnh).

326 | Bản quyền thuộc về Tạp chí Olympiad Hóa học KEM
Bài 13
Sự thế đồng vị gây ảnh hưởng đến các tính chất lí-hóa của các phân tử và
thường được áp dụng để đánh giá các cân bằng và nghiên cứu cơ chế
phản ứng. Bài tập này liên quan đến ứng dụng của kĩ thuật này đến việc
đánh giá sự chuyển các hydrogen ion và tương tác giữa các nucleotide
base của adenine (A) và uracil (U) trong ribonucleic acid (RNA) (hình 1).

Hình 1: Cấu tạo của các nucleotide base A và U gắn với một đơn vị
ribose-phosphate (R).
Một mô hình phổ biến được sử dụng để đánh giá tương tác giữa các chuỗi
nucleic acid được tạo thành bởi các chuỗi thuần nhất của RNA kiểu poly(A)
và poly(U); 30 nucleotide trong mỗi nucleic acid. Tương tác này bị nhiễu
loạn bởi độ pH của môi trường và chịu sự khống chế của các cân bằng
acid-base (biết các giá trị pKa của adenine và uracil lần lượt là 3.5 và 9.2).
1.1) Viết các cân bằng acid-base đối với adenine và uracil, biểu diễn cấu
tạo của mỗi tiểu phân.
1.2) Dựa vào các giá trị pKa của adenine và uracil, hãy tính xem tại pH nào
tương tác giữa các chuỗi poly(A) và poly(U) có thể đạt cực đại?
Các phép đo nhiệt độ nóng chảy (Tm) là phương pháp tiêu chuẩn để ước
lượng tương tác giữa 2 chuỗi nucleic acid. Các thí nghiệm này được thực
hiện bằng cách xác định độ hấp thụ của dung dịch nucleic acid ở dạng
hàm của nhiệt độ (hình 2).

327 | Bản quyền thuộc về Tạp chí Olympiad Hóa học KEM
Hình 2: Các đường cong để xác định nhiệt độ nóng chảy của một hỗn
hợp đẳng mol poly(A) và poly(U). “% Transition” chỉ mức độ diễn tiến của
điều kiện cân bằng giữa hai trạng thái. Nồng độ poly(A) là 1.96 μM, pH
7.0 (hệ đệm phosphate 5 mM) trong các dung dịch khác nhau: (a) 100%
H2O; (b) 25% D2O trong H2O; (c) 75% D2O trong H2O; (d) 100% D2O.
(Absorbance at 260 nm = Độ hấp thụ ở 260 nm.)
Sự biến đổi độ hấp thụ phụ thuộc vào nồng độ của tiểu phân trong môi
trường, độ dài quang trình (độ dài cuvette) và độ hấp thụ mol (ε) của các
tiểu phân trong môi trường. Độ hấp thụ mol của adenine nucleotide (nctA)
trên một chuỗi đơn poly(A) là ε/nctA = 15.4 cm-1/mM ở 260 nm. Dựa vào
hình 2 - trong đó cho thấy các đường cong được sử dụng để xác định nhiệt
độ nóng chảy của một hỗn hợp đẳng mol poly(A) và poly(U); mỗi nucleic
acid có 30 nucleotide. Nồng độ của poly(A) được sử dụng trong thí nghiệm
là 1.96 μM và độ dài cuvette là 1 cm.
2.1) Chọn giá trị độ hấp thụ quang phù hợp nhất trên đồ thị và tính độ hấp
thụ quang của poly(U). Bỏ qua các tính toán về ảnh hưởng của nhiệt độ
đến độ hấp thụ mol.

328 | Bản quyền thuộc về Tạp chí Olympiad Hóa học KEM
2.2) Tính lượng ánh sáng truyền qua theo % với chuỗi kép poly(A)·poly(U)
dưới các điều kiện cho trong hình 2.
Như đã biểu diễn trong hình 2, nhiệt độ nóng chảy (Tm) phụ thuộc vào
nồng độ của D2O trong H2O.
3.1) Xây dựng giản đồ năng lượng phù hợp với quan sát ở trên. Chỉ rõ trong
giản đồ các mức năng lượng liên quan đến sự các bước chuyển trong H2O
và D2O.
Bảng 1 cung cấp dữ kiện δTm ở các nồng độ khác nhau của K+. Chú ý rằng
δTm = TmD - TmH, trong đó D và H chỉ các phép đo được thực hiện lần lượt
trong điều kiện 100 % H2O và 100 % D2O.
Bảng 1: Sự phụ thuộc của δTf (oC) vào các nồng độ mol khác nhau của
potassium ion.

4.1) Dựa vào dữ kiện trong bảng 1, hãy tính chênh lệch năng lượng
( Go
H2 O )
-GDo2O theo J·phân tử-1 ở nồng độ K+ thấp nhất và cao nhất.

4.2) Xác định tỉ lệ các hằng số lực trong các điều kiện nồng độ K+ thấp
nhất và cao nhất.
Việc đun nóng adenine và các dẫn xuất của nó trong D2O ở pH > 10 sẽ
thúc đẩy sự thay thế hydrogen ở vị trí 8 bởi deuterium. Phản ứng này thuận
nghịch nếu các hợp chất được làm giàu đồng vị được chuyển vào dung
dịch chỉ có nước.
5.1) Dựa vào phát biểu ở trên, trình bày cơ chế theo từng bước của phản
ứng làm giàu đồng vị của adenine bởi deuterium. Chỉ rõ cấu tạo của tất cả
các tiểu phân và hợp chất trung gian trong phản ứng.
Hợp chất được làm giàu có thể nhận được theo các tương tự với tritium ở
vị trí 8. Hằng số tốc độ của sự thay thế tritium bởi hydrogen ion là 3.18·10-
3 -1
h ở pH 11, tại 37 oC.

329 | Bản quyền thuộc về Tạp chí Olympiad Hóa học KEM
6.1) Biết rằng tần số kéo căng của liên kết C-H là 3000 cm-1, hãy ước tính
hằng số tốc độ của sự thay thế deuterium bởi hydrogen ion trong điều kiện
tương tự.

330 | Bản quyền thuộc về Tạp chí Olympiad Hóa học KEM
Hướng dẫn
1.1)

1.2)

2.1)

2.2)

331 | Bản quyền thuộc về Tạp chí Olympiad Hóa học KEM
3.1)

(Distância = khoảng cách)


4.1)

Nồng độ thấp nhất và cao nhất trong bảng dưới đây:

4.2)

332 | Bản quyền thuộc về Tạp chí Olympiad Hóa học KEM
(baixa = thấp; alta = cao)
5.1)

333 | Bản quyền thuộc về Tạp chí Olympiad Hóa học KEM
6.1)

334 | Bản quyền thuộc về Tạp chí Olympiad Hóa học KEM
Bài 14
Tổng hợp nền (matrix synthesis) là cơ chế chung, được công nhận cho
quá trình sinh tổng hợp protein. Trong quá trình dịch mã, sự cộng thêm 1
gốc amino acid khác được mã hoá bởi bộ ba nucleotide của RNA thông
tin. Tuy nhiên, một hướng tổng hợp thay thế (không liên quan đến các
nucleic acid) đã được tìm thấy trong nhiều olygopeptide. Hướng này gồm
một số phản ứng enzyme liên tiếp. Phức enzyme của chủng Bacillus brevis
cho phép tổng hợp kháng sinh gramicidin S - một decapeptide vòng. Phân
tích gramicidin cho thấy nó được tạo thành bởi những lượng bằng nhau
của các α-amino acid sau:
amino L-valine L- L- L-proline D-
acid leucine ornithine phenylalanine
Kí hiệu Val Leu Orn Pro D-Phe
M, 117 131 132 115 165
gam/mol
Quá trình sinh tổng hợp bắt đầu với sự racemic hoá L-phenylalanine (2-
amino-3-phenylpropanoic acid).
1) Vẽ công thức chiếu Fischer của cả hai dạng đồng phân quang học của
phenylalanine và cho biết cấu hình tuyệt đối của các nguyên tử 𝛼-
carbon.
Giai đoạn tiếp theo là hoạt hoá các amino acid bằng tương tác với
adenosine triphosphate (ATP) theo sơ đồ sau:
𝛼-amino acid + ATP → 𝛼-aminoacyl adenylate + PPi
tron đó PPi là pyrophosphate vô cơ.
2) Vẽ công thức của aminoacyl adenylate dựa vào công thức của adenine
dưới đây:

Giai đoạn tiếp theo, nhóm aminoacyl được chuyển tới nhóm thiol của gốc
cystein (thuộc về enzyme xúc tác cho phản ứng).
335 | Bản quyền thuộc về Tạp chí Olympiad Hóa học KEM
3) Viết phương trình phản ứng diễn ra. Kí hiệu enzyme tự do là:

Gaminoacyl được chuyển tới nhóm thiol của protein mang aminoacyl. Sau
đó, sự ghép cặp của hai gốc được hoạt hoá khác nhau liên kết với các
phân tử của protein mang aminoacyl diễn ra.
4) Viết sơ đồ tạo thành liên kết peptide. Sử dụng kí hiệu R1 và R2 cho các
nhóm thế ở 𝛼-carbon; kí hiệu protein mang aminoacyl là

Sau chuỗi phản ứng được mô tả ở ý 4 là sự ngưng tụ của hai pentapeptide


giống hệt nhau theo kiểu “đầu nối đuôi”, tạo thành gramicidin S.
5) Xác định cấu tạo của gramicidin S, nếu biết khi thuỷ phân không hoàn
toàn thì thu được sản phẩm có chứa các mảnh với khối lượng phân tử
278, 262 và 245. Proline được tìm thấy ở đầu N của mảnh dipeptide
với khối lượng phân tử nhỏ nhất có thể có. Sử dụng kí hiệu 3-chữ cái
của các amino acid khi vẽ cấu tạo gramicidin S. Chú ý, trong cấu tạo
peptide vòng, hướng từ đầu N đến đầu C của các gốc amino acid là
theo chiều kim đồng hồ.
Hướng dẫn
1)

2)

336 | Bản quyền thuộc về Tạp chí Olympiad Hóa học KEM
3)

4)

5)

337 | Bản quyền thuộc về Tạp chí Olympiad Hóa học KEM
Bài 15
Sinh tổng hợp protein, được biết đến như là sự dịch mã, tiến hành ở các
ribosomes được tìm thấy trong các phức đại phân tử nhiều thành phần
có chứa các ribosom RNA và protein. Bức dịch mã đầu tiên (thường biết
đến như là bước khơi mào) bao gồm sự ráp nối của các ribosom sơ cấp
lớn và nhỏ lại với nhau cùng với RNA thông tin (mRNA) như được minh họa
ở Hình.1:

Hình 1. Sơ đồ chung của sự dịch mã trong tế bào sống


1. Amino axit bất kỳ được mã hóa bởi một codon, bộ ba nucleotit liên tiếp
nằm trên mRNA. Có bao nhiêu codon tồn tại thực sự, nếu chỉ có bốn
ribonucleotit chính được khảo sát? Tất cả các codon mã hóa amino axit
không?
2. Có thể điều khiển một chuỗi rinonucleotit đơn cho một protein với một
chuỗi amino axit đã biết hay không?
Amino axit được phân phát tới một ribosom chức năng bởi một RNA đặc
biệt nhỏ (còn gọi là RNA vận chuyển hoặc tRNA). Mỗi tRNA chỉ tương ứng
với một codon.
3. Có bao nhiêu tRNA khác nhau có thể phân phát mỗi amino axit tới
ribosom? Hãy xét leucine và methionine.
Để phân phát tới 1 ribosome, một amino axit phải tạo liên kết cộng hóa trị
với ribosom tRNA. Phản ứng này đòi hỏi năng lượng được cung cấp bởi
thủy phân ATP và được xúc tác bởi aminoaxyl-tRNA synthetase (aaRS),
một enzym đặc biệt cho mỗi một amino axit riêng biệt. Mạch cạnh của

338 | Bản quyền thuộc về Tạp chí Olympiad Hóa học KEM
amino axit gắn vào không liên quan gì đến liên kết cộng hóa trị gắn với
tRNA.
4. Viết các phương trình phản ứng được xúc tác bởi aaRS trong quá trình
nối amino axit vào tRNA. Hãy chỉ ra các nhóm của tRNA và amino axit liên
quan đến liên kết tạo thành.
5. Dùng bảng mã hóa gen viết các chuối aminoaxit cho các oligopeptit
sau:
a) Được ghi mã bởi mRNA dưới đây
b) Được ghi mã bởi mRNA dưới đây với đầu và cuối C thay bằng U
c) Được ghi mã bởi mRNA dưới đây với đầu G thay bằng C
d) Được ghi mã bởi mRNA dưới đây với cuối nhưng một G thay bằng
U
5'AUGGAUCACGCCAUCAAUGUUGUCGGUUGGAGUGUGGAUACGUUGGA
UGAUGG AACUGAAGCU3'.
6. Viết chuỗi nucleotit của mRNA mã hóa Met-Asp-Val-Asn-His-Pro-Glu-
Tyr-Gly-Lys. Dùng A, U, G và C để làm rõ vị ví quyết định, N1/N2 nếu bất kỳ
hai chuỗi nucleotit có thể ở một vị trí đặc biệt, và N nêu bất kỳ bốn chuỗi
nucleotit có thể ở vị trí đặc biệt (N1 và N2 có thể là bất ký của A, U, G và
C).
7. Khối lượng phân tử của protein E.coli là khoảng 51 kDa. Hãy ước tính
độ dài mã mRNA (theo nm, làm tròn đến số thập phân). Lấy khối lượng
phân tử trung bình của một amino axit là 110 g/mol, và chiều dài trung
bình của một ribosom là 0,34 nm. Cần mất bao nhiêu thời gian để một tế
bào tổng hợp protein này nếu ribosom đạt 20 ribonucleotit trên giây?
Một nhóm các nhà nghiên cứu thực hiện tổng hợp protein trong hệ không
có tế bào (in vitro). Tất cả các thành phần (ribosom, tRNA, ATP, GTP, muối,
aminoaxit, aaRS, hệ số dịch mã, v.v…) được cho vào hệ. Một polinucleotit
tổng hợp chỉ có chứa A và C theo tỉ lệ phản ứng 1:5 được dùng như là RNA
thông tin (nucleotit lưu giữ trữ được sắp xếp ngẫu nhiên trong mRNA).
8. Xác định thành phần aminoaxit của protein tổng hợp. Tỉ lệ giữa các
aminoaxit lưu giữ trong protein là bao nhiêu?

339 | Bản quyền thuộc về Tạp chí Olympiad Hóa học KEM
Câu trúc 3D của một tRNA được mô tả trong hình 2. Có hai miền chính:
điểm kết thúc CCA3’ mà liên kết với aminoaxit, và một anticodon tương
ứng chính xác với codon mRNA.

Hình 2. Cấu trúc 3D của một tRNA.


9. Một tRNATyr đột biến với anticodon riêng chứa bộ ba Ser (thay thế codon
Tyr) được thêm bào hệ tổng hợp ở mục 8. Sản phẩm protein là gì?
Một nhà sinh hóa chuyên ngành hóa học protein đã mô tả khám phá của
ông về một protein đột biến có Glu thành đột biến His cho phân tử di
truyền. Sau này, kết quả là khá lạ lẫm và người ta khuyên nhà sinh hóa nên
kiểm tra lại.
10. Tại sao nhà di truyền học đã không nghi ngờ gì về khả năng đột biến
trên? Đột biến nào có thể xảy ra hơn?
Hướng dẫn
340 | Bản quyền thuộc về Tạp chí Olympiad Hóa học KEM
1. Có 43 = 64 tổ hợp ba nucleotit khác nhau của 4 nucleotit. Chỉ có 61 bộ
ba mã hóa amino axit thêm vào mạch polipeptit dài. Ba tổ hợp còn lại bộ
ba DỪNG xác định sự kết thúc của quá trình dịch mã.
2. Không, bởi vì sự suy giảm của mã gen: hầu hết amino axit được mã hóa
bởi nhiều bộ ba.
3. Leucin được mã hóa bởi 6 bộ ba khác nhau, như thế nó được phân phát
đến một ribosom bởi 6 tRNA. Để được mã hóa bởi một bộ ba, methionin
được vận chuyển bởi một tRNA duy nhất. Trong một số tế bào bộ ba mã
hóa sau cũng tương ứng với sự chuyển dịch bắt đầu, amino axit N-
formylmethionin của N-amino axit kết thúc. Cũng thế, methionin và
N-formylthentionin được vận chuyển bởi tRNAs khác nhau.
4. Phương trình của các phản ứng nối tiếp là:
Amino axit +ATP=aminoaxyl adenylat+Ppi (chất vô cơ pyrophosphat) (1)
Aminoaxyl adenylat + tRNA = aminoaxyl + AMP (2)

341 | Bản quyền thuộc về Tạp chí Olympiad Hóa học KEM
Như vậy, nhóm cacboxylic của amino axit phản ứng với nhóm 3’-OH của
tRNA
5.a) Met-Asp-His-Ala-Ile-Asn-Val-Val-Gly-Trp-Ser-Val-Asp-Thr-Leu-Asp-
Asp-Gly-Thr-Glu-Ala hoặc fMet-Asp-His-Ala-Ile-Asn-Val-Val-Gly-Trp-Ser-
Val-Asp-Thr-Leu-Asp-Asp-Gly-Thr-Glu-Ala, phụ thuộc vào các tiểu phân
sinh tổng hợp (Eukaryotes, Prokaryotes, hoặc Archaea).
b) Amino axit thứ ba là tyrosin và amino axit cuối cùng là valin. Tất cả các
vị trí còn lại là như nhau.
c) Amino axit N-cuối là leucin. Tất cả các vị trí còn lại là giống nhau. Phải
chú ý rằng sự dịch mã trong vi khuẩn không bắt đầu nếu thiếu bộ ba KHỞI
ĐỘNG.
d) Cái cuối nhưng một bộ ba bị thay đổi thành bộ ba DỪNG mà sẽ cho ra
oligopeptit ngắn hơn bởi 2 amio axit lưu lại so với ở 5a.
6. AUG-GAU/C-GUN-AAU/C-CAU/C-CCN-GAA/G-UAU/C-GGN-AAA/G
7. Protein chứa 51000/110 ≈ 464 amino axit còn lại
Ở đây, nó được mã hóa bởi mRNA chứa 464*3 +3 = 1395 nucleotit còn lại
kể cả bộ ba DỪNG.
Chiều dài của mRNA là 1395 * 0,34 =474,3 ≈ 474 nm.
Thời gian cần thiết để sinh tổng hợp protein là: 1395/20 = 69,7 ≈ 90 giây
8. Ta cho rằng tỉ lệ A:C là 1:5, xác suất tìm thấy A và C ở vị trí bất kỳ là 1/6 và
5/6 tương ứng. Như thế, khả năng tìm thấy của một bộ ba condon đã cho là:
AAA = (1/6)3 = 1/216 CCC = (5/6)3 = 125/216
AAC = (1/6)2*5/6= 5/216 CCA = (5/6)2*1/6 = 25/216
ACA = 1/6*5/6*1/6 = 5/216 CAC = 5/6*1/6*5/6 = 25/216
ACC = 1/6*(5/6)2 = 25/216 CAA = 5/6*(1/6)2 = 5/216
Sử dụng bảng mã gen, tính được
Lys : Asn:Thr:Pro:His:Gln = 1:5:30:150:25:5
9. Anticodon không ảnh hưởng đến CCA3’ kết thúc. Như vậy, sự đột biến
tRNA sẽ cộng tyrosin vào các vị trí mà serin lúc đầu dự định vào chuỗi

342 | Bản quyền thuộc về Tạp chí Olympiad Hóa học KEM
mRNA. Điều này có thể dẫn đến sự cuộn không mong muốn của protein
và sự mất toàn bộ hoặc 1 phần hoạt tính chức của nó.
10. Glu được mã hóa bởi GAA và GAG, và His bởi CAU và CAC. Hai sự thế
(của phần dư đầu tiên và thứ ba) cần để gây lên sự đột biến đúng, đột biến
hoàn toàn không thể. Một sự đột biến dư xảy ra thường xuyên hơn, và đột
biến Glu đến Gln có thể xem như là ví dụ điển hình (cùng với nhiều đột biến
kểu này).

343 | Bản quyền thuộc về Tạp chí Olympiad Hóa học KEM
Bài 16
Lysozyme là một protein dạng hình cầu, nhỏ (15 kDa), được tìm thấy trong
lòng trắng trứng, nước tiểu người, sữa mẹ, nước mắt. Do chứa 4 cầu S-S,
phân tử này có thể trải qua các chuyển hóa oxid hóa-khử.
1) Viết bán phản ứng khử cầu S-S. Đánh dấu tất cả các nguyên tố thuộc
về các gốc amino acid tương ứng.
Phương pháp quét thế chu kì (quét thế vòng) được sử dụng rộng rãi để
nghiên cứu cơ chế của các phản ứng điện hóa tương tự nhau. Trong
phương pháp này, dòng điện được đo dưới dạng hàm của thế được quét
theo chu kì. Thông thường, sự phụ thuộc vào thời gian của điện thế là hàm
tuyến tính: điện thế tăng đều (tuyến tính) từ cực tiểu đến cực đại, rồi sau
đó lại giảm đều về cực tiểu. Khi bán phản ứng xảy ra, số electron được vận
chuyển tăng, cường độ tăng và các tín hiệu đặc trưng được ghi lại. Từ các
đường cong ghi lại, có thể xác định thế điện hóa của một bán phản ứng.
Một giản đồ điển hình (ghi lại được ở pH = 5.00 với điện cực giọt thủy
ngân) được cho trong hình. Các đường cong tương ứng với chu kì thứ 1, 2
và 3 được đánh số phù hợp. Điện thế được xác định tương đối với điện
cực calomel (Hg2Cl2/Hg, Eo = +0.268 V).

2) Điện cực calomel là điện cực tham chiếu, phụ trợ hay điện cực làm
việc?
3) Viết bán phản ứng xảy ra trên điện cực calomel.

344 | Bản quyền thuộc về Tạp chí Olympiad Hóa học KEM
4) Tín hiệu nào trong hình (1, 2, 3 hay 4) tương ứng với quá trình oxid hóa
(“cathode peak”) và khử hóa (“anode peak”) chất nền?
5) Giá trị vật lí nào có thể được xác định:
a) bằng cách tính trực tiếp tích phân đường cong thực nghiệm;
b) sau khi chia giá trị tính được từ ý 5a cho tốc độ quét (nghĩa là tốc
độ biến thiên điện thế)?
6) Có thể giải thích như thế nào (lỗi thực nghiệm, sự phân cực điện cực,
hoặc khử cực, hấp thu/giải hấp thu sản phẩm và/hoặc các tác nhân,
các phản ứng phụ, …) về các giá trị khác nhau nhận được với các anode
và cathode peak (mũi)?
7) Ở sự oxid hóa chất nền, điện tích Qcathode = 1.80·10-6 C được dẫn qua
bề mặt điện cực 1 cm2; ở sự khử thì điện cực Qanode = 1.72·10-6 C. Trong
trường hợp đơn giản nhất, phương trình bán phản ứng khử có thể biểu
diễn là
Protein + nH+ + ne- → ProteinHn (1)
Dẫn ra biểu thức sự phụ thuộc của n vào Qcathode và S (bề mặt hiệu
dụng, chiếm bởi 1 phân tử trên điện cực). Tính n với bán phản ứng này,
biết S = 2000 Å2, hằng số Faraday = 96500 C mol-1.
8) Phương trình (1) không thể giải thích thực tế rằng cường độ peak thay
đổi theo thời gian. Trong một loạt thí nghiệm, một điện cực tiếp xúc
với dung dịch enzyme trong một chu kì thời gian xác định t. Sự phụ
thuộc của cường độ Icathode, tương ứng với peak 1 trong đồ thị, vào thời
gian được quan sát thấy.
Viết một hệ - không ít hơn 3 phương trình của các bán phản ứng - có
thể giải thích:
a) sự phụ thuộc vào thời gian quan sát được của Icathode;
b) sự biến mất của mối liên hệ như vậy với Ianode.

345 | Bản quyền thuộc về Tạp chí Olympiad Hóa học KEM
Hướng dẫn
1)

2) Điện cực calomel là điện cực tham chiếu.


3) Hg2Cl2 + 2e- → 2Hg + 2Cl-.
4) Peak 4 là anode peak; peak 1, 2, 3 là cathode peak.
5) Chỉ rõ các đơn vị.
a) ampere·volt = watt;

ampere  volt
b) = ampere  s = coulomb
volt
s
6) Sự khác biệt có thể được giải thích bởi sự hấp thu (giải hấp): trong một
thời gian của chu kì, một phần chất bị khuếch tán vào dung dịch.

( QS )
vs F = QSmNA = 2.14
7) n = =
vLyy  S  F
  NA
 Sm 

346 | Bản quyền thuộc về Tạp chí Olympiad Hóa học KEM
Trong vs số mol electron vận chuyển, vLys là số mol protein phản ứng, S là
diện tích điện cực, Sm là diện tích bị chiếm bởi đơn phân tử.
8) Lys(SS)4(sol) → Lys(SS)4(ads)
Lys(SS)4(ads) + Hg → Lys(SS)3(SS2-)Hg2+(ads)
Lys(SS)3(SS2-)Hg2+(ads) + 2H+ +2e → Lys(SS)3(SH)2(ads) + Hg
Lys(SS)3(SH)2(ads) – 2H+ - 2e → Lys(SS)4(ads)

347 | Bản quyền thuộc về Tạp chí Olympiad Hóa học KEM
Bài 17
Azerbaijan được biết
đến vì những lĩnh vực
liên quan đến dầu khí.
Mũi khoan dầu đầu tiên
được Babi-Heybat thực
hiện vào năm 1846, 13
năm trước khi giếng dầu
thương mại đầu tiên
được xây dựng ở
Pennsylvania (Mỹ). Sự
kiện đáng ghi nhớ này
trong lịch sử Azerbaijan được xem như là điểm khởi đầu của công nghiệp
dầu hiện đại, lĩnh vực dẫn đầu kinh tế thế giới ngày nay. Hiện nay,
Azerbaijan đang phát triển các quá trình sản xuất dầu trên đất và thềm
biển. Mặc dù các cảnh báo nghiêm trọng đã được đưa ra, nhưng nguy cơ
về ô nhiễm môi trường bởi hydrocarbon vẫn luôn tồn tại trong quá trình
sản xuất, vận chuyển và xử lí dầu. Trong bài tập này, chúng ta sẽ xem xét
các công nghệ khác nhau để làm sạch tràn dầu và các đặc trưng cụ thể
của những hướng chuyển hóa liên quan.
Việc ứng dụng các dung môi phức (những chất phân tán) để thu nạp dầu
tràn nằm trong số các phương pháp làm sạch triển vọng nhất. Hợp chất
hữu cơ X (có 11.94 % H về khối lượng) là thành phần điển hình của các
chất phân tán như vậy. Tính an toàn của X với cơ thể người đã được tranh
cãi rất gay gắt. Hợp chất X1 (có 54.53 % carbon về khối lượng) được tạo
thành từ 3 nguyên tố và bị bài tiết cùng với nước tiểu là chất chuyển hóa
chính của X trong cơ thể người. Số nguyên tử của các nguyên tố khác nhau
trong X1 là 3 số hạng liên tiếp của một cấp số nhân (n, nq, nq2) và tổng
các số này không vượt quá 25.
1)
1a) Xác định mối liên hệ giữa số nguyên tử carbon và oxygen trong X1.
Không có dữ
n(C) > n(O) n(C) < n(O) n(C) = n(O)
kiện phù hợp
  

348 | Bản quyền thuộc về Tạp chí Olympiad Hóa học KEM
1b) Dẫn ra công thức kinh nghiệm của X1 (từ ý này hãy luôn trình bày các
bước làm của bạn khi cần thiết).
Quá trình sinh chuyển hóa của X thành X1 diễn ra qua hai giai đoạn xúc
tác bởi enzym, theo các phương trình phản ứng dưới đây (NAD+ và NADH
là các dạng oxid hóa và khử của nicotinamide adenine dinucleotide):
Х + NAD+ → X0 + NADH + H+ (1)
X0 + NAD+ + H2O → X1 + NADH + H+ (2)
1b) Dẫn ra công thức phân tử của X.
Một biến đổi chất chuyển hóa phụ của X được xúc tác bởi monooxygenase
phụ thuộc cytochrome P450. Phản ứng này tạo thành hai hợp chất X2
(51.56 % oxygen và 9.74 % hydrogen về khối lượng) và X3.
1c) Dẫn ra công thức phân tử và cấu tạo của X2.
1d) X chỉ chứa các nguyên tử carbon bậc nhất và bậc hai. X0 và X3 chứa
các nhóm chức thông thường. Xác định công thức cấu tạo của X, X1
và X3.
Trong một nghiên cứu thuốc, các tình nguyện viên phải tiếp xúc thường
xuyên với các dung môi tạo nên từ X mà không có sự bảo vệ phù hợp. Khi
xét nghiệm, đã phát hiện chất X với nồng độ cố định trong máu.
2) X1 bị bài tiết ra với nước tiểu. Hãy chọn đồ thị ứng với hàm lượng hàng
ngày của X1 trong cơ thể một tình nguyện viên trong thí nghiệm này.
1 2
mass
mass

0
0 0 6 12 18 24
0 6 12 18 24 time, h
time, h

3 4

349 | Bản quyền thuộc về Tạp chí Olympiad Hóa học KEM
mass
mass

0
0
0 6 12 18 24 0 6 12 18 24
time, h time, h

5 6

mass
mass

0 0
0 6 12 18 24 0 6 12 18 24
time, h
time, h

Việc sử dụng các vi khuẩn khác cũng được xem như một hướng triển vọng
để loại bỏ các tạp chất chứa hydrocarbon (thậm chí là chất thơm) khỏi
nước biển và dầu. Trong điều kiện háo khí, benzene bị thoái phân sinh học
theo chuỗi phản ứng sau (ba giai đoạn đầu tiên đã cân bằng):

Trong cùng điều kiện, một hydrocarbon thơm đơn vòng P (có 91.25 %
carbon về khối lượng) trải qua chuyển hóa sau (ba giai đoạn đầu tiên đã
cân bằng):

P3 cho kết quả dương tính với phản ứng iodoform. Với 100 mg mẫu P3
cần 6.41 mL dung dịch KOH 0.100 M để trung hòa hoàn toàn.
350 | Bản quyền thuộc về Tạp chí Olympiad Hóa học KEM
3) Xác định công thức cấu tạo của P-P3. Xác định dạng tautomer bền
nhất của P3.
Các vi sinh vật Alicycliphilus có khả năng gây ra sự thoái phân sinh học
các hydrocarbon thơm, thậm chí ở trong đất. Quá trình này đòi hỏi một
nguồn nhận electron phù hợp như anion vô cơ Y1 (ba giai đoạn đầu tiên
đã cân bằng).

Tiểu phân trung gian Y2 bị phân hủy xúc tác enzyme theo phương trình
phản ứng đã cân bằng sau:
Y2(aq)  Y3(aq) + Y4(g)
trong đó mỗi chất Y3 và Y4 đều chỉ được tạo thành bởi một nguyên tố. T2
không có các nhóm chức chứa oxygen giống nhau. Khi xử lí với dung dịch
ammonia của Ag2O thì T2 tạo kết tủa còn Y3 thì không.
4) Xác định công thức phân tử của Y1-Y4 và công thức cấu tạo của T1-
T2. Hãy cho biết dạng tautomer bền nhất của T2.
Hướng dẫn
1a) n(C) > n(O)
1b) Theo ý 1a), ba trường hợp có thể xảy ra với X1: n(H)>n(C)>n(O),
n(C)>n(H)>n(O), và n(C)>n(O)>n(H). Với mỗi bất đẳng thức, có thể viết
các công thức tương ứng, sử dụng các nguyên tắc của cấp số nhân (q
là công bội), các phương trình để tính hàm lượng của carbon và các
nghiệm của nó.
Nghiệ Nghiệ
Bất đẳng Công m thứ m thứ
Phương trình
thức thức nhất hai
(q1) (q2)

n(H)>n(C) СqnHq2nO 12.01qn


= 0.5453 2.00 7.93
>n(O) n 12.01qn + 1.008q2n + 16.00n

351 | Bản quyền thuộc về Tạp chí Olympiad Hóa học KEM
n(C)>n(H) Сq2nHqnO 12.01q2n
= 0.5453 –1.21 1.32
>n(O) n 12.01q2n + 1.008qn + 16.00n

n(C)>n(O) Сq2nHnOq 12.01q2n


= 0.5453 –0.06 1.66
>n(H) n 12.01q n + 1.008n + 16.00qn
2

Chỉ có duy nhất một giá trị nghiệm nguyên dương, do vậy công thức kinh
nghiệm là C2H4O.
Quá trình sinh chuyển hóa của X thành X1 diễn ra qua hai giai đoạn xúc
tác bởi enzym, theo các phương trình phản ứng dưới đây (NAD+ và NADH
là các dạng oxid hóa và khử của nicotinamide adenine dinucleotide):
Х + NAD+ → X0 + NADH + H+ (1)
X0 + NAD+ + H2O → X1 + NADH + H+ (2)
1c) Từ phương trình phản ứng (1), (2) có thể viết công thức phân tử của
X dưới dạng: C2nH4nOn + 2H − 1O = C2nH4n+2On-1. Dựa vào dữ kiện về
hàm lượng hydrogen:

1.008(4n+2)
=0.1194
12.01×2n+1.008(4n+2)+16.00(n-1)
tìm được n = 3, công thức phân tử của X là C6H14O2.
Một biến đổi chất chuyển hóa phụ của X được xúc tác bởi monooxygenase
phụ thuộc cytochrome P450. Phản ứng này tạo thành hai hợp chất X2
(51.56 % oxygen và 9.74 % hydrogen về khối lượng) và X3.
1d) X2 được tạo thành từ X gồm 3 nguyên tố (C, H, and O) qua một phản
ứng xúc tác bởi monooxygenase:

100 − 51.56 − 9.74 9.74 51.56


n(C):n(H):n(O) = : : = 1: 3:1 .
12 1.008 16.00
Do số nguyên tử hydrogen phải là số chẵn, nên công thức phân tử X2 là
C2H6O2. Các chất khác với số hydrogen lớn hơn không thỏa mãn.
Công thức cấu tạo thỏa mãn là: HOCH2CH2OH
1e)

352 | Bản quyền thuộc về Tạp chí Olympiad Hóa học KEM
2) Đồ thị 1

mass

0 6 12 18 24
time, h

3) Dioxygenase sẽ gắn thêm 2 nguyên tử oxygen vào các vị trí vicinal (1,
2) của chất nền, tiếp sau đó có thể xảy ra sự sắp xếp lại các liên kết
hóa học. Có thể xác định được công thức kinh nghiệm của P là C7H8 (
91.25 100-91.25
C:H= : =7:8 ). Chỉ có 1 hợp chất thơm thỏa mãn là
12.01 1.008
toluene.

100
Khối lượng mol của P3 chứa (các) nhóm có tính acid là = 156
6.41  0.100
(gam/mol). Do có hai giai đoạn với dioxygenase nên công thức đề nghị là
C7H8O4.
P3 cần phải là monocarboxylic acid nếu nó vẫn có 7 nguyên tử carbon.
Các mảnh chứa nhóm CH3CO- (hoặc CH3CH(OH) – nhóm này sẽ tiếp tục
bị chuyển hóa thành CH3CO-) sẽ tham gia vào phản ứng iodoform. Do vậy,
có thể xảy ra sự phân cắt phần benzene trong giai đoạn oxygenase thứ
hai ở vị trí carbon nối với nhóm methyl.

353 | Bản quyền thuộc về Tạp chí Olympiad Hóa học KEM
P P1 P2 P3

4)
Y1 Y2 Y3
ClO3- ClO2- Cl–

Y4 T1 T2
O2

354 | Bản quyền thuộc về Tạp chí Olympiad Hóa học KEM
Bài 18
Quá trình tổng hợp một axit béo A trải qua vài bước thoái phân sau đây

Trong phản ứng chuyển B thành C thì quá trình oxy hóa β không khả thi, ở
phản ứng chuyển C thành D chỉ xảy ra quá trình oxy hóa ω-.
Hợp chất I ở cuối dãy tổng hợp có thành phần phân tích nguyên tố như
sau: 49.30 % C, 6.91 % H, 43.79 % O.
Phổ 1H-NMR của chất I cho các tín hiệu sau: một vân đơn ở 12.2 ppm (2
H), một vân đa ở 2.6 ppm (2 H), và một vân đôi ở 1.0 ppm (6 H). Phổ 13C-
NMR của chất I cũng cho ba tín hiệu ở 176 ppm, 41 ppm, và 13 ppm.
1. Vẽ công thức cấu tạo cho các chất từ A đến I. Đối với công thức của
coenzym A hãy sử dụng ký hiệu „HS-CoA“.
Từ I chúng ta biết thêm rằng nó tồn tại ở dạng meso.
Về A thì ta biết rằng nó gồm ba trung tâm bất đối, trong đó đầu gần đầu
ω có cấu hình R.
2. Vẽ công thức cấu trúc của A.

355 | Bản quyền thuộc về Tạp chí Olympiad Hóa học KEM
Hướng dẫn
1)
A O

OH

B O

SCoA

C
SCoA

D O
SCoA
HO
O

E O
SCoA
CoAS
O

F O
SCoA
CoAS
O

356 | Bản quyền thuộc về Tạp chí Olympiad Hóa học KEM
G O
SCoA
CoAS
O

H O
SCoA
CoAS
O

I O
OH
HO
O

2)
O

OH

357 | Bản quyền thuộc về Tạp chí Olympiad Hóa học KEM
Bài 19
Vai trò quan trọng của lipit là góp phần vào sự tạo thành màng tế bào và
đóng vai trò chất tạo nhũ. Lòng đỏ trứng được sử dụng nhiều làm chất tạo
nhũ trong nhiều lĩnh vực khác nhau, nó nhũ hóa cả pha ưa nước lẫn pha kị
nước chẳng hạn như trong việc làm bánh mì cũng như thêm vào sơm
tempera (để sơn biển quảng cáo). Chất có vai trò chính ở đây là lecithin
chiếm 1/10 trong lòng đỏ trứng.
1. Một loại lecithin được tạo thành từ axit panmitic (axit
hexadecanoic), axit linoleic ((9Z,12Z)-octadeca-9,12-dienoic) vàd cholin
((2-hydroxyetyl)-trimetylamoni). Đề nghị một cấu trúc có thể có của
lecithin.
2. Phân tử lecithin có dạng hình trụ với độ dài 1.7 nm. Chất này có thể
tạo thành dạng tổ hợp nào? Chọn ý đúng trong tờ phiếu trả lời, lưu ý
rằng có thể có nhiều hơn một đáp án đúng.
3. Axit linoleic acid cùng với lecithin được dùng để tổng hợp ATP trong
quá trình trao đổi chất. Đưa ra tất cả các phản ứng trong quá trình
tạo thành 1 mol axetyl-CoA từ 1 mol axit linoleic. Các phương trình
buộc phải ghi rõ cấu trúc của các dẫn xuất của axit linoleic cũng như
ghi rõ các ký hiệu viết tắt của ngành Hóa sinh đối với các chất khác.
Một vai trò quan trọng khác của lipit là khả năng cung cấp năng lượng cho
cơ thể, đặc biệt là lúc cơ thể đã sử dụng hết các nguồn năng lượng sẵn
có. Nghiên cứu một vận động viên chạy đường dài cho ta các dữ liệu sau:

- Anh ta sử dụng 7.50·1021 phân tử ATP / giây để tạo thành nguồn


năng lượng cần thiết.
- Cho rằng chất béo trong cơ thể vận động viên hoàn toàn là các
triglyxerit. Bên cạnh đó ta giả thiết rằng 30 % tổng số axit béo là
axit panmitic, 15 % axit stearic (axit octadecanoic), 45 % axit oleic
(axit Z-octadec-9-enoic) và 10 % axit linoleic (%mol/mol).
- Vận tốc trung bình của vận động viên là 12 km/h.
- Sự chuyển hóa một mol glycerol tạo thành 18.5 mol ATP.

358 | Bản quyền thuộc về Tạp chí Olympiad Hóa học KEM
4. Hoàn tất bảng thông tin về các axit béo đã cho trong tờ phiếu trả
lời.
5. Có bao nhiêu phân tử ATP được sinh ra trong quá trình chuyển hóa
thoái phân 10.0 kg mỡ cơ thể có thành phần đã được đề cập ở trên?
6. Tính khoảng cách mà người vận động viên đã chạy được để sử
dụng hết 10.0 kg mỡ cơ thể có thành phần đã cho. Giả thiết sự
chuyển hóa chất béo chỉ xảy ra sau hai giờ từ khi anh ta bắt đầu
chạy và từ thời điểm đó toàn bộ năng lượng của anh ta chỉ được
cung cấp bằng con đường chuyển hóa thoái phân chất béo.
Hướng dẫn
1. Công thức của một phospholipit:
O
O O
O O
+ O P O
N
O

2. Lớp kép
3. Sự chuyển hóa thoái phân axit linoleic:
O
OH

+HS-CoA, + ATP O
S-CoA
-OH-, -AMP, -PP

+ FAD O
S-CoA
- FAD.H2

OH
+ H2O O
S-CoA

O
+ NAD+ O
S-CoA
- NADH+H+

O O
+ HS-CoA
S-CoA S-CoA

359 | Bản quyền thuộc về Tạp chí Olympiad Hóa học KEM
360 | Bản quyền thuộc về Tạp chí Olympiad Hóa học KEM
4.
mol
mol/mol ATP/axit m/mol
CTPT ATP/ch KLPT
chất béo béo. chất béo
ất béo
Axit
C16H3 3·0.3 =
palmiti 106 95.4 256 230.4
2O2 0.9
c
Axit C18H3 3·0.15 =
120 54 284 127.8
stearic 6O2 0.45
Axit C18H3 3·0.45 =
118.5 159.975 282 380.7
oleic 4O2 1.35
Axit C18H3 3·0.1 =
117 35.1 280 84
linolic 2O2 0.3
glycero C3 H 8
1 18.5 92 92
l O3
5. Số phân tử ATP:
Tổng khối lượng: 914.9 g – 3·18 g (water) = 860.9 g/mol
ATP: 362.975 mol/mol chất béo
10000g
nfat = = 11.62mol
860.9g  mol−1

nATP = 11.62·362.975 = 4216 mol = 2.539·1027 phân tử


6. Khoảng cách mà vận động viên đã chạy được:

2.539  1027
t= = 338535s = 5642min = 94.04h
7.50  1021
ttot= 94.04 + 2 = 96.04 h
s = v·t = 12 km/h · 96.04 h = 1152 km ≈ 1.2·103 km

361 | Bản quyền thuộc về Tạp chí Olympiad Hóa học KEM
Polymer
Bài 1
Các polymer là nhóm phân tử đặc biệt quan trọng, tạo thành bởi các đơn
vị cấu trúc nhỏ, trùng lặp gọi là các monomer. Có nhiều loại polymer và
nhiều phương pháp để polymer hoá (tổng hợp polymer), tuy nhiên hai quá
trình quan trọng nhất là polymer hoá gốc và polymer hoá ngưng tụ.
Trong Hoá hữu cơ, chúng ta dùng công thức thu gọn (hay công thức khung
sườn) để biểu diễn các phân tử hữu cơ. Khi dùng công thức này, chúng ta
không vẽ nguyên tử carbon và những nguyên tử hydrogen gắn vào carbon.

Bảng 1: Các nhóm chức phổ biến.


Chú ý rằng R có thể là hydrogen hoặc một mạch carbon, ngoại trừ trường
hợp R là hydrogen thì có thể thay đổi nhóm chức, ví dụ như trong aldehyde
và ketone. Đối với các alkene và alkyne, R có thể là một loại nguyên tử
khác.
Trước khi bắt đầu xem xét các polymer, hãy quan sát một số phân tử nhỏ
hơn có nhóm chức giống nhau. Chúng ta chủ yếu quan tâm đến các ester
và amide cho quá trình polymer hoá ngưng tụ. Chúng ta cũng có thể sử
dụng nhiều loại nhóm chức để tạo các ester và amide, nhưng phản ứng
thì tương tự nhau.

362 | Bản quyền thuộc về Tạp chí Olympiad Hóa học KEM
a) Các phản ứng khác của những phản ứng ester hoá và amide hoá này
là gì?

Các nylon là nhóm polymer quen thuộc được con người sử dụng trong
may mặc. Một số nylon có thể tổng hợp từ diamine và dicarboxylic, ví dụ

363 | Bản quyền thuộc về Tạp chí Olympiad Hóa học KEM
như Nylon 6,6 được tổng hợp dưới đây. Các số 6,6 để chỉ số carbon trong
diamine và trong dicarboxylic acid.

b)
i. Xác định cấu tạo của Nylon 4,4
ii. Nylon 12 được tạo thành bởi một monomer khác với loại dùng
trong Nylon 4,4 hay 6,6. Xác định cấu tạo monomer của polymer
này.
Một nhóm polymer sinh hoạt quan trọng là tinh bột, rất quen thuộc trong
nhiều loại thực phẩm. Trước tiên, chúng ta xem xét glucose, là monomer
của tinh bột. Nó có thể tồn tại ở dạng đồng phân mạch hở hoặc mạch
vòng. Quá tình nội chuyển đổi giữa mạch hở và mạch vòng gọi là tautomer
hoá (hỗ biến), do đó giữa hai đồng phân có một cân bằng. Tuy nhiên,
glucose chủ yếu tồn tại ở dạng mạch vòng.
c)
i. Đánh dấu nguyên tử carbon của dạng mạch hở tương ứng với
nguyên tử carbon(*) của dạng mạch vòng.
Một nhóm chức khác (hemiacetal, biểu diễn dưới đây) được tạo thành
trong dạng mạch vòng:

ii. Khoanh tròn nhóm hemiacetal trong dạng mạch vòng sau đây:

364 | Bản quyền thuộc về Tạp chí Olympiad Hóa học KEM
Trong tinh bột, các đơn vị cấu trúc liên kết với nhau bởi các nhóm acetal,
khi ngưng tụ giải phóng nước. Cấu trúc nhóm acetal như sau:

iii. Khoanh tròn nhóm acetal trong đoạn cấu trúc tinh bột sau:

iv. Gọi tên hai nhóm chức phản ứng với nhau tạo thành hemiacetal.
v. Gọi tên hai nhóm chức phản ứng với nhau tạo thành acetal.
Ngưng tụ không phải là phương pháp duy nhất để điều chế polymer. Trong
các quá trình polymer hoá gốc tự do, một liên kết đôi yếu hơn bị phá vỡ để
tạo thành hai liên kết đôi mạnh hơn. Quá trình này cần một chất khơi mào,
như potassium persulfate hoặc benzoyl peroxide để bắt đầu phản ứng.
Dưới đây là một ví dụ phản ứng polymer hoá ethene thành polyethylen, sử
dụng benzoyl peroxide làm chất khơi màu. Giai đoạn tắt mạch khá phức
tạp và chúng ta không cần quan tâm.

365 | Bản quyền thuộc về Tạp chí Olympiad Hóa học KEM
d)
i. Polymer dưới đây được tạo thành từ monomer nào?

Polybutadiene cũng được tạo thành bởi phản ứng polymer hóa butadiene.

ii. Do butadiene có hai liên kết đôi nên có hai hướng khả thi để
polymer hoá. Vẽ một mảnh cấu trúc của polymer được tạo thành
theo hai hướng này.
Polyvinyl alcohol là một polymer tan được trong nước và được dùng trong
các phản ứng polymer hoá nhũ tương – là một phương pháp khác để điều
chế polymer.

Bạn được giao nhiệm vụ tổng hợp một mẫu chất và được cung cấp một
số lọ hoá chất. Một lọ có dán nhãn “ethenol (vinyl alcohol)” có cấu tạo như
hình dưới. Tuy nhiên, bạn biết rằng cấu tạo này là không chính xác, bởi
ethenol cũng như glucose, sẽ bị tautomẻ hoá gần như hoàn toàn thành
một cấu tạo khác.

iii. Cấu tạo khác đó là gì?

366 | Bản quyền thuộc về Tạp chí Olympiad Hóa học KEM
Bên cạnh đó, còn có các lọ được dán nhãn “acetic acid”, “potassium
persulfate”, “ethanol”, “ethenol” “butanal” và “vinyl acetate”.

iv. Đề xuất các giai đoạn của tổng hợp PVA sử dụng các hoá chất
trên. Bạn có thể tham khảo các phản ứng ở phần a.

367 | Bản quyền thuộc về Tạp chí Olympiad Hóa học KEM
Hướng dẫn
a)

Theo thứ tự 3 ô trống: H2O, HCl, CH3OH


b)
i.

ii.

c)
i.

368 | Bản quyền thuộc về Tạp chí Olympiad Hóa học KEM
ii.

iii.

iv. aldehyde và alcohol


v. hemiacetal và alcohol
d)
i.

ii.

iii.

369 | Bản quyền thuộc về Tạp chí Olympiad Hóa học KEM
iv.
1) Polymer hóa viny acetate với K2S2O8 làm chất khơi mào để tạo thành
poly(vinyl acetate).
2) Cho poly(vinyl acetate) phản ứng với ethanol để tạo thành poly(vinyl
alcohol) và ethyl acetate.

370 | Bản quyền thuộc về Tạp chí Olympiad Hóa học KEM
Bài 2
Có 5 polime sau:
a. Nilon-6,6 được hình thành từ axit ađipic và hexametylenđiamin.
b. Nilon-6 được hình thành từ ε-caprolactam.

c. Đacron được hình thành từ đimetyl terephtalat và etylen glicol.


d. Gliptan được hình thành từ glixerin và anhiđrit phtalic.
e. Poliuretan được hình thành từ điisoxianat (thí dụ toluenđiisoxianat) và
etylen glicol.
Hãy cho biết các polime trên thuộc loại nào? Viết phương trình phản ứng
tạo thành sản phẩm, chỉ ra công thức một đoạn mạch của polime và gọi
tên theo danh pháp IUPAC các polime a, b, c.
Hướng dẫn
Nilon-6,6 là poliamit do phản ứng của cả 2 nhóm –COOH của axit ađipic
phản ứng với cả 2 nhóm –NH2 của hexametylenđiamin. Các liên kết –
CONH- được hình
thành do tách loại H2O. Ban đầu phản ứng cho muối nilon, sau đó đun
nóng.

HOOC(CH2)4COOH + H2N(CH2)6NH2 OOC(CH2)4COOH3N(CH2)6NH3


o
t
-C-(CH2)4-C-NH-(CH2)6-NH-C-(CH2)4-C-NH-(CH2)6-NH-
O O O O
Poli(hexametylen a®ipamit)
Mét ®o¹n m¹ch cña polime: NH-(CH2)6-NH-C-(CH2)4-C
O O

b. Nilon-6 cũng là poliamit nhưng được hình thành từ monome

ε-caprolactam. Đó là amit vòng của axit ε-aminocaproic . Khi đun nóng


vòng lactam được mở ra cho muối aminoaxit sau đó hình thành liên kết
amit với phân tử khác khi tách loại nước.

371 | Bản quyền thuộc về Tạp chí Olympiad Hóa học KEM
O
o + o
t t
N H H3N-(CH2)5COO- NH-(CH2)5-C-NH-(CH2)5-C-N H-(CH2)5-C
H2O - H 2O
O O O
Poli(amit cña axit 6-aminohexanoic)
ε-caprolactam

Mét ®o¹n m¹ch cña polime: NH-(CH2)5-C


O

c. Đacron là polieste do phản ứng chuyển đổi este giữa đimetyl terephtalat
với etylen glicol.

HO-CH2CH2OH + H3COOC- -COOCH3 + HO-CH2CH2OH + H3COOC- -COOCH3

to
O-CH2CH2-O-C- -C-O-CH2CH2O-C- -C
- CH3OH
O O O O

Poli(etylen terephtalat)

Mét ®o¹n m¹ch cña polime: C- -C-O-CH2CH2-O


O O

d. Gliptan cũng là polieste, nhưng glyxerrin sẽ tạo thành nhựa mắt lưới dẻo
cứng. Ở giai đoạn đầu tiên sẽ hình thành polime thẳng với nhóm OH bậc
một.

O O
O C C O O C C O

+ OH-CH2CHCH2-OH + + OH-CH2CHCH2-OH
OH OH

C C O-CH2CHCH2-O C C O-CH2CHCH2-O
O O OH O O OH
-H2O

O-CH2CHCH2-O C C
Mét ®o¹n m¹ch cña polime: O O
OH

Nhóm OH tự do bậc hai sẽ liên kết chéo với nhiều phân tử anhiđrit phtalic
khác.

372 | Bản quyền thuộc về Tạp chí Olympiad Hóa học KEM
e. Poliuretan được hình thành từ etylen glicol với điisoxianat.

HO-CH2CH2OH + O=C=N N=C=O + HO-CH2CH2OH + O=C=N N=C=O

CH3 CH3
O H H O O H H O
OCH2CH2O C N N C OCH2CH2O C N N C

CH3 CH3
O H H O
Mét ®o¹n m¹ch cña polime: C N N C OCH2CH2O

CH3

373 | Bản quyền thuộc về Tạp chí Olympiad Hóa học KEM
Bài 3
Policacbonat là một loại nhựa trong suốt, bền và không giòn được sử dụng
làm mũ bảo hiểm. Từ cumen và các chất vô cơ cần thiết, hãy viết phương trình
tổng hợp policacbonat có công thức ở dưới đây:

Hướng dẫn

374 | Bản quyền thuộc về Tạp chí Olympiad Hóa học KEM
Bài 4
Các phương pháp trùng hợp ethylene sau đây đều được ứng dụng trong
công nghiệp.
Phương pháp 1: áp suất cao (100-200 MPa), nhiệt độ 80-120 oC, có mặt
các peroxide;
Phương pháp 2: áp suất trung bình (3-7 MPa), nhiệt độ 120-150 oC, có mặt
các oxide kim loại chuyển tiếp;
Phương pháp 3: áp suất thấp (0.2-0.6 MPa), nhiệt độ 60-70 oC; có xúc tác
Ziegler-Natta.
Xúc tác Ziegler-Natta được điều chế bằng cách trộn lẫn xúc tác chính (A)
với đồng xúc tác (B). Tương tác của 10.00 gam A (chất lỏng bay hơi, không
màu) với nước dư, tạo thành 0.2627 mol ethane. Tương tác của B (chất
lỏng bay hơi, không màu) với nước dư, tạo thành kết tủa trắng. Thêm kẽm
vào hỗn hợp A và B thì có khí tạo thành và xuất hiện màu tím, còn khi thêm
hydrogen peroxide trong kiềm vào hỗn hợp thì xuất hiện màu vàng. Dịch
lọc nhận được sau khi tách kết tủa tạo ra từ B, được đem xử lí với dung
dịch AgNO3, thu được kết tủa trắng. Tỉ khối hơi của B so với hydrogen (giả
sử rằng các phân tử không liên kết ở thể hơi) là 95.
1) Xác định công thức của A, B.
2) Với các phương pháp trùng hợp 1 và 2, viết sơ đồ cơ chế giai đoạn phát
triển.
Các hợp chất chứa nhóm hydroxyl (ví dụ, methanol) phản ứng với A gây
nổ. Ngoài ra, A cũng tự cháy trong không khí. Trong sản xuất công nghiệp,
nitrogen được phụt vào toàn hệ và quy trình được thực hiện trong phase
lỏng bằng cách cho dòng ethylene đi qua dung dịch xúc tác trong các
hydrocarbon lỏng.
3) Viết các phương trình phản ứng của A với methanol và không khí.
4) Trong sơ đồ dưới đây của phương pháp 3, hãy bổ sung tên gọi các quá
trình (ghi số tương ứng vào các hình vuông) và tên chất (ghi chữ cái
tương ứng vào hình tròn) còn thiếu.

375 | Bản quyền thuộc về Tạp chí Olympiad Hóa học KEM
Hướng dẫn
1) Do tương tác của A với nước dư tạo thành ethane nên A có thể là hợp
chất cơ kim dạng Me(C2H5)n.
Me(C2H5)n + nH2O → Me(OH)n + nC2H6
Dễ tìm được Me = 9n gam/mol, chỉ có aluminum (n = 3) thoả mãn. Do vậy,
A là triethylaluminum Al(C2H5)3. Dựa vào các phản ứng và tính toán khối
lượng mol, có thể xác định được B là titanium chloride TiCl4.
2) I) R + CH2=CH2 → R-CH2-CH2•
II) R+ + CH2=CH2 → R-CH2-CH2+
3) 3.1) Al(C2H5)3 + 3CH3OH → Al(OCH3)3 + 3C2H6
3.2) 2Al(C2H5)3 + 21O2 → Al2O3 + 12CO2 + 15H2O
4) 1 = quá trình cất lại; 2 = trộn; 3 = rửa; 4 = làm khô
a = methanol; b = nitrogen; c = hỗn hợp nước - methanol; d = xăng; e =
không khí; f = nước.

376 | Bản quyền thuộc về Tạp chí Olympiad Hóa học KEM
Bài 5
Ngày xửa ngày xưa, một nhà hóa học trẻ quyết chí đi khắp thế gian tìm vợ.
Trước tiên, anh đã tổng hợp một số polymer có ích từ ethylene (P1),
styrene (P2), hỗn hợp của ethylene glycol và terephthalic acid (P3).
Polymer (P4) là trường hợp khó khăn nhất. Ban đầu, nhà hóa học trẻ thực
hiện phản ứng giữa hexanediol-1,6 và 1,4-butane diisocyanate, với
diisocyanate được lấy dư một chút. Sau đó anh thêm nước vào polymer
nhận được rồi đun nóng hợp. Kết quả, anh thu được một polymer thật sự
tuyệt diệu! Nhà hóa học này đã làm được những thứ rất hữu dụng từ mỗi
loại polymer: một chiếc túi từ polyester, một chiếc bình du lịch từ loại
polymer khác có chứa các hệ thơm, một chiếc áo mưa từ polyolephine.
Với polyurethane thì anh dùng để làm những chiếc đế giày.
1) Viết các sơ đồ tổng hợp P1 - P3.
2) Cho biết các sản phẩm được làm ra polymer tương ứng số mấy?
Kể từ khi nhận ra không được làm đổ rượu mạnh vào vật liệu polyester,
nhà hóa học quyết định chế tạo lại từ một vật liệu khác.
3) Phản ứng nào xảy ra khi đổ rượu vào các vật làm bằng polyester? Viết
phương trình phản ứng và chỉ ra sản phẩm gây độc.
Polymer P4 được tạo liên kết chéo và làm căng bởi do lấp đầy bởi các
bóng khí. Kết quả là thu được vật liệu để chế tạo những đôi giày rất nhẹ,
thuận tiện cho việc đi lại.
4) Viết phản ứng và cấu trúc của polymer tạo thành từ diol và
diisocyanate ở giai đoạn đầu của quá trình tổng hợp.
5) Viết phương trình phản ứng ngưng tụ dẫn đến sự giải phóng sản phẩm
khí khi có nước ở giai đoạn hai.
6) Viết phương trình phản ứng tạo liên kết chéo khi xử lí sản phẩm ngưng
tụ với diisocyanate.
Sau nhiều ngày chu du khắp thế gian, nhà hóa học đã may mắn tìm được
người bạn đời của mình. Suốt quãng đường ấy, 7 cặp đế giày đã mòn. Nhà
hóa học đi với tốc độ 1 bước mỗi giây, và mỗi bước chân độ lớn khoảng 1
m. Độ bền của vật liệu được tính bởi công thức kinh nghiệm:

377 | Bản quyền thuộc về Tạp chí Olympiad Hóa học KEM
−4
 = 3.10  108  e−2.210 P
giây, trong đó P là áp suất tác động lên vật liệu (Pa).
Cho biết nhà hóa học nặng 70 kg và diện tích mỗi đế giày là cỡ 200 cm2.
7) Ước tính quãng đường nhà hóa học đã đi trước khi tìm thấy người bạn
đời của mình. Giả sử rằng trong suốt chuyến du hành, nhà hóa học
luôn bước đều.
Hướng dẫn
1) P1 và P2 được tạo thành bởi phản ứng trùng hợp alkene. P3 được tạo
thành bởi sự ngưng tụ các nhóm hydroxyl và carboxyl.
HO O
n HO
* n* n OH + n
* * O OH
n O O
n *
O O *
n

2) Hai trong số các polymer trên (P2 and P3) có chứa các hợp phần thơm.
P3 (polyethylene terephthalate) là polyester, và được dùng để chế tạo túi.
Chiếc bình được làm từ P2 (polystyrene). P1 (polyethylene) là
polyolephine dùng để làm áo mưa. Polyurethane P4, sản phẩm ngưng tụ
của diol với diisocyanate, là nguyên liệu cho những chiếc giày.
3) Khi tiếp xúc với rượu (alcohol), sẽ xảy ra phản ứng ester chéo hóa dẫn
tới sự tạo thành ethylene glycol độc. Do vậy, không được đổ rượu mạnh
vào các sản phẩm làm bằng PET. Thay vào đó, có thể sử dụng các chai
thủy tinh hoặc polycarbonate để chứa rượu.
O O 2EtOH O O
* 2 O O HO
+ OH
O O * Et O O
O O H

4) Các polyurethane mạch thẳng được tạo thành bởi phản ứng ngưng tụ
giữa alcohol và các nhóm isocyanate. Do diisocyanate được lấy dư, nên
các nhóm isocyanate có thể được tìm thấy ở các đầu mạch polymer.
OH O
HO H
+ N O C NCO
OCN OCN C O N
NCO H
O

378 | Bản quyền thuộc về Tạp chí Olympiad Hóa học KEM
5) Khi có nước và được đun nóng, các nhóm isocyanate đầu mạch bị thủy
phân một phần thành gốc carbamic acid NHCOOH, rồi sau đó tiếp tục
decarboxyl hóa tạo thành các nhóm amino. Các nhóm amino phản ứng
với các nhóm isocyanate vẫn còn trong hỗn hợp, tạo thành các mảnh urea.
Phản ứng tổng (trong đây mảnh polymer không tham gia vào phản ứng
được kí hiệu bởi đường lượn sóng) là:
O
H2O OCN NCO
NCO NCO + NCO NCO N N + CO2
H H

Carbon dioxide giải phóng đã làm giãn nở polymer.


6) Somng song với phản ứng thủy phân và tạo thành các mảnh urea, thì
các urea cũng tương tác với các nhóm isocyanate chưa tham gia phản
ứng của polymer mạch thẳng. Đây chính là cách để các chuỗi polymer
chứa các mảnh NHCONH tạo liên kết chéo:
O
O O N HN
NCO NCO + + O HN NH O
N N N N
H H H H NH N
O

7) Áp lực nhà hóa học tạo lên mỗi đế giày là: P = 70·9.8 / (200·0.0001) =
34300 Pa (chúng ta tính với diện tích một đế giày, do khi bước thì một chân
ở trên cao). Độ bền tính được là:
τ = 3.10∙108exp(-2.20∙10-4 ∙ 3.43∙104) = 163700 giây
Một cặp đế giày sẽ hao mòn sau khoảng 327400 giây và 7 cặp là sau
2292000 s - tương đương với quãng đường 2300 km.

379 | Bản quyền thuộc về Tạp chí Olympiad Hóa học KEM
Bài 6
Vinyl monomer Z (26.40% N về khối lượng) được tạo thành theo nhiều
cách. X1-X3 lần lượt chứa 51.83, 46.68 và 82.24% N và A, B là các đồng
phân cấu tạo.

1) Xác định công thức phân tử X1-X3.


2) Xác định công thức cấu tạo A, B, C, Z.
Z dễ bị trùng hợp theo cơ chế gốc, sơ đồ đơn giản hóa gồm có: sự tạo
thành các gốc từ chất khơi mào, phát triển mạch bởi phản ứng lặp lại với
Z, tắt mạch bởi tương tác của hai gốc đang phát triển.
3) Biểu diễn sơ đồ khơi mào của quá trình trùng hợp Z với hỗn hợp K2S2O8
và FeCl2 trong môi trường nước ở nhiệt độ thường. Nhóm nào có thể
được đặt ở cuối phân tử polymer P tạo thành từ Z?
Ứng dụng hiện đại của P là sản xuất một loại sợi cứng, nhẹ, chống chịu tác
động hóa chất bằng cách nhiệt phân từng bước một polymer điều hòa kiểu
“đầu-đuôi”:

Các sản phẩm P1-P3 có màu đen, P2 và P3 là các polymer thơm, P1 và


P2 là các polymer mạch thẳng chỉ chứa các vòng ngưng tụ - ví dụ như:

và P3 (không chứa N) là polymer có cấu trúc 2 chiều.


4) Sản phẩm khí nào được giải phóng ở giai đoạn P1 → P2?

380 | Bản quyền thuộc về Tạp chí Olympiad Hóa học KEM
5) Vẽ cấu trúc P1 → P3.
Hướng dẫn
1, 2) X1 = HCN, X2 = NO, X3 = NH3.

3)

4) Sản phẩm khí là H2.


5)

381 | Bản quyền thuộc về Tạp chí Olympiad Hóa học KEM
Bài 7
Các phản ứng gốc không được dùng nhiều trong tổng hợp hữu cơ, nguyên
nhân chính là bởi đa số các phản ứng này có độ chọn lọc thấp. Tiềm năng
ứng dụng tổng hợp của các phản ứng gốc được mở rộng bằng cách sử
dụng các tác nhân và điều kiện phản ứng đặc biệt. Ví dụ, các chuyển hoá
alkyl halide RX khi xử lí với cặp azobis(isobutyronitrile)-Bu3SnH (hệ I) gồm
các giai đoạn khơi mào (1), tạo gốc Bu3Sn∙ (2) và phát triển mạch (3, 4):

1) Viết sản phẩm của phản ứng giữa 1-bromohexane với hệ I.


Khi xử lí 6-bromohex-1-ene với hệ I, sự vòng hoá R∙ thành hai gốc đồng
phân R1∙ và R2∙ được ưu tiên trong giai đoạn (4). Các phản ứng sau của
các gốc R1∙ và R2∙ với giai đoạn (4) dẫn đến sự tạo thành các sản phẩm
P1 và P2 tương ứng.
2) Xác định công thức cấu tạo của R1∙, R2∙, P1, P2.
3) Tính thành phần hỗn hợp thu được (theo % số mol) nếu biết trong phổ
1
H NMR, tỉ lệ các proton của methylene và methyl trong hỗn hợp là
2.8:1. Hãy cho biết thành phần này được quyết định bởi sự khống chế
động học hay nhiệt động học?
Phản ứng của I với bromodiene X không vòng, không phân nhánh, tạo
thành gốc tự do R∙, gốc này chuyển hoá thành 2-methylbicycloalkyl Y sau
2 quá trình vòng hoá nội phân tử liên tiếp.
4) Viết công thức cấu tạo của X và Y, biết rằng kích thước các vòng trong
giống với đồng phân chính trong hỗn hợp P1, P2.
Khi xử lí iodide A với hệ I sẽ gây ra sự vòng hoá domino theo cơ Y chế gốc
tự do, tạo thành hợp chất hirsutene (B) có hoạt tính sinh học tự nhiên,
thuộc họ triquinanes (các hợp chất chứa 3 vòng với kích thước xác định).
Trong sơ đồ dưới đây, nguyên tử C(1) trong A có cấu hình S, nguyên tử
C(2) có cấu hình R.

382 | Bản quyền thuộc về Tạp chí Olympiad Hóa học KEM
5) Viết công thức cấu tạo của A, B và xác định cấu hình tuyệt đối của các
tâm quang hoạt.
Hướng dẫn
1)

2)

3) %(cyclohexane) = 3 %; %(methylcyclopentane) = 97 %; đây là thành


phần được quyết định bởi sự khống chế động học.
4) Y là 2-methylbicyclopentyl.

5)

383 | Bản quyền thuộc về Tạp chí Olympiad Hóa học KEM
Bài 8
Block copolymer (chất đồng trùng hợp dạng khối) là những polymer được
cấu thành bởi từng đoạn polymer khác nhau. Nếu copolymer có cả phần
ưa nước lẫn phần kị nước thì nó có thể tạo thành dạng sắp xếp tâm-vỏ
(kiểu như các micelle) với phần kị nước hướng vào bên trong và phần ưa
nước hướng ra bên ngoài. Dưới đây là một block copolymer kiểu ABA, có
thể tạo thành micelle trong nước và bao bọc các phân tử thuốc rồi sau đó
cho phép giải phóng có kiểm soát thuốc trong điều kiện oxid hóa hoặc khử
hóa.

Copolymer khối ba, kiểu ABA.


1) Phương pháp tổng hợp polymer này như sau: Thực hiện phản ứng
trùng hợp monomer X với lượng dư monomer Y trong dung môi khan
để tạo thành mảnh polyurethane trung tâm; sau đó, thêm vào lượng
dư CH3(OCH2CH2)nOH. Viết cấu tạo các monomer X, Y.
2) Trong các điều kiện phản ứng oxid hóa - khử, liên kết disulfide có thể
bị bẻ gãy. Sử dụng công thức thu gọn R-S-S-R, viết công thức các sản
phẩm bẻ gãy liên kết (sản phẩm oxid hóa O và sản phẩm khử P).
3) Loại thuốc chống ung thư nào trong hình dưới đây có thể được bao
bọc bởi các micelle tạo thành bởi block copolymer trên. Giải thích ngắn
gọn.

Hướng dẫn
1)

384 | Bản quyền thuộc về Tạp chí Olympiad Hóa học KEM
2) O: RSO3H P: RSH
3) Chất đầu tiên (gọi là Paclitaxel), bởi paclitaxel khó tan trong nước, dễ
tan trong chất béo nên phân tử lớn này dễ được bao bọc bởi phần kị nước
bên trong micelle. Chất thứ hai (Adriamycin hydrochloride) là hợp chất ion,
tan nhiều trong nước nên ngược lại.

385 | Bản quyền thuộc về Tạp chí Olympiad Hóa học KEM
Bài 9
Nhằm giảm thiếu tác động tới môi trường, công
ty Coca-Cola đã nghiên cứu và triển khai sản
xuất những chai nhựa mà (theo báo cáo từ nhà
sản xuất) có chứa tới 30 % vật liệu có nguồn gốc
thực vật. Trong thực tế, những chai này được
đúc từ một polymer đơn giản X, là sản phẩm
ngưng tụ của ethylene glycol với carboxylic acid
thơm, hai chức C8H6O4.
1) Đề xuất 3 cấu trúc đồng phân của X và chọn ra cấu trúc được sử dụng
trong sản xuất công nghiệp.
2) Dựa trên thành phần các vật liệu có nguồn gốc từ thực vật trong bao
bì bằng nhựa (không có nhãn), hãy xác định có bao nhiêu % nhựa được
sản xuất từ nguyên liệu sinh học thô.
Mía đường chủ yếu được dùng làm nguyên liệu thô để sản xuất chai sinh
học, do nó là một trong những thực vật có khả năng hấp thụ CO2 trong khí
quyển tốt nhất và thực hiện quang hợp hiệu quả nhất.
3) Đề xuất phương pháp công nghiệp để sản xuất thành phần cần thiết
của nhựa từ mía đường. Viết các phương trình phản ứng tương ứng.
4) Có thể phân biệt chai sinh học với chai nhựa truyền thống không nếu
như hình dạng, kích thước, hình thái bề mặt, … của các chai này giống
nhau?
Trong một bài đánh giá giáo dục khoa học, hàm lượng các thành phần có
nguồn gốc sinh học trong nhựa do Coca-Cola sản xuất đã được tính theo
hai phương pháp khác nhau. Các giá trị được báo cáo là 20 % và 31.25 %.
5) Trình bày các cách tính dẫn tới những kết quả báo cáo này.
Công nghệ PlantBottle được khởi động vào năm 2009 và 20 tỉ chai sinh
học đã được tạo ra từ năm 2014. Công ty Coca-Cola báo cáo rằng điều
này cho phép tiết kiệm khoảng 445000 thùng dầu (1 thùng = 159 L.)
6) Ước tính khối lượng chai đã được dùng trong các tính toán do bộ phận
Tiếp thị của Coca-Cola thực hiện, để chứng minh cho những tác động
sinh thái của hình thức đóng chai mới này.

386 | Bản quyền thuộc về Tạp chí Olympiad Hóa học KEM
Hướng dẫn
1) Có 3 đồng phân của dicarboxylic acid thơm, có công thức phân tử
С8H6O4:

Các polyester sau được tạo thành từ phản ứng ngưng tụ các acid trên với
ethylene glycol

Lựa chọn phổ biến nhất là polyethylene terephthalate (PET), một loại nhựa
nhiệt dẻo được sử dụng nhiều trong đóng gói thực phẩm:

2) Mọi phản ứng ngưng tụ được đề cập tới ở trên đều mô tả tương tác
giữa ethylene glycol với carboxylic acid. Tính toán dưới đây xác nhận
rằng có khoảng 27 % khối lượng polymer có nguồn gốc từ ethylene
glycol (EG):

62
(EG) = = 27%
62 + 166
3) Mía đường chứa một lượng đáng kể sucrose (lên tới 21 % khối lượng
trong thân cây), chất này bị chuyển hoá thành ethanol bởi quá trình lên
men:
C12H22O11 + H2O → C6H12O6 (glucose) + C6H12O6 (fructose);
C6H12O6 (glucose) → 2C2H5OH + 2CO2

387 | Bản quyền thuộc về Tạp chí Olympiad Hóa học KEM
Ethanol có thể được chuyển đổi thành ethylene glycol qua một quá trình
công nghiệp 3 giai đoạn:

4) Các chai này có thể được phân biệt bởi phương pháp phân tích đồng
vị. Chai truyền thống được sản xuất qua quá trình xử lí nhiên liệu hoá
thạch, gần như không chứa đồng vị 14C - do chu kì bán rã (5700 năm)
quá thấp so với thời điểm hình thành nhiên liệu hoá thạch (khoảng 10
triệu năm). Ngược lại, loại chai được tạo thành từ nguyên liệu thô có
nguồn gốc thực vật gần đây gần như có hàm lượng đồng vị 14C lớn
nhất có thể có.
5) Giá trị đầu tiên nhận được nếu xét số nguyên tử carbon trong cấu trúc
polymer có nguồn gốc từ ethylene glycol:

2
(C) =  100% = 20%
2+8
Giá trị thứ hai nhận được nếu xét đến phần ethylene glycol -O-CH2-CH2-O-
trong cấu trúc mắt xích X (trên cơ sở cơ chế phản ứng ester hoá được làm
sáng tỏ bằng cách dùng các alcohol đánh dấu với đồng vị oxygen).

60
(moiety) =  100% = 31.25%
192
(moiety = phần cấu trúc)
6) Xét khối lượng riêng dầu là 0.85 kg/L (đây là một giá trị ước lượng! Giá
trị thực tế nằm trong khoảng 0.82 đến 0.95 kg/L, tuỳ thuộc vào từng
công ty; nhìn chung giá trị trong khoảng 0.6-1 kg/L là được chấp nhận)
và hàm lượng carbon trong dầu là 85 % (được tính với octane; mọi giá
trị trong khoảng 83-85 % được chấp nhận), khối lượng carbon trong
lượng dầu được tiết kiệm là
m(C) = 445000 bl ∙ 159 L/bl ∙ 850 g/L ∙ 0.85 ≈ 5∙1010 gam
Như trước đó đã biết, ethylene glycol chỉ góp phần tạo 1/8 số carbon trong
cấu trúc X, do đó khối lượng polymer tạo thành là

5  1010  5
m(X) =  4  1011 gam
0.63
388 | Bản quyền thuộc về Tạp chí Olympiad Hóa học KEM
hoặc khoảng 20 gam/chai (giá trị này gần với khối lượng của một chai PET
0.33 L).

389 | Bản quyền thuộc về Tạp chí Olympiad Hóa học KEM
Bài 10
Giữa thập niên 1960, một loại sợi có độ bền cao được chế tạo từ polymer
X (được điều chế qua phản ứng ngưng tụ trùng hợp hỗn hợp đẳng mol các
monomer B và F) đã được phòng thí nghiệm của hãng DuPont phát triển.
Việc có sự tạo thành liên các liên kết hydrogen liên phân tử mà loại sợi
này chống chịu tốt với các áp lực động (tốt hơn thép nhiều lần). Ngày nay,
loại sợi này được dùng để sản xuất những phần nhẹ của các thiết bị thể
thao, áo giáp, sợi cáp, … Loại sợi này vẫn giữ được các tính chất cơ học
xuất sắc trong khoảng nhiệt độ từ -96oC đến 420oC, cho phép nó được ứng
dụng trong việc phát triển các thiết bị chống cháy và đông lạnh.
B và F có thể nhận được qua sự tái sinh các chai nhựa dẻo làm từ
polyethylene terephthalate (PET).

1) Xác định các chuyển hoá trong tái sinh PET.


2) Vẽ cấu trúc mắt xích của X; b) Xác định độ trùng hợp hoá trung bình n
(số mắt xích) trong polymer được tạo thành từ B và F dưới các điều
kiện cân bằng. Hằng số cân bằng của phản ứng giữa các nhóm chức
trong B và F là Keq = 106.
3) Cần bao nhiêu chai nhựa (20 gram mỗi chai) dùng tái sinh để sản xuất
một bộ áo giáp chứa 2 kg X, biết mỗi giai đoạn trong sơ đồ trên đạt
hiệu suất 90 % (xem chuyển hoá PET → A là quá trình một giai đoạn)?
4) Xác định các liên kết hydrogen được tạo thành giữa các chuỗi polymer
(vẽ ít nhất 2 mắt xích trong mỗi chuỗi). Tính số liên kết hydrogen trên
mỗi mắt xích.
X bền khi tiếp xúc với các môi trường trung tính và kiềm nhưng bị thủy
phân trong acid.
5) Tính các khối lượng của các thành phần bảo vệ kích thước 60cm‧60
cm của các bộ áo giáp được làm từ thép và sợi X, cả hai đều có khả
năng chống được viên đạn 4.7 gram với đường kính 5 mm di chuyển ở
tốc độ 247 m/s theo hướng vuông góc với bộ áo giáp. Cho biết khối
390 | Bản quyền thuộc về Tạp chí Olympiad Hóa học KEM
lượng riêng của thép và sợi X lần lượt là 7.7 và 1.44 gram/cm3. Kích
2
V
thước của vết đạn bắn được tính bởi công thức: L =   m    trong
d
đó d, V, m lần lượt là đường kính, tốc độ và khối lượng của viên đạn.
Giá trị của hệ số α của thép và X lần lượt là 5.84‧10-10 và 1.35‧10-9
m.s2/kg.
Hướng dẫn
1)

2) a) Phản ứng trùng ngưng của p-phenylenediamine (F) và terephthaloyl


chloride (B) tạo thành poly(pphenylene terephthalamide). Đây là sợi
Kevlar.

b) Hằng số cân bằng của phản ứng tạo thành liên kết amide:

K=
 −NHCO −HCl = 106
 −NH2  −COCl

391 | Bản quyền thuộc về Tạp chí Olympiad Hóa học KEM
Đặt số mol ban đầu của các nhóm amine và acyl chloride là x mol (nhớ
rằng hỗn hợp B và F là đẳng mol) và độ chuyển hóa thành các liên kết
x  x
amide là α, ta có: = 106   = 0.999
(1 − )  x (1 − )  x
Độ chuyển hóa 99.9% nghĩa là 999 liên kết amide tạo thành nằm cân bằng
với một cặp nhóm -NH2 và -COCl chưa phản ứng. Các nhóm này được tìm
thấy ở các đầu mạch của đại phân tử tạo thành. Cuối cùng, mỗi mắt xích
chứa 2 liên kết amide, độ trùng hợp trung bình của Kevlar (n) là 500.
3) Khối lượng mắt xích của X là 238.25 g/mol. Do đó, 2 kg X được tạo
thành từ 2000/238.25 = 8.39 mol mỗi chất B và F. Cả B và F đều nhận
được từ phần terephthalate của PET, cần phải được điều chế từ các mẻ
PET khác nhau. B được tạo thành từ PET theo 2 giai đoạn, để điều chế
8.39 mol B cần 8.39/0.92 = 10.36 mol mắt xích PET. Tương tự, 8.39 mol F
được điều chế theo 3 giai đoạn từ 8.39/0.93 = 11.51 mol mắt xích PET.
Tổng lại thì cần 21.87 mol. Nếu đến khối lượng mol của mắt xích PET là
192.17 g/mol thì cần 4202 g (khoảng 210 chai) PET.
4) Mỗi mắt xích tạo thành 4 liên kết hydrogen:

5) Độ dày của các thành phần bảo vệ bằng độ sâu của viên đạn găm vào.
1.35  10−9  4.7  10−3  (247)2
Với Kevlar: L = = 0.0155 m = 1.55 cm
(5  10−3 )2

5.84  10−10  4.7  10−3  (247)2


Trong khi đó với thép: L = = 0.67 cm
(5  10−3 )2
Khối lượng tương ứng của các thành phần bộ áp giáo là
mthép = 60∙ 60 ∙ 0.67 ∙ 7.7 = 18.6 kg

392 | Bản quyền thuộc về Tạp chí Olympiad Hóa học KEM
mKevlar = 60 ∙ 60 ∙ 1.55 ∙ 1.44 = 8.04 kg
Bài 11
Ô nhiễm nhựa được định nghĩa là sự tích tụ các sản phẩm nhựa trong môi
trường, gây ra tác động tiêu cực tới đời sống động vật và con người. Vấn
đề này là một trong những thách thức quan trọng nhất mà ngày nay nhân
loại phải đối mặt. Đáng ngạc nhiên thay, đôi khi thiên nhiên lại tự có những
hỗ trợ, như là trị liệu sinh học2. Các nhà nghiên cứu Nhật Bản gần đây đã
phát hiện ra rằng, vi khuẩn Ideonella sakaiensis có thể sử dụng
polyethylene terephthalate (PET), một polymer có độ kháng cực kì cao với
sự phân huỷ sinh học, làm nguồn carbon và năng lượng.

Sự phân huỷ sinh học PET gồm hai giai đoạn xúc tác bởi enzyme
hydrolases, tạo thành 3 sản phẩm có khối lượng phân tử thấp (M < 600
gam.mol-1) A, B, C có số loại hydrogen tương ứng là 2, 4, 6. Sau khi thâm
nhập vào bên trong các tế bào Ideonella sakaiensis, C bị chuyển hoá thành
A, trở thành nguồn carbon duy nhất cho vi khuẩn.
1) Sử dụng kí hiệu mũi tên, chỉ ra các liên kết hoá học trong công thức
PET bị phân cắt trong quá trình phân huỷ sinh học, xúc tác bởi
hydrolase.
2) Vẽ tất cả các cấu tạo có thể có của A, B, C thoả mãn với dữ kiện ở trên.
A trải qua một chuyển hoá trao đổi chất xúc tác enzyme, 3 giai đoạn (đều
là phản ứng hoá học.)

X là một nhóm chức trong A3, NADP+ và NADPH là các dạng co-enzyme
bị oxide hoá và khử hoá tương ứng.

2
Bioremediation: Việc sử dụng các sinh vật sống để làm sạch dầu tràn hay loại bỏ các
chất ô nhiễm khỏi đất, nước, nước thải; việc sử dụng các sinh vật như côn trùng có ích
để tiêu diệt các côn trùng có hại cho nông nghiệp hay chống lại bệnh tật cho cây cối.
393 | Bản quyền thuộc về Tạp chí Olympiad Hóa học KEM
3) Suy ra công thức của A3 nếu biết: phần mol của oxygen và hydrogen
bằng nhau và thấp hơn carbon.
4) Xác định công thức cấu tạo của A1, A2.
5) Chọn ra phát biểu đúng về Ideonella sakaiensis trong Phiếu trả lời. (Bỏ
qua)
Các báo cáo chỉ ra rằng tốc độ phân huỷ sinh học những lớp màng phủ,
chế tạo từ PET, bởi một khuẩn lạc được phân lập với số lượng tế bào không
đổi là 0.13 mg∙cm-2∙ngày-1.
6) Hãy tính xem cần bao nhiêu ngày để khuẩn lạc phân huỷ 1 % khối lượng
của các hạt PET hình cầu có đường kính 1 cm và khối lượng riêng 1.45
gam.cm-3.
Khả năng thuỷ phân PET trước đây được cho là bởi cutinase, một loại
enzyme tự nhiên, tham gia vào quá trình phân huỷ tương tự của cutin
(polymer sinh học, thành phần chính của lớp biểu bì bao phủ lá cây). Hợp
chất D (70.54 % C; 11.84 % H và 17.62 % O về khối lượng) là monomer
chính trong quá trình sinh tổng hợp cutin.
7) Dự đoán cấu trúc của D (M < 300 gam.mol-1) nếu biết nó là hợp chất
không phân nhánh, không có carbon chiral.
8) Vẽ đơn vị cấu trúc mắt xích của polymer tạo thành từ D.
Hướng dẫn
1) Mục tiêu thuỷ phân duy nhất trong cấu trúc PET là các liên kết ester:

2) Chỉ có các liên kết ester bị thuỷ phân, nên chỉ có các nhóm hydroxyl
hoặc carboxylic ở đầu mạch A, B, C. Số loại hydrogen trong các sản
phẩm cho phép kết luận rằng chúng là những hợp chất có tính đối xứng
cao. Do giới hạn về khối lượng phân tử, chỉ có duy nhất 1 cấu trúc của
A (terephthalic acid), 2 cấu trúc của B và 3 của C.

394 | Bản quyền thuộc về Tạp chí Olympiad Hóa học KEM
Các sản phẩm B, C tìm thấy trong thực tế được đánh số (1).
Trong A3, số nguyên tử O và H bằng nhau (tương đương với phần mol
bằng nhau.) Khung sườn cấu trúc A3 (ngoài hai nhóm chức X) chứa 2
nguyên tử O và 4 nguyên tử H. Giả sử X có x nguyên tử O và y nguyên tử
H, khi đó: 2x + 2 = 2y + 4, hay x = y + 1. Phần mol của C lớn nhất, cho thấy
trong X có chứa carbon. Kết hợp tất cả các dữ kiện này, suy ra rằng X là
nhóm carboxylic (-COOH). Công thức cấu tạo của A3 là

3) Biểu diễn lại quá trình chuyển hoá A → A3

4) Tất cả các giai đoạn trong sơ đồ đều là phản ứng hoá học; công thức
phân tử của các tiểu phân trung gian là A1 (C8H8O6) và A2 (C7H6O4).
Công thức cấu tạo tương ứng có thể xác định dựa vào các cơ chế phản
ứng:

Dưới đây là một hướng chuyển hoá lí thuyết khác, nhưng sản phẩm cuối
không thoả mãn.
395 | Bản quyền thuộc về Tạp chí Olympiad Hóa học KEM
5) Ideonella sakaiensis sử dụng oxygen phân tử trong ít nhất hai giai
đoạn của chuyển hoá terephthalic acid, chứng tỏ nó có đặc tính hiếu
khí. Sự thâm nhập của C vào trong tế bào từ không gian ngoại bào cho
thấy ban đầu sự thuỷ phân PET xúc tác bởi một enzyme tiết ra ở ngoài
tế bào.
6) Diện tích của một bề mặt hạt PET riêng lẻ là S = 4·π·r2 =3.14 cm2.
Mỗi ngày, lạc khuẩn Ideonella sakaiensis làm giảm: 3.14 cm2·0.13 mg·cm-
2
·ngày-1·1 ngày = 0.41 mg.
Khối lượng ban đầu của hạt PET là 754 mg, và 1 % tương đương với 7.54
mg. Bỏ qua sự biến đổi diện tích bề mặt hạt, để làm giảm 1 % khối lượng
hạt thì lạc khuẩn cần 7.54/0.41 ≈ 18 ngày.
7) Từ công thức đơn giản nhất
70.54 11.84 17.62
n(C) : n(H) : n(O) = : :  16 : 32 : 3
12.01 1.008 16.00
và giới hạn khối lượng phân tử, có thể kết luận công thức phân tử của D là
C16H32O3. Cutinase thuỷ phân các liên kết ester trong PET. Do đó, có thể
giả thiết rằng cutin cũng chứa các liên kết ester. Có thể dùng mỗi D là
monomer để tạo ra cutin, do vậy D phải chứa cả nhóm chức carboxylic và
hydroxyl. Kết hợp với các dữ kiện khác, chỉ có duy nhất 1 cấu tạo thoả mãn

8) Mắt xích tương ứng là

396 | Bản quyền thuộc về Tạp chí Olympiad Hóa học KEM
Bài 12
Phản ứng trùng hợp mô phỏng lại một sự lắp ráp các phân tử. Quá trình
cơ bản này bao gồm việc kết nối các thành phần kéo dài mạch vào thành
phần lõi để tạo thành một đại phân tử với cấu trúc và khối lượng nhất định.
Thành phần lõi có thể là các phân tử đơn chức hoặc đa chức (phải tương
đồng hoàn toàn), còn thành phần kéo dài mạch là phân tử nhỏ có một (và
chỉ một) nhóm chức có khả năng ngưng tụ với thành phần lõi. Chẳng hạn
monoalcohol ROH có thể đóng vai trò là thành phần lõi, và H2NCH2CH2CN
có thể đóng vai trò là thành phần kéo dài mạch. Sơ đồ tổng hợp loại polime
này như sau:
(a) ROH + CH2=CHCN → ROCH2CH2CN → ROCH2CH2COOH
→ROCH2CH2COCl
(b) ROCH2CH2COCl + H2NCH2CH2CN → ROCH2CH2CONHCH2CH2CN
(c) lập lại n lần: ROCH2CH2CO(NHCH2CH2CO)nNHCH2CH2CN
(d) Cuối cùng là phản ứng thuỷ giải, ester hoá để tạo thành một loại
polymer bền. Để làm tăng khối lượng của phân tử polymer, người ta sử
dụng các thành phần lõi và thành phần kéo dài mạch đa hướng. Chẳng
hạn:
Thành phần lõi hai hướng: (CH2OCH2CH2COCl)2
Thành phần kéo dài mạch hai hướng: HN(CH2CH2OCH2CH2CN)2
1) Hãy viết công thức của polymer được tạo thành từ thành phần lõi hai
hướng ngưng tụ 1 lần, 2 lần và n lần với thành phần kéo dài mạch đơn
hướng.
2) Xác định các nguyên liệu thích hợp để điều chế thành phần kéo dài hai
hướng sau: HN(CH2CH2OCH2CH2CN)2.
3) Hãy viết công thức polymer được tạo thành từ các thành phần lõi và
kéo dài mạch hai hướng đã cho: (CH2OCH2CH2COCl)2 và
HN(CH2CH2OCH2CH2CN)2 sau khi ngưng tụ lần 1 và lần 2.
4) Hãy cho biết nghĩa của từ “kéo dài” trong thành phần kéo dài.
5) Tại sao sự trùng hợp đa hướng lại cần điều kiện các thành phần lõi và
thành phần kéo dài có các nhóm chức “tương đồng về hoàn toàn” về
mặt hoá học. Cho biết glycerol có thể đóng vai trò thành phần lõi và
397 | Bản quyền thuộc về Tạp chí Olympiad Hóa học KEM
Beta-glucosamine có thể đóng vai trò thành phần kéo dài trong phản
ứng trùng hợp đa hướng không ?
6) Trong phản ứng trùng hợp đa hướng, khối lượng phân tử tăng rất
nhanh. Hiện nay, bằng việc tiến hành trùng hợp giữa một thành phần
lõi 4 hướng và thành phần kéo dài 3 hướng, người ta có thể thu được
một polimer có trọng lượng phân tử lên tới 60604. Hãy cho biết công
thức của một thành phần lõi 4 hướng và thành phần kéo dài 3 hướng,
và sơ đồ điều chế chúng từ các hợp chất hữu cơ đơn giản (<3 carbon).
Hướng dẫn
1) Công thức các polymer:
(CH2OCH2CH2CONHCH2CH2COOR)2
(CH2OCH2CH2CONHCH2CH2CONHCH2CH2COOR)2
[CH2OCH2CH2CO(NHCH2CH2CO)nOR]2
2) Con đường tổng hợp:
NH3 + 2(CH2)2O → HN(CH2CH2OH)2

HN(CH2CH2OH)2 + 2CH2=CHCN ⎯⎯⎯


NaOH
→ NH(CH2CH2OCH2CH2CN)2
3) Công thức polymer được tạo thành:

Công thức polymer được tạo thành sau lần ngưng tụ thứ hai

4) Chữ ”kéo dài” muốn nói đến khối lượng phân tử (hay kích thước phân
tử) của polymer tăng luỹ tiến theo từng giai đoạn phản ứng.
5) Nếu các nhóm chức không tương đồng về mặt hoá học thì khả năng
phản ứng của mỗi nhóm sẽ khác nhau, điều đó không đảm bảo việc
phản ứng đồng thời ở các trung tâm hoạt động, cũng như không thể
kiểm soát tốt việc tăng trọng lượng phân tử. Glycerol không thê được
398 | Bản quyền thuộc về Tạp chí Olympiad Hóa học KEM
sử dụng để làm thành phần lõi trong phản ứng polymer đa hướng vì
ba nhóm hydroxyl của nó không tương đồng hoàn toàn về mặt hoá
học. Tương tự, Beta-glucosamine không thể được sử dụng làm thành
phần kéo dài mạch do bốn nhóm hydroxyl của nó không tương đồng
hoàn toàn về mặt hoá học.
6) Thành phần lõi 4 hướng: C(CH2OH)4

CH3CHO + 3HCHO ⎯⎯⎯⎯


NaOH 10%
→ (HOCH2)3CCHO ⎯⎯⎯⎯
NaOH 33%
HCHO
→ C(CH2OH)4

Thành phần kéo dài mạch 3 hướng: H2NCH2C(CH2OCH2CH2CN)3

(HOCH2)3CCHO ⎯⎯⎯
1) NH3
2) H /Ni
→ H2NCH2C(CH2OH)3
2

⎯⎯⎯⎯NaOH
3CH2C =CHCN
→ H2NCH2C(CH2OCH2CH2CN)3

399 | Bản quyền thuộc về Tạp chí Olympiad Hóa học KEM
Bài 13
Phương trình mô tả sự phụ thuộc của áp suất thẩm thấu π vào nồng độ
của dung dịch c có dạng tương tự như phương trình khí lí tưởng (phương
trình Clapeyron). Cũng như phương trình Clapeyron, việc mô tả mối liên hệ
p(V) chỉ được thỏa mãn với các khí có khối lượng riêng thấp, phương trình
π(c) chỉ mô tả tốt các dung dịch rất loãng (c → 0). Nghĩa là sự phụ thuộc
áp suất thẩm thấu vào nồng độ của dung dịch thực không có dạng tuyến
tính và thể hiện độ lệch đáng kể so với phương trình quen thuộc.
Áp suất thẩm thấu của các dung dịch polymer có nồng độ đã biết được
đo đối chứng với áp suất thẩm thấu của dung môi tinh khiết nhằm tìm ra
khối lượng phân tử trung bình, Mav, của polymer hòa tan. Tuy nhiên, do các
nồng độ khác nhau sẽ thường nhận được các kết quả đo khối lượng phân
tử khác nhau, nên cần thực hiện nhiều phép đo với các nồng độ khác nhau,
ci, rồi tính các giá trị khối lượng phân tử biểu kiến, Mi, và ngoại suy các giá
trị với nồng độ c = 0. Sự phụ thuộc của M(c) rõ ràng không tuyến tính, do
đó nếu chúng ta không biết được mối liên hệ chính xác của M(c), thì một
phép ngoại suy đơn giản các giá trị Mi đến c = 0 là không thể.
Một phản ứng trùng hợp gốc của styrene đã được thực hiện để xác định
sự phụ thuộc của khối lượng phân tử polymer vào thời gian tiến hành trùng
hợp. Một lượng 10.414 gram styrene đã loại khí (degas) và không có chất
ức chế, cùng một lượng nhỏ benzoyl peroxide (với tỉ lệ mol 1:100) -đóng
vai trò chất khơi mào cho phản ứng trùng hợp gốc - được cho vào bình
dưới khí quyển argon tinh khiết. Phản ứng được tiến hành ở 120 oC và và
sau một quãng thời gian phù hợp, hỗn hợp phản ứng được làm nguội
nhanh chóng trong băng để dừng phản ứng.
Hỗn hợp sau phản ứng được chia thành nhiều mẫu để tiếp tục thực hiện
các thí nghiệm tiếp theo. Một mẫu có khối lượng 5.678 gram được làm
bay hơi trong chân không ở nhiệt độ đủ thấp, thu được 3.456 gram polymer
khô. Mỗi mẫu trong 4 mẫu tiếp theo của hỗn hợp sau phản ứng, có cùng
nồng độ polymer c, và có khối lượng, m, liệt kê trong Bảng 1 được hòa tan
vào toluene trong bình định mức rồi pha loãng toluene tới thể tích 100
cm3. Các dung dịch được dùng cho các phép đo áp suất thẩm thấu với
dung môi tinh khiết (nghĩa là toluene). Chênh lệch về độ cao của các chất
lỏng trong hai nhánh của thẩm áp kế (máy đo áp suất thẩm thấu), h, đã
được đo lại. Các kết quả được tóm tắt trong Bảng 1.

400 | Bản quyền thuộc về Tạp chí Olympiad Hóa học KEM
Bảng 1
Các thí nghiệm được tiến hành ở t = 30.85 oC. Khối lượng riêng của toluene
ở nhiệt độ này là d = 0.8566 g∙cm-3 và có thể giả sử rằng khối lượng riêng
của các dung dịch polymer là giống nhau. Do đường kính rất nhỏ của các
mao quản trong các nhánh của thẩm áp kế, chúng ta bỏ qua ảnh hưởng
của sự pha loãng dung dịch bởi dung môi dẫn qua màng bán thấm; các
giá trị h trong Bảng 1 đã được hiệu chỉnh với hiệu ứng mao dẫn.
Chú ý: Benzoyl peroxide bị phân hủy thành benzene trong phản ứng; trong
các phép đo thẩm thấu, có thể giả sử rằng các phân tử styrene và toluene
có kích thước giống nhau. Trong tính toán, sử dụng các giá trị khối lượng
mol sau (g∙mol-1): C - 12.010, H - 1.008; giá trị của gia tốc trọng trường là
g = 9.81 m/s2.
a) Với cả 4 dung dịch đã cho, hãy tính nồng độ c theo g∙dm-3 của polymer
và áp suất thẩm thấu π, được xác định trong các phép đo riêng biệt.
b) Vẽ đồ thị của hàm π(c), trong đồ thị phải thể hiện điểm ở c = 0.
c) Từ phương trình mô tả hàm π(c), dẫn ra công thức sự phụ thuộc của
khối lượng phân tử, M, của polymer vào π, c và T.
d) Tính khối lượng phân tử Mi của polymer ở mỗi (trong số 4) thí nghiệm.
e) Tìm hàm f(M), vốn là một hàm tuyến tính của nồng độ c và cho phép
ngoại suy f(M) tới c = 0. Tính các giá trị của hàm f(Mi) với mỗi nồng độ
riêng của polystyrene.
Chú ý: Phân tích các kết quả cho thấy rằng sự phụ thuộc của áp suất thẩm
thấu của các dung dịch polystyrene vào nồng độ của chúng có dạng: π(c)
= A∙c + B∙c2.
f) Tương tự ý b, hãy biểu diễn ở dạng đồ thị các giá trị tính được của các
giá trị f(Mi) ở dạng hàm số của nồng độ c. Vẽ đường thẳng đi qua tất
cả các điểm và ngoại suy hàm tuyến tính này tới nồng độ c = 0. (Một
đường thẳng đạt yêu cầu là đường thẳng có khoảng cách đến các
401 | Bản quyền thuộc về Tạp chí Olympiad Hóa học KEM
điểm là bé nhất, hay còn gọi là “đường trung bình”. Bài này yêu cầu vẽ
một đường như thế.)
g) Từ 2 điểm xa nhất của đồ thị f(M) được vẽ ở dạng hàm số của nồng
độ c (đồ thị từ ý f), tính các thông số “a” và “b” của đường thẳng lấy
gần đúng các điểm thực nghiệm và được dùng để ngoại suy đến nồng
độ c = 0. Tính giá trị hàm f(M) với c = 0 và giá trị của khối lượng phân
tử trung bình, Mav, của polystyrene.
Gợi ý: Thay vì tính các thông số của đường thẳng thông qua 2 điểm được
chọn, bạn có thể áp dụng với cả 4 điểm chức năng hồi quy tuyến tính có
sẵn trong máy tính.
h) Tính độ trùng hợp hóa của polystyrene, là tỉ lệ của khối lượng phân tử
trung bình tính được của polystyrene Mav với khối lượng phân tử của
monomer Mm. Tính mức độ diễn ra phản ứng, λ, đạt được trong hệ ở
thời điểm “đóng băng” phản ứng trùng hợp.
Chú ý: Mức độ diễn ra phản ứng, λ, được định nghĩa là tỉ lệ số mol chất
phàn ứng hoặc sản phẩm được dùng hoặc được tạo thành trong phản ứng
cho đến thời điểm khi phản ứng được dừng lại với hệ số tỉ lượng phù hợp:
λ = (nz - n0)/z
Trong đó: nz - số mol của chất phản ứng cụ thể z ở thời điểm dừng phản
ứng, n0 - số mol ban đầu của chất phản ứng (với sản phẩm n0 = 0), z - hệ
số tỉ lượng của phản ứng (với chất phản ứng, phải lấy giá trị âm.)

402 | Bản quyền thuộc về Tạp chí Olympiad Hóa học KEM
Hướng dẫn
Bảng 2

a) Từ khối lượng của hỗn hợp sau phản ứng chứa trong V = 100 cm3 cuả
dung dịch và biết tỉ lệ của khối lượng polymer với khối lượng hỗn hợp sau
phản ứng, chúng ta tính nồng độ c:

Áp suất thẩm thấu, π, trong các thí nghiệm là áp suất thủy tĩnh của cột
dung dịch xuất hiện trong mao quản của thẩm áp kế với độ cao h. Ta sử
dụng công thức:

Trong đó d là khối lượng riêng của dung dịch ở nhiệt độ xác định (ở đây là
khối lượng của toluene), g là gia tốc trọng trường, h là chênh lệch độ cao
của cột chất lỏng trong các nhánh của thẩm áp kế. Các giá trị c và π tính
được (sau khi chuyển đổi đơn vị) được điền vào Bảng 2 ở trên.

403 | Bản quyền thuộc về Tạp chí Olympiad Hóa học KEM
Hình 1: Sự phụ thuộc của áp suất thẩm thấu của nồng độ của các dung
dịch polystyrene.
b) Đồ thị π(c) sẽ có dạng như trong Hình 1.
c) Dữ kiện đã cho trong bài là: nồng độ của polystyrene, c [g∙dm-3], áp suất
thẩm thấu π [Pa], nhiệt độ t [oC]. Nhiệt độ của thí nghiệm theo thang Kelvin
là T = 30.85 oC + 273.15 K = 304.00 K. Áp suất khí quyển π là hàm của
nồng độ mol, cm, được mô tả bởi công thức:

Nồng độ mol, cm, được tính là: cm = c/M, trong đó M là khối lượng mol
[g∙mol-1] của polystyrene. Thế biểu thức trên vào (3), ta có:

Chuyển (4) về dạng “M là hàm của c”:

d) Xem Bảng 2.
e) Dựa vào biểu thức đã cho: π(c) = A∙c + B∙c2, ta có thể bình đẳng hóa liên
hệ này với liên hệ được mô tả bởi (4):

Sau khi chia cả hai vế của (6) cho c (với giả định rằng c ≠ 0), có thể nhận
thấy rằng nghịch đảo của khối lượng phân tử cần phải được mô tả bởi một
đường thẳng:

Trong đó: a = A/R∙T và b = B/R∙T


Hàm số ta đang tìm là f(M) = M-1(c). Chúng ta có thể tính các giá trị nghịch
đảo của Mi và đưa vào Bảng 2.
f)

404 | Bản quyền thuộc về Tạp chí Olympiad Hóa học KEM
Hình 2: Sự phụ thuộc của nghịch đảo khối lượng mol biểu kiến, 1/M(c)
vào nồng độ polystyrene.
g) Mav = 198700 g∙mol-1.
a = 0.5025∙10-5 mol∙g-1
b = 0.1538∙10-5 mol∙g-1 ∙K-1
h) Phản ứng trùng hợp:

Khối lượng mol của styrene là is Mm = 104.144 g∙mol-1. Độ trùng hợp của
styrene là:

Mức độ diễn tiến của phản ứng, λ:

Số mol styrene chưa phản ứng còn lại:

Do đó:

405 | Bản quyền thuộc về Tạp chí Olympiad Hóa học KEM
Bài 14
Các đại phân tử polyelectrolyte (polymer điện li) có chứa đồng thời cả
nhóm chức có tính acid và base gọi là polyampholyte, mà một trong những
ví dụ thường gặp nhất là protein. Giá trị pH của dung dịch polyampholyte,
mà tại đó điện tích toàn phần của đại phân tử bằng 0, gọi là điểm đẳng
điện (IEP). Các nhóm tạo ion của protein, như carboxylic (COOH) và amino
(NH2), có thể được biến đổi bởi phản ứng amide hoá, chuyển chúng thành
các nhóm amide không phân li.
1) Viết các phương trình phản ứng amide hoá của a) nhóm amino; b)
nhóm carboxylic của protein.
Protein B có các nhóm tạo ion sau đây trên bề mặt: 12 nhóm amino và 7
nhóm carboxylic. Không có gốc histidine hoặc arginine nào trên bề mặt.
pK(NH2) = 11 và pK(COOH) = 3.
2) Tính khoảng pH IEP(1) của protein B.
3) Tính khoảng pH IEP(2) của protein B sau khi amide hoá, khi a) 2 nhóm
amino trong B bị biến đổi; b) 10 nhóm amino trong B bị biến đổi.
Các polyampholyte có thể tham gia vào các phản ứng interpolyelectrolyte
(IPR, giữa các polymer điện li có điện tích trái dấu) tạo thành các phức
chất interpolyelectrolyte (IPC). Trong các IPC như vậy, các polymer điện li
có điện tích trái dấu gắn kết với nhau qua các cầu muối. Các nhóm mang
điện trái dấu của polyampholyte tạo thành cặp ion lưỡng cực và không
tham gia vào phản ứng IPC.
4) Nhóm nào của protein B (amino hay carboxylic) và với số lượng bao
nhiêu sẽ bị biến đổi bởi phản ứng amide hoá, làm cho protein tham gia
vào phản ứng IPR với các polymer điện li sau, tạo thành cầu muối trong
dung dịch.
a) sodium polystyrenesulfonate (PSSNa) ở pH = 2;
b) sodium polystyrenesulfonate (PSSNa) ở pH = 7;
c) poly-N-ethyl-4-vinylpyridinium bromide (PEVPB) ở pH = 2;
d) poly-N-ethyl-4-vinylpyridinium bromide (PEVPB) ở pH = 7;
e) polyacrylic acid (PAC) ở pH = 2;
f) polyacrylic acid (PAC) ở pH = 7.

406 | Bản quyền thuộc về Tạp chí Olympiad Hóa học KEM
Trong mỗi trường hợp, phản ứng amide hoá chỉ xảy ra với một loại nhóm
chức.
Hướng dẫn
1) a) R1NH2 + (R2CO)2O → R1NHCOR2 + R2COOH
b)

2) Do số nhóm amino dư so với số nhóm carboxylic nên điện tích toàn


phần quyết định bởi 12 - 7 = 5 nhóm amino. Để ngăn sự proton hoá các
nhóm amino thì phải bổ sung kiềm, do vậy EPT(1) cần phải ở pH > 7.
3) a) pH 7 < IEP(2) < IEP (1).
b) IEP(2) < pH 7 < IEP (2).
4)
số nhóm amino bị số nhóm carboxyl
biến đổi bị biến đổi
Câu hỏi PE pH
nhỏ lớn nhỏ lớn
nhất nhất nhất nhất
a PSSNa 2 0 11 0 7
b PSSNa 7 0 4 0 7
c PEVPB 2 - - - -
d PEVPB 7 6 12 - -

407 | Bản quyền thuộc về Tạp chí Olympiad Hóa học KEM
e PAC 2 - - - -
f PAC 7 0 4 0 7

408 | Bản quyền thuộc về Tạp chí Olympiad Hóa học KEM
Bài 15
Polymer điện li là những polymer có chứa các nhóm chức có khả năng
phân li thành ion. Các polymer điện li có thể trao đổi ion với các chất tan
(polycationite - cation, polyanionite - anion) và có thể được sử dụng làm
chất trao đổi ion.
1) Hoàn thành các sơ đồ tổng hợp polycationite B và polyanionite E.

2) Viết cân bằng được thiết lập bởi tương tác giữa polymer điện li E với
dung dịch NaCl.
3) Phản ứng sulfonate hóa polystyrene tạo thành polycationite với độ
trùng hợp trung bình là 1000 và khối lượng phân tử trung bình là
116000 gram/mol. Xác định tỉ lệ phần trăm của các mắt xích bị biến
tính trong sản phẩm.
4) Đề xuất sơ đồ tổng hợp polyanionite từ polystyrene, không quá 3 giai
đoạn.
Phenol để điều chế nhựa phenol-formaldehyde thường có lẫn tạp chất
pyridine, khiến cho nhiều tính chất của sản phẩm bị phá hủy. Có thể làm
sạch phenol bằng phương pháp lọc qua chất trao đổi ion.
5) Trình bày sự tạo thành liên kết với pyridine của một trong các chất trao
đổi ion đã được đề cập đến trong câu hỏi này.
6) Xác định tất cả các nguyên tử trong phân tử phenol có thể kết hợp với
các phân tử khác trong nhựa phenol-formaldehyde.
7) Phản ứng của p-tertbutylphenol (C4H9)(C6H4)(OH) với formaldehyde
khi có mặt NaOH tạo thành sản phẩm F có khối lượng mol 648. Biểu
diễn cấu trúc của F ở dạng mắt xích.

409 | Bản quyền thuộc về Tạp chí Olympiad Hóa học KEM
Hướng dẫn
1)

2)

3)

M = 104 gram/mol
M = 184 gram/mol
Giả sử số mắt xích bị biến tính là x. Khối lượng 1000 mắt xích là 116000
gram/mol, khi đó 184x + 104(1 - x) = 116  x = 0.15 hay 15 %.
4)

5)

6)

410 | Bản quyền thuộc về Tạp chí Olympiad Hóa học KEM
7)

411 | Bản quyền thuộc về Tạp chí Olympiad Hóa học KEM
Bài 16
Đôi khi, tính chất của các polymer lại có sự khác biệt rõ rệt so với các chất
tương tự có phân tử khối thấp hơn. Các biến đổi hoá học của polymer có
thể chịu tác động của các hiệu ứng cấu hình (không gian), như là hiệu ứng
nhóm kề. Tác động này bao gồm sự thay đổi tốc độ và cơ chế phản ứng
với một tác nhân có phân tử khối nhỏ, do sự khác biệt về môi trường phản
ứng của nhóm chức ở thời điểm bắt đầu và trong quá trình phản ứng. Hãy
xét một polymer chứa các nhóm chức A, có thể chuyển thành các nhóm
chức B trong một lượng dư tác nhân có phân tử khối nhỏ, theo sơ đồ sau:

Có 3 loại tiểu phân trung gian chứa A:


a) có 2 nhóm A kế cận (AAA), nồng độ là c1;
b) có 1 nhóm A và 1 nhóm B kế cận (AAB), nồng độ là c2;
c) có 2 nhóm B kế cận (BAB), nồng độ là c3.
Phương trình động học chung là

dcA
− = k1c1 + k2c2 + k3c3
dt
trong đó cA = c1 + c2 + c3
Hãy xét một số trường hợp với các mối quan hệ khác nhau giữa các hằng
số phản ứng:
1) k1 = k2 = k3 không có hiệu ứng nhóm kề;
2) k1 < k2 < k3 làm tăng (gia tốc) hiệu ứng nhóm kề;
3) k1 > k2 > k3 làm giảm hiệu ứng nhóm kề
1) Tính tỉ lệ (%) giữa các mảnh AA, AB và BB ứng với độ chuyển hoá 50
% (khi ½ số nhóm A chuyển thành nhóm B) khi không có hiệu ứng
nhóm kề.
2) Những nghiên cứu về biến đổi hoá học tương tự của các polymer
tương đồng cấu trúc cho phép tìm được sự phụ thuộc động học của
nồng độ nhóm chức ban đầu (cA) vào thời gian. Xác định hiệu ứng
nhóm kề kèm theo sự chuyển hoá của mỗi polymer.

412 | Bản quyền thuộc về Tạp chí Olympiad Hóa học KEM
Sự phụ thuộc của nồng độ nhóm chức ban đầu (cA) vào thời gian.
3) Xét hai phản ứng, một trong số đó được đặc trưng bởi sự làm tăng
hiệu ứng nhóm kề (1), còn phản ứng kia là bởi sự làm giảm (2). Cả hai
phản ứng đều diễn ra theo sơ đồ ban đầu. Trình tự nào trong bảng dưới
đây sẽ phù hợp trong các trường hợp (1) và (2) ở các độ chuyển hoá
khác nhau.
Độ chuyển hóa 1 2 Độ chuyển hóa 1 2 Độ chuyển hóa 1 2
25 % 50 % 75 %
(AABBAAAA)n (BAAAABBB)n (BBBBABAB)n
(BAAAAAAB)n (ABABABAB)n (BBBBAABB)n
(AABABAAA)n (ABBBBAAA)n (ABBBBBBA)n

4) Phản ứng chlorine hoá 20.0 gam polyethylene tạo thành 44.6 gam
polymer X. Xác định độ chuyển hoá của phản ứng chlorine hoá
polyethylene mà tại đó polymer X được tạo thành. Hiệu ứng nhóm kề
nào kèm theo phản ứng tạo thành X?
5) Vẽ công thức cấu tạo của mắt xích trong sản phẩm X.
6) Vẽ công thức cấu tạo của polymer Y được tạo thành trong phản ứng
chlorine hoá polyethylene với độ chuyển hoá 50 % (giả sử rằng hiệu
ứng nhóm kề cũng giống như trong phản ứng tạo thành X).
Hướng dẫn
1) AA : AB : BB = 25 % : 50 % : 25 % do AB = BA
413 | Bản quyền thuộc về Tạp chí Olympiad Hóa học KEM
2) 1 - làm chậm; 2 - không ảnh hưởng; 3 - tăng tốc
3)
Độ chuyển hóa 1 2 Độ chuyển hóa 1 2 Độ chuyển hóa 1 2
25 % 50 % 75 %
(AABBAAAA)n ⁕ (BAAAABBB)n ⁕ (BBBBABAB)n ⁕
(BAAAAAAB)n ⁕ (ABABABAB)n ⁕ (BBBBAABB)n ⁕
(AABABAAA)n ⁕ (ABBBBAAA)n ⁕ (ABBBBBBA)n ⁕
4) Độ chuyển hóa 25 %
Hiệu ứng nhóm kề làm chậm do cản trở không gian.
5)

6)

414 | Bản quyền thuộc về Tạp chí Olympiad Hóa học KEM
Bài 17
Biến tính hóa học các polymer là phương pháp tiện lợi để giúp chúng có
những tính chất cần thiết, để phân tích cấu trúc và để nhận được các vật
liệu polymer mới. Chuỗi chuyển hóa có sự tham gia của các polymer có
sản lượng lớn B và Y được mô tả dưới đây:

Các phản ứng trùng hợp gốc monomer chứa halogen A (tỉ khối hơi so với
hydrogen là 31.00) tạo thành B - nguyên liệu polymer được sử dụng rộng
rãi. Những đặc tính hữu hiệu của B phụ thuộc vào độ điều hòa của cấu
trúc đại phân tử. Để xác định công thức, B được chuyển thành D bằng cách
đun nóng với bột kẽm trong dung môi trơ. Polymer D có thể chứa nhiều
mắt xích khác nhau, một trong số đó được cho trong giản đồ.
1) Xác định công thức phân tử của A.
2) Vẽ cấu trúc 3 “cặp đôi (diad) có thể có của B (các cặp mắt xích) khác
nhau về cấu trúc (không cần quan tâm đến hóa lập thể). Mắt xích của
D trong hình được tạo ra từ cặp monomer nào?
3) Xác định các mắt xích còn lại có thể có trong D.
Khi đun nóng B trong khí quyển trở, một polymer C có màu đậm và khí
không màu C1 có mùi hăng được tạo thành.
4) Xác định cấu trúc của C và C1.
Monomer Z (88.82 % carbon về khối lượng) được tạo thành từ ethanol khi
đun nóng có mặt nhôm và zinc oxide. Trùng hợp Z khi có mặt lithium trong
ether dầu hỏa tạo thành polymer Y điều hòa lập thể có cùng cấu hình như
cao su thiên nhiên.
5) Viết phương trình tổng hợp Z từ ethanol.
6) Xác định cấu trúc Y.

415 | Bản quyền thuộc về Tạp chí Olympiad Hóa học KEM
Chlorine hóa hoàn toàn Y trong bóng tối, sau đó xử lí với kẽm tạo thành
sản phẩm W không điều hòa lập thể là đồng phân của Y. Đun nóng hỗn
hợp C và W (200 oC) dẫn dến sự tạo thành sản phẩm N có liên kết chéo.
7) Xác định cấu trúc liên kết chéo giữa các chuỗi polymer trong cấu trúc
N.
8) Tính khối lượng lưu huỳnh cực tiểu cần để chuyển 100 gram mẫu Y
thành polymer có liên kết disulfide chéo biết khối lượng mol trung bình
là 54090 gram/mol.
Hướng dẫn
1) A là vinyl chloride (C2H3Cl).
2) Các “cặp đôi” có thể có là: đầu-đuôi, đầu-đầu, đuôi-đuôi.

Mắt xích của D như trong hình là trường hợp đầu-đuôi:

1) “Cặp đôi” đầu-đầu:

“Cặp đôi” đuôi-đuôi:

Cũng có thể là dạng liên kết chéo nội hoặc liên phân tử:

416 | Bản quyền thuộc về Tạp chí Olympiad Hóa học KEM
4) C1 là hydrogen chloride. C là:

5) Z là buta-1,3-diene: 2C2H5OH → C4H6 + 2H2O + H2


6) Y là poly (cis-1,4-butadiene):

7)

8) Cấu trúc sản phẩm lưu hóa:

100 g Y chứa 100/54090 g / mol = 1.85 · 10 -3 mole chuỗi polymer. Khối


lượng sulfur cần là 1.85 · 10-3 mol · 2 · 32.07 g / mol ≈ 0.12 g.

417 | Bản quyền thuộc về Tạp chí Olympiad Hóa học KEM
Bài 18
Bên cạnh các polymer truyền thống, còn có các polymer phối trí trong đó
chuỗi polymer được tạo thành bởi sự tham gia của các liên kết phối trí kim
loại-phối tử. Những polymer như vậy được tạo thành, ví dụ, bởi tương tác
giữa chất X với nickel acetylacetonate Ni(acac)2 hoặc với HgBr2.

1) Vẽ cấu trúc đơn vị mắt xích của polymer phối trí X·HgBr2 và X·Ni(acac)2
với đa diện được tạo thành bởi các phối tử xung quanh nguyên tử kim
loại.
2) Các polymer phối trí có thể được tạo thành từ các chất sau không: a)
2,2'-bipyridyl; b) 4,4'-bipyridyl?

Dung dịch methanol của copper(II) hexafluoroacetylacetonate được đổ


lên bề mặt dung dịch X trong chloroform. Một polymer phối trí được tạo
thành ở dạng các tấm màu xanh lục. Theo kết quả phân tích nguyên tố,
sản phẩm chứa 47.92 % carbon, 6.50 % đồng và 1.96 % hydrogen.
3) Xác định công thức phân tử hợp chất tạo thành.
Để thu được hợp chất X, phản ứng Sonogashira đã được sử dụng, bao
gồm tương tác của aryl halide ArHal với acetylene đầu mạch HC≡CR khi
có mặt base và một lượng xúc tác muối copper(I) và các phức
palladium(II). Trong trường hợp này, hoạt tính của các aryl halide càng
mạnh nếu liên kết C-Hal càng yếu.
4) Hoàn thành sơ đồ tổng hợp chất X sau:

418 | Bản quyền thuộc về Tạp chí Olympiad Hóa học KEM
5) Tính lượng polymer có thể nhận được từ 113.2 gam ortho-
bromiodobenzene, nếu sự tạo thành polymer phối trí giữa X và HgBr2
có hiệu suất 64 %.
Hướng dẫn
1) X·HgBr2

X·Ni(acac)2

2) a) Không. Trong trường hợp của 2,2'-bipyridyl một phức càng bền hơn
được tạo thành:

b) Có. 4,4'-bipyridyl không thể tạo thành phức càng. Do đó, nó tạo thành
polymer phối trí.
3) С39H19Cl3CuF12N2O4.
4)

419 | Bản quyền thuộc về Tạp chí Olympiad Hóa học KEM
5) Phân tử khối của o-bromiodobenzene là 282.90. Phân tử khối của
polymer X·HgBr2 là 740.85. Khối lượng polymer nhận được là (0.61 · 0.49
· 1 · 0.61 · 0.64 · 113.2 · 740.85) / (282.9 · 2) = 17.3 gram.

420 | Bản quyền thuộc về Tạp chí Olympiad Hóa học KEM

You might also like